[河北] 切换
河北省学业规划中心 系统讲解

logo

专属河北报考与数据查询一站式平台

  • 1. 普通高校招生有哪些形式?
  • 2. 普通高校招生的主要环节有哪些?
  • 3. 普通高考文化总成绩是如何组成的?
  • 4. 招生计划是如何编制的?什么是历史科目组合、物理科目组合?
  • 5. 什么是強基计划?
  • 6. 强基计划试点高校有哪些?
  • 7. 强基计划如何招生?
  • 8. 强基计划的报考流程是什么?
  • 9. 什么是强基计划招生 “十严禁”?
  • 10. 综合评价如何招生?
  • 11. 普通高校是怎样分类的?
  • 12. 公办院校、民办院校与独立学院有哪些区别?
  • 13. 什么是本硕、本硕博和本博连读?
  • 14. 什么是本科学术互认课程(ISEC)?
  • 15. 哪些高校(专业)对考生有特殊要求?
  • 16. 普通体育招生、高水平运动队招生、体育单招有什么区别?
  • 17. 体育单招如何招生?
  • 18. 高水平运动队如何招生?
  • 19. 艺术类专业考试形式有哪些?
  • 20. 什么是艺术类专业考试“考评分离”?
  • 21. 什么是“定向就业招生”?
  • 22. 什么是国家专项计划?
  • 23. 什么是高校专项计划?
  • 24. 什么是地方专项计划?
  • 25. 什么是公费师范生?
  • 26. 什么是优师专项计划?
  • 27. 什么是免费医学定向计划?
  • 28. 什么是对口升学考试?
  • 29. 对口升学考试与普通高校招生考试能否兼报?
  • 30. 什么是高职单招?怎样报考?
  • 31. 中等职业学校对口升学考试与高职單招考试有什么区别?
  • 32. 保送生有哪些条件?
  • 33. 什么是高校招生“阳光工程”?
  • 34. 考生档案有哪些内容和作用?
  • 35. 香 港 、 澳 门 的 高 校 如 何 招 生 ?
  • 36. 高考各阶段考生密码有哪些重置办法?
  • 37. 考生如何查看自己的高考报名及体检信息?
  • 1. 哪些人员需要参加高考报名?
  • 2. 考生在我省参加高考报名须具备什么条件?
  • 3. 哪些人员不能参加高考报名?
  • 4. 外省进城务工人员随迁子女在我省参加高考报名须 具备哪些条件?
  • 5. 考生报考 “三个专项”计划,须具备哪些条件?
  • 6.“ 三个专项”计划在我省实施区域和范围有哪些?
  • 7. 考生报考免费医学定向计划须具备哪些条件?
  • 8. 考生报名时需要提供哪些材料?
  • 9. 高考报名时间和地点是怎样规定的?
  • 10. 报名工作流程是怎样的?
  • 11. 考生报名时如何选择考试类型和报考类别?
  • 12. 报考艺术类考生如何选择艺术类别? 46
  • 13. 考生可以选择哪些外语语种?
  • 14. 享受优惠加分政策的考生如何申请加分?
  • 15. 符合优先录取条件的考生如何申请?
  • 16. 报考普通体育类专业有哪些具体要求?
  • 17“ 统考”和“对口”两类考试能否兼报?
  • 18. 报名和参加各种专业(专门)考试及文化统一考试时是否 需要携带本人居民身份证?
  • 19. 身份证、户口簿、学考及学籍的姓名、身份证号、民族、 出生年月等内容不一致的如何办理报名手续?
  • 20. 在外省就读回我省参加高考的考生,普通高中学生 学业水平考试和综合素质评价信息如何办理?
  • 21. 考生号的作用是什么
  • 22. 准考证的作用是什么?
  • 23. 考生个人信息表的使用范围有哪些?
  • 24. 考生为什么要签订 《河北省普通高校招生考试考生诚信承诺书》?
  • 25. 如何确定考试科目和交纳考试费?
  • 26. 报考选择性考试科目有什么要求?
  • 1. 怎样进行思想政治品德考核?
  • 2. 哪些情况属于思想政治品德考核不合格?
  • 3. 报考军队院校的考生怎样进行政治考核?
  • 4. 报考公安院校的考生怎样进行政治考察?
  • 5. 报考司法类高校提前录取专业的考生怎样进行政治考察?
  • 6. 报考军队飞行学员的考生怎样进行政治考核?
  • 7. 报考北京电子科技学院的考生怎样进行政审?
  • 8. 综合素质评价包括哪些内容?
  • 9. 综合素质评价如何使用?
  • 1. 考生为什么要体检?
  • 2. 考生体检应注意什么?
  • 3. 高考体检有哪些项目?
  • 4. 高考体检是怎样进行的?
  • 5. 对体检结果有异议的考生怎样申请复查?
  • 6. 《普通高等学校招生体检工作指导意见》的具体内容是什么?
  • 7. 报考军队院校军检是如何进行的?
  • 8. 军队院校招收学员体格检查标准具体内容是什么?
  • 9. 报考公安院校面试体检和体能测评是如何安排的?
  • 10. 公安院校招生对面试体检和体能测评有哪些要求?
  • 11. 司法类高校提前录取专业的面试、体能测试是如何安排的?
  • 12. 司法类高校提前录取专业对身体条件有哪些要求?
  • 13. 报考军队飞行学员体检是如何安排的?
  • 14. 报考民航飞行学员面试体检是如何安排的?
  • 15. 普通高校为什么对新生进行身体复查?
  • 16. 新旧视力对照表
  • 17. 血压原用计量单位与法定计量单位换算表
  • 1. 普通高校招生文化考试时间如何安排?
  • 2. 普通高校招生文化课考试科目是如何设置的?
  • 3. 思想政治、地理、化学、生物4门再选科目考试成绩 是如何进行等级赋分的?
  • 4. 中等职业学校对口升学考试科目及考试时间如何安排?
  • 5. 考点、考场是如何安排的?
  • 6. 普通高考外语听力考试如何进行?
  • 7. 残疾考生如何申请高考合理便利?
  • 8. 艺术类专业统考是如何安排的?
  • 9. 考生如何查询艺术类专业统考成绩?
  • 10. 戏曲类专业省际联考是如何安排的?
  • 11. 院校艺术类专业校考是如何安排的?
  • 12. 我省对艺术类专业校考的相关规定有哪些?
  • 13. 普通体育类专业考试有哪些要求?
  • 14. 普通体育专业考试是怎样安排测试顺序的?
  • 15. 河北体育学院少数民族传统体育项目测试是怎样安排的?
  • 16. 对口专业考试是如何安排的?
  • 17. 考生如何参加外语口试?
  • 18. 《考场规则》的主要内容有哪些?
  • 19. 考生如何配合考试工作人员进行安全检查?
  • 20. 考生考前需做好哪些心理准备?
  • 21. 考生考前应注意哪些主要事项?
  • 22. 考生遇有特殊情况应如何处理?
  • 23. 答题过程中考生需要注意哪些事项?
  • 24. 针对网上评卷,考生答题时要注意哪些问题?
  • 25. 网上评卷有哪些流程?
  • 26. 网上评卷有何优点?
  • 27. 考生如何查询高考成绩?
  • 1. 考生填报志愿的作用是什么?
  • 2. 填报志愿的时间是如何安排的?
  • 3. 考生填报志愿方式和流程是怎样的?
  • 4. 考生在网上填报志愿时应注意哪些事项?
  • 5. 高校专业(类)对考生选考科目有哪些要求?
  • 6. 如何查询招生院校对选考科目的要求?
  • 7.2024 年各录取批次及志愿是如何设置的
  • 8. 征集计划是怎样产生的?
  • 9. 考生填报志愿的主要依据和参考资料有哪些?
  • 10. 填报平行志愿有哪些建议?
  • 11. 对平行志愿的认识误区有哪些?
  • 12. 什么是志愿填报辅助系统?
  • 13. 志愿填报辅助系统操作流程及注意事项有哪些?
  • 14. 什么是河北省高考志愿填报智能参考系统??
  • 15. 志愿填报系统、志愿填报辅助系统和志愿填报智能参考系统 有什么区别?
  • 16. 填报志愿时考生如何对待优惠加分政策?
  • 17. 艺术类专业考生填报志愿应注意哪些问题?
  • 18. 怎样填报艺术类平行志愿?
  • 19. 体育类考生填报志愿有哪些注意事项?
  • 20. 艺术类、体育类考生是否可以填报普通类专业志愿?
  • 21. 填报军队院校志愿应注意哪些事项?
  • 22. 填 报 招 飞 计 划 应 注 意 哪 些 事 项 ?
  • 23. 填报公安院校志愿应注意哪些事项?
  • 24. 如何填报定向就业招生志愿?
  • 25. 高水平运动队招生如何填报志愿?
  • 26. 国家专项计划招生如何填报志愿?
  • 27. 高校专项计划招生如何填报志愿?
  • 28. 地方专项计划招生如何填报志愿?
  • 29. 如何填报公费师范生(含优师专项)志愿?
  • 30. 如何填报预科班志愿?
  • 31. 志愿填报截止后还可以补报志愿吗?
  • 32. 未填报集中志愿的考生,是否可以填报该批(段)的 征集志愿?
  • 33. 如何对待“冷门”和“热门”专业?
  • 34. 填报志愿时如何兼顾个性因素?
  • 35. 河北招生考试信息服务网网上咨询活动何时开始?
  • 1. 什么是“学校负责、招办监督”的录取原则?
  • 2. 各批各类录取控制分数线是如何划定的?
  • 3. 投档原则是什么?
  • 4. 高校招生录取规则是什么?
  • 5. 各批次(段)在录取时是否相互影响?
  • 6. 网上录取基本流程是什么?
  • 7. 平行志愿投档是如何进行的?
  • 8. 征集志愿是如何投档的?
  • 9. 什么是投档线?
  • 10. 国家确定的优惠加分项目有哪些?分值是多少?
  • 11. 河北省确定的优惠加分项目有哪些?分值是多少?
  • 12. 如何查询优惠加分信息?
  • 13. 投档录取时如何使用优惠加分?
  • 14. 优先录取的规定有哪些?
  • 15. 军队院校如何录取?
  • 16. 公安院校如何录取?
  • 17. 司法类院校提前录取专业如何录取?
  • 18. 艺术类专业如何录取?
  • 19. 普通体育类专业如何录取?
  • 20. 高水平运动队招生如何录取?
  • 21. 定向就业招生如何录取?
  • 22. 国家专项计划招生如何录取?
  • 23. 高校专项计划招生如何录取?
  • 24. 地方专项计划招生如何录取?
  • 25. 国家专项计划、高校专项计划、地方专项计划录取考生 放弃入学资格或退学有什么后果?
  • 26. 少数民族本科预科班如何录取?
  • 27. 边防军人子女预科班如何录取?
  • 28. 残疾考生报考普通高校录取有什么规定?
  • 29. 单独招收残障考生的院校有哪些?
  • 30. 控制分数线上考生未被录取的主要原因有哪些?
  • 31. 如何准确查询录取结果?
  • 32. 如何识别和防范招生诈骗行为?
  • 33. 录取常见问题有哪些?
  • 34. 新生入学报到前应做哪些准备?
  • 35. 新生入学需要携带哪些档案?
  • 36.考生如何申请国家助学贷款?
  • 37. 高校如何进行新生入学复查?
  • 38. 新 生 入 学 时 如 何 购 买 合 格 的 学 生 公 寓 床 上 用 品 ?
  • 1. 如何认定与处理考生的违规行为?
  • 2. 考试工作人员违纪舞弊应受何种处理?
  • 3. 对教育考试机构作出的违规处理决定不服的如何申诉?
  • 4. 发现考生违规行为如何举报?
  • 5. 违规招生的学校应受何种处理?
  • 6. 在艺术、体育、对口升学和高职单招等各类单招中违规的 考生如何处理?
  • 7. 《中华人民共和国刑法》涉及考试作弊入刑的 主要内容是什么?
  • 8. 涉及高考作弊入刑的 主要内容是什么?
  • 9. 《中国共产党纪律处分条例》涉及考试招生违规处理的 主要内容是什么?
  • 10. 《中华人民共和国教育法》涉及考试招生违规处理的 主要内容是什么?


河北省2024年普通高校

招生工作新视点

 1.深化艺术类专业考试招生改革。按照教育部统一部署,2024年起,普通高校招生艺术类专业考试分为美术与设计类、音乐类(包括音乐表演、音乐教育两类,其中音乐表演类考试包括声乐、器乐两个方向)、舞蹈类、表(导)演类(包括戏剧影视表演、服装表演、戏剧影视导演三个方向)、播音与主持类、书法类、戏曲类等7个科类。其中,美术与设计类、音乐类、舞蹈类、表(导)演类、播音与主持类、书法类等6个科类实行省级统考;戏曲类实行省际联考。

2024年起,进一步提高文化课成绩要求,艺术史论、戏剧影视文学等部分艺术类专业,直接依据考生高考文化课成绩择优录取;美术与设计类、书法类、舞蹈类、表(导)演类、音乐类等5类省级统考的高考文化课成绩占比调整为50%,播音与主持类仍为70%;校考专业高考文化课成绩要求,调整为执行普通类专业批次录取控制分数线(有关破格录取的,按高校公布的考试招生办法执行)。严格控制校考范围,仅少数高校经教育部批准后,可在省级统考基础上组织校考。

2.改革高校高水平艺术团招生办法。2024年起,高校高水平艺术团不再从高校招生环节选拔,由相关高校从在校生中遴选培养。

3.深化高校高水平运动队考试招生改革。2024年起,进一步完善和规范高校高水平运动队考试招生工作,通过优化招生项目范围,严格报考条件和资格审核,改进考试评价方式,提高文化成绩要求,完善招生录取机制,选拔培养德智体美劳全面发展且具有较高体育竞技水平的学生,为奥运会、世界大学生运动会等重大体育比赛和国家竞技体育后备人才培养体系提供人才支撑。

4.高校选考科目优化调整。教育部印发的《普通高校本科招生专业选考科目要求指引(通用版)》,从2024年高考开始实施。这次高校选考科目优化调整,主要是强化了相关专业对高中学习物理、化学科目的基础性要求。学生可根据自身兴趣爱好及特长,统筹考虑国家和社会需要,结合拟报考高校及专业(类)的选考科目要求,合理确定选考科目。5.优化高职单招考试模式。2024年起,报考高职单招的考生在高考报名所在县(市、区)参加考试;普通高中毕业生文化素质不再使用高中学业水平合格性考试成绩折算替代,所有考生均须参加考试;高职单招报考缴费、打印准考证、查询成绩、填报志愿、查询录取结果等均须登录河北省教育考试院官方网站进行;志愿填报数量由5所院校增加至10所院校。

重要日程备忘

image.png


image.png


 



1. 普通高校招生有哪些形式? 

普通高校招生主要有统一考试招生和单独考试招生两种形式。

统一考试招生是指普通高校招生统一文化考试(含普通高中学业水平选择性考试)招生。

单独考试招生是指经教育部或省级教育行政部门批准的普通高校单独组织的考试招生。目前,主要有运动训练、武术与民族传统体育专业考试招生(简称体育单招),少年班,职教师资班,残障单招,消防单招等单独考试招生和普通高等职业教育单独考试招生(简称高职单招)。




  2. 普通高校招生的主要环节有哪些?

   普通高校招生包括报名、思想政治品德考核、体检、确定选考科目、考试、填报志愿、录取、入学复查等环节。

报名,报名是基础工作,符合报名条件的考生完成报名后,才能取得考试资格或录取备案资格。考生报名时填写的信息,用于建立考生电子档案、编排考场和录取备案等。

思想政治品德考核,主要是考核考生本人的现实表现。由考生所在学校或单位对考生的政治态度、思想品德作出全面鉴定,是高校录取的重要依据之一。

体检,即身体健康状况检查,由县级以上招生考试机构和卫生健康行政部门组织实施,在指定的二级甲等以上医院或相应的医疗单位进行。根据身体检查结果,考生可以选择报考适合自己身体状况的专业(类)。招生高校录取时,根据本校招生章程中公布的体检要求,选择符合条件的考生。

  确定选考科目,选考科目为思想政治、历史、地理、物理、 化学、生物6门,考生须从历史、物理2门首选科目中选择1门,再从思想政治、地理、化学、生物4门再选科目中选择2门参加考试。普通高中在校学生应在相应科目学业水平合格性考试合格的基础上报考选择性考试科目。 

  考试。检验考生对知识掌握和运用能力的评价方式,考试成绩将作为高校招生录取的重要依据。 

  填报志愿。考生通过填报志愿表达自己的升学愿望,志愿信息是高校录取新生的重要依据。 

  录取。高校依据本校录取规则和考生志愿,择优选拔德智体美劳全面发展的新生。

  入学复查。新生入学报到时,各高校将依照《普通高等学校学生管理规定》对新生进行身份复核、体检复查。同时,对部分艺术、体育等特殊类型招生录取的新生,高校将开展入学专业复测和复核。具体要求请向录取高校咨询。


3. 普通高考文化总成绩是如何组成的?

考生普通高考文化总成绩由统一高考的语文、数学、外语3门成绩和考生选择的3门选考科目成绩组成,满分为750分。统一高考科目的语文、数学、外语每门满分为150分,均按原始成绩计入考生高考文化总成绩。选考科目每门满分为100分,其中,首选科目历史、物理使用原始成绩计入考生高考文化总成绩,再选科目思想政治、地理、化学、生物按等级赋分后的成绩计入生高考文化总成绩。


 

4. 招生计划是如何编制的?什么是历史科目组合、物理科目组合?

我省普通高校招生,普通类、体育类按历史科目组合、物理科目组合分别编制招生计划;艺术类不区分历史科目组合、物理科目组合,统一编制招生计划。

历史科目组合和物理科目组合以首选科目为界定标准,首选科目选择历史的为历史科目组合,首选科目选择物理的为物理科目组合。


   5. 什么是強基计划? 

为深入贯彻习近平新时代中国特色社会主义思想,落实全国教育大会精神,服务国家重大战略需求,加强基础学科拔尖创新人才选拔培养,教育部在深入调研、总结高校自主招生和上海等

地高考综合改革试点经验的基础上,制定出台了《关于在部分高校开展基础学科招生改革试点工作的意见》(也称强基计划),逐步建立基础学科拔尖创新人才选拔培养的有效机制。

立足服务国家战略需求。强基计划主要选拔有志于服务国家重大战略需求且综合素质优秀或基础学科拔尖的学生,突出基础 学科的支撑引领作用,由有关高校结合自身办学特色,聚焦高端 芯片与软件、智能科技、新材料、先进制造和国家安全等关键领 域以及国家人才紧缺的人文社会科学领域,重点在数学、物理、化学、生物及历史、哲学、古文字学等相关专业招生。

遴选部分高校开展试点。教育部组织专家综合考虑高校的办学定位、人才培养质量、科研项目及平台建设情况、招生和人才培养方案等因素,按照“一校一策”的原则,研究确定强基计划招生高校、专业和规模。

改革招生选拔模式。在保证公平公正的前提下,探索建立多维度考核评价考生的招生模式。考生参加统一高考和高校考核后,高校根据考生高考成绩、高校综合考核结果及综合素质评价情况等按比例合成考生综合成绩(其中高考成绩所占比例不得低85%),按考生综合成绩由高到低进行录取。对于极少数在相关学科领域具有突出才能和表现的考生,有关高校可制定破格入围高校考核的条件和办法,并提前向社会公布,考生须参加统一高考。

创新人才培养模式。高校对通过强基计划录取的学生单独制订培养方案,采取导师制、小班化等培养模式。建立激励机制,

   增强学生的荣誉感和使命感。畅通成长发展通道,对学业优秀的学生,高校可在免试推荐研究生、直博、公派留学、奖学金等方面予以优先安排,加强本硕博衔接培养工作。强基计划录取的学生入校后原则上不得转到相关学科之外的专业就读。

健全公平公正保障机制。严格高校考核,笔试、面试均须安排在国家教育考试标准化考点进行,面试采取专家、考生双随”抽签的方式,全程录音录像。完善信息公开公示,加强违规违纪查处。将强基计划招生及人才培养工作纳入巡视和督导的工 作范围,建立动态准入退出机制。

拟报考强基计划的考生,须按有关试点高校招生简章要求,一般于4月份登录教育部“阳光高考”平台报考,无须在我省志愿填报系统填报强基计划志愿,录取结果由高校确定。我省根据高校确定的拟录取考生名单,在本科提前批录取开始前办理录取备案手续。强基计划考生可正常填报我省其他类志愿,如被强基计划录取,将不能参加其他各类型和批次的录取。


6.强基计划试点高校如下:


2023年强基计划试点高校共有39所:


image.png

2024年强基计划试点高校名单,可登录教育部“阳光高考”平台网站查询或直接向相关招生院校咨询。


7. 强基计划如何招生?

有关高校应根据本校的办学定位、学科特色等,制定强基计划招生简章,内容包括领导机构、招生专业及计划、报考条件及方式、入围高校考核的办法、考核程序及办法、学生综合素质评价使用办法、综合成绩折算办法及录取规则、监督机制、咨询及申诉渠道等。招生简章报经教育部核准备案后,提前向社会公布。

符合生源所在地当年高考报名条件以及强基计划招生学校报考条件的考生,由本人提出申请,按高校招生简章要求进行网上报名。

报名考生均须参加全国统一高考。对于以高考成绩围高校考核的,有关高校在各省(区、市)本科一批录取最低控制分数线(合并录取批次省份应单独划定相应分数线,下同)上,按照 在生源所在省份强基计划招生名额的一定倍数,以考生高考成绩 从高到低确定参加高校考核名单。对于符合高校破格入围条件的考生,考生高考成绩应达到高校招生简章确定的要求,且原则上 不得低于各省(区、市)本科一批录取最低控制分数线。有关高校应按照教育部规定确定入围高校考核的考生名单并公示入围标准,对入围考生组织高校考核(含笔试、面试)和体育测试,其中体育测试结果作为录取的重要参考。高校考核是国家教育考试的组成部分,由招生高校负责组织实施。有关高校要严格执行教育部关于特殊类型考试招生工作相关规定,合理确定高校考核的 内容和形式。积极探索通过笔试、面试、实践操作等方式,考查学生分析问题、解决问题的能力和创新思维,增强选才的科学性。充分运用学生综合素质档案,全面、深入地考察学生的能力和素养。高校考核的笔试、面试应安排在国家教育考试标准化考点进行,面试采取专家、考生“双随机”抽签的方式,全程录音录像。

有关高校将考生高考成绩、高校综合考核结果及综合素质评价情况等按比例合成考生综合成绩(其中高考成绩所占比例不得低于85%),并根据考生填报志愿,按综合成绩由高到低确定录取名单,提交生源所在省级招办办理录取手续。破格录取的考生,按照高校招生简章公布办法进行录取。被录取考生不再参加后续高考志愿录取。有关高校须及时确定录取考生名单并公示录取标准。

具体招生程序及要求以各有关高校公布的招生简章为准。 



 


 



8.  强基计划的报考流程是什么? 


报考流程包括向高校申请网上报名、参加统一高考、高校确 定考核名单并组织考核、高校录取等环节。具体报考流程见下图:

 


image.png

考生可登录教育部“阳光高考”平台网站查询相关报考政 策,也可直接登录招生院校网站查询招生简章,或向相关招生院校咨询

9.  什么是强基计划招生 “十严禁”? 

高校不得发布未经教育部备案的强基计划招生简章或进行虚假招生宣传;不得以任何形式组织与强基计划招生挂钩的冬令营、夏令营及考核工作,或委托个人或中介组织开展报名、考核等有关工作;高校招生工作人员、专家评委不得参与社会机构组织的各类培训、辅导活动;不得以“新生高额奖学金”“入校后重新选择专业”等方式进行恶性生源竞争或向考生违规承诺录取;未经批准不得突破强基计划的招生计划录取;不得在发放新生录取通知书或新生学报到环节更改考生录取专业;通过强基计划录取的学生入校后原则上不得转到相关学科之外的专业就读;省级高校招生委员会和省级教育行政部门不得擅自扩大强基计划招生高校范围或出台与国家招生政策相抵触的招生办法;省级招生考试机构不得为不符合要求的考生或违反规定程序办理录取手续;有关中学等不得出具与事实不符的考生推荐材料、证明材料等或在考生综合素质档案中虚构事实或故意隐瞒事实。


10. 综合评价如何招生? 

高校综合评价招生,是以综合素质为主,综合考量考生高考 成绩、高校考核结论、高中学业水平测试成绩、综合素质评价以及高校自身培养特色要求等五个维度的内容,对高考成绩达到相关标准的考生进行综合评价,择优录取。在我省进行综合评价招生的高校包括南方科技大学、昆山杜克大学、北京外国语大学、深圳北理莫斯科大学、上海纽约大学等5所。


11. 普通高校是怎样分类的?

目前,我国具有普通高等学历教育资格的高校2700多所,站在考生填报志愿的角度对高校进行简单分类如下:

按办学层次,可分为本科院校和专科院校等;按办学类型(办学性质),可分为普通或成人高校、公办或民办院校或独立学院、高等专科学校或高等职业技术学校等;按隶属关系,可分为教育部直属高校、中央其他部门所属高校、省(区、市)所属高 校以及行业所属高校等;此外,按照高校主体专业设置,还可分 为综合院校、工科院校、农业院校、林业院校、医药院校、师范 院校、语言院校、财经院校、政法院校、体育院校、艺术院校、民族院校、军事院校等。




12. 公办院校、民办院校与独立学院有哪些区别?

公办院校是指由政府举办的高校;民办院校是指企业事业组织、社会团体及其他社会组织和公民个人利用非国家财政性教育经费,面向社会举办的高校;独立学院则是指实施本科以上学历教育的普通高校与国家机构以外的社会组织或者个人合作,利用非国家财政性经费举办的实施本科学历教育的高校,属民办性质,其中一些独立学院与校本部共享教学资源。个别高校的举办主体(办学性质)可能会发生变化,考生在填报志愿前,要登录高校官方网站认真查看高校招生章程或向高校咨询。

根据《中华人民共和国民办教育促进法》的规定,民办院校的受教育者在升学、就业、社会优待以及参加先进评选等方面享有与同级同类公办院校的受教育者同等权利。


13. 什么是本硕、本硕博和本博连读?

本硕连读专业是指学生完成本科学业要求,可直接攻读硕士研究生;本硕博连读专业,即本科、硕士研究生、博士研究生连续攻读,中间没有间隔;本博连读专业,即学生获得学士学位 后,直接攻读博士学位,中间不再授予硕士学位,一旦不能完成 博士毕业答辩,则只能获得学士学位。


14. 什么是本科学术互认课程 (ISEC)?

本科学术互认课程 (International   Scholarly   Exchange   Curri- culum,以下简称ISEC) 是由国家留学基金管理委员会(直属于 教育部的非营利性事业法人机构,China Scholarship Council, CSC) 依托服务国家国际教育交流合作的丰富资源,着眼于全 方位服务国内院校的教育国际化工作,联合国内外院校和国际教育专家研发的集合通用型本科学术互认课程方案、国际标准师资 培育平台以及国际化教育资源交流路径为一体的多元化、融通型 国际化人才培养服务体系,是我国高等教育领域引进国外优质教 育资源和促进国内外院校教育交流的有益尝试。

本科学术互认课程采用中外方高校双向学分互认的创新型人 才培养模式,学生就读的国内阶段部分课程可被境外合作院校认可,学生在国外学习阶段的部分课程可被 ISEC项目中方高校审 核认定。本科阶段专业总计学制一般为四年,除国内正常的四年 学制形式外,也可以采用国内、境外两个学习阶段的模式。其中,国内阶段不能少于两年。学生如在境外完成本科最后阶段业,满足境外院校对授予学位所规定的学习时间和学业表现要求的,可申请境外大学学士学位。不出国留学的学生,或在境外 交流一段时间,回到国内院校完成学业,其课程仍将采用本科学 术互认课程的国际化课程。学生顺利完成学业,经毕业资格审查 合格后颁发国内院校的学历和学位证书。本科学术互认课程办学信息详见相关高校官方网站或向高校咨询。




15. 哪些高校(专业)对考生有特殊要求? 

一些高校(专业)在录取时,对考生思想政治品德、身体健康状况、专业特长等方面有特殊要求。主要包括:外语专业、体 育专业、艺术专业、军队院校(含武警)、公安院校、司法院校等。这里简单介绍报考上述院校(专业)的基本要求,在本书以后的相关章节中还有更详细的介绍。

报考外语专业和有外语口试要求专业的,如高校招生有外语口试要求,须按要求参加外语口试。

报考体育专业的,须参加体育专业考试。包括体育教育和社会体育指导与管理专业(以下简称普通体育专业)、体育单招,少数民族传统体育项目。

报考高水平运动队的,须是国家一级(含)及以上运动员且参加全国专业统考。

   考生如报考使用艺术专业考试成绩的相关艺术类专业,应按要求参加相应的艺术类专业考试。可授予艺术学学位的非艺术类专业,如高校对考生有艺术专业基础要求,考生应按照高校要求 参加对应科类的省级统考。

报考用蒙语授课专业的,应具有一定的蒙语基础,须加试蒙 语文。

报考军队院校的,须是普通高中毕业(应、往届毕业生均),普通高中学业水平考试合格性考试体育与健康科目成绩为“合格”,且须参加政治考核、面试和军检。

报考飞行学员的,须本着自愿的原则,符合飞行员体检要求和心理选拔测评要求,并进行相应的政治考核。

报考公安、司法院校(专业)的,应按要求参加公安或司法院校组织的(政治)考察、体检(或体能测试)。报考公安院校(专业)须为河北省户籍的普通高中毕业生。

报考少年班、英才班等基础学科拔尖创新人才选拔培养的,应是德智体美劳全面发展、智力超常、学习成绩优异,须按高校招生简章要求申请并参加相关测试

报考需要提前进行面试、体检的空中乘务等专业的,应按高校要求提前进行面试和体检。


16. 普通体育招生、高水平运动队招生、体育单招有什么区别?

普通体育招生是指普通高等院校体育教育、社会体育指导与管理等体育类专业的招生。高水平运动队招生是指学校根据教育部文件精神和高水平运动队招生规定,依据本校当年高水平运动队项目建设的需要,招收一定数量的高水平运动员。体育单招(部分体育专业单独招生的简称)是指经教育部、国家体育总局批准的部分院校对运动训练、武术与民族传统体育专业实行单独招生。三者在报考条件、招生院校、考试方式、录取政策、录取 专业等方面均有所不同。

报考普通体育专业的,须参加全省统一组织的体育专业考试 (即100米跑、立定三级跳远、原地推铅球、800米跑)。文化课考试参加全国统一高考。其中报考河北体育学院社会体育指导与管理专业(少数民族传统体育项目)的,须是少数民族考生,且须参加河北体育学院单独组织的专业测试,但不要求参加全省统 一组织的体育专业考试。

报考高水平运动队的,应符合生源省份高考报名条件,且获 得国家一级运动员(含)以上技术等级称号〔2027年起,获得国家一级运动员(含)以上技术等级称号且近三年在体育总局、教育部规定的全国性比赛中获得前八名者方可以报考高校运动队〕。高水平运动队招生采取“文化考试+专业测试”相结合的考试评价方式。2024年起,文化考试成绩全部使用全国统一高考文化课 考试成绩,专业测试全部纳入全国统考,由国家体育总局牵头组织实施。

报考体育单招的,应具备运动训练、武术与民族传统体育专业招生所列项目二级运动员(含)以上技术等级称号。考生须参加体育专项考试(全国统考和分区统考方式)和省级招生考试机构组织的文化考试。体育单招录取后就读运动训练、武术与民族 传统体育专业。


17. 体育单招如何招生?

体育单招实行文化考试和体育专项考试相结合的办法。文化考试由生源地省级招生考试机构组织实施,体育专项考试分项目 采用全国统考和分区统考方式,由国家体育总局委托高校组织 实施。

报考的考生应当符合以下条件:符合高考报名条件;具备下述所列项目的二级运动员(含)以上运动技术等级称号。

报考体育单招的考生必须参加生源所在地招生考试机构组织的普通高考报名。考生还须依据各高校招生简章要求,在规定时间内登录“中国运动文化教育网”(网址: http://www.ydyedu-cation.com) 或“体教联盟”APP 体育单招管理系统进行注册 (验证考生报考资格)并报名。具体时间:

冬季项目:

注册时间为2023年12月4日12:00至1231日12:00  报名时间为2023年12月22日12:00至12月31日12:00 夏季项目(含武术与民族传统体育专业项目,下同):

注册时间为2024年2月1日12:00至3月10日12:00 报名时间为2024年3月1 12:00至3月10日12:00

2024年报名考生的等级证书审批日期:冬季项目为2014年11日至2023年12月31日;夏季项目为2014年1月12024年3月10日。

image.png

体育专项考试安排:冬季项目考试时间为2024年1月4日至 5月12日期间;夏季项目考试时间为2024年3月20日至5月12日期间。

考生若已报名运动训练、武术与民族传统体育专业志愿并被

录取,不再参加普通高考及高校高水平运动队的录取。

达到免试条件的运动员(含退役运动员)在履行相关手续后,可进入相关高校学习深造。具体招生事宜可向招生院校咨询。

举办运动训练专业的招生院校有:

 

image.png

image.pngimage.png


举办武术与民族传统体育专业的招生院校有:

 image.png

image.png

 

运动训练专业所设项目为:

冬季项目:速度滑冰、短道速滑、花样滑冰、冰球、冰壶、 越野滑雪、高山滑雪、跳台滑雪、自由式滑雪(空中技巧、雪上技巧)、单板滑雪(平行大回转、U 型场地)、冬季两项。

夏季项目:射击、射箭、场地自行车、公路自行车、山地自 行车、BMX 小轮车、击剑、现代五项、铁人三项、马术、帆船、赛艇、皮划艇静水、皮划艇激流回旋、冲浪、蹼泳、滑水、摩托  艇、举重、摔跤(自由式、古典式)、柔道、拳击、跆拳道、空手道、田径、游泳、公开水域游泳、跳水、水球、花样游泳、体  操、艺术体操、蹦床、技巧、手球、曲棍球、棒球、垒球、足球(十一人制)、篮球、排球、沙滩排球、乒乓球、羽毛球、网球、 橄榄球、高尔夫球、围棋、象棋、国际象棋、登山、攀岩。

其中,篮球项目考生使用篮球或三人篮球运动员技术等级证书均可报名,按照篮球项目考试方法与评分标准参加篮球专项考试;橄榄球项目考生使用七人制或十五人制运动员技术等级证书 均可报名,按照七人制橄榄球考试方法与评分标准参加橄榄球专项考试。

武术与民族传统体育专业所设项目为:武术套路、武术散 打、中国式摔跤。


18. 高水平运动队如何招生?

有关高校在奥运会、世界大学生运动会项目范围内,按照教 育部评估确定的项目,结合学校实际,根据本校运动队建设规 划,确定运动队招生项目和招生计划。对于不具备相关师资、设备、场地等组队条件、退队率超过20%的高校,和非奥运会或世界大学生运动会项目、未设运动员技术等级标准、生源严重不足且连续两年录取数为零的相关项目,不再安排运动队招生。本校 运动训练、武术与民族传统体育专业已涉及的运动项目,原则上 不安排运动队招生。2024年招收高水平运动队的院校和项目,请考生登录“阳光高考”平台(网址:https://gaokao.chsi.com.cn) 查阅相关院校招生简章。

(1)报名

考生须参加统一高考报名,获得国家一级运动员()以上 技术等级称号者可报考高水平运动队。考生须在高校简章规定时 间内登录“中国运动文化教育网”或“体教联盟”APP的高水平 运动队报名系统进行报名。考生所持本人运动员技术等级证书中的运动项目应与报考高校的运动项目一致(原则上运动小项也应 对应一致,田径项目须严格对应)。

(2)体育专业测试

考生须参加报考项目的体育专业测试。体育专业测试实施全 国统考,与运动训练、武术与民族传统体育专业招生的体育专项 考试统一组织,统一采用体育总局相关体育专项考试方法与评分 标准进行评分。高校不得再组织相关项目的校考,不得以文化测 试、体能测试、资格审查等名义变相组织校考和提前筛选。具体 考试安排及其他相关信息以“中国运动文化教育网”“体教联”APP 通知为准。

文化课考试:报名考生均须参加全国统一高考,文化课考试

成绩全部使用全国统一高考文化课考试成绩。

(3)录取

高考前,高校根据全国体育专业测试成绩分布情况提出本校 相关项目专业合格成绩要求,结合报名资格审核结果、体育单招 录取结果和招生计划,确定并公示本校专业考试合格名单,明确 相应考生的高考文化课成绩录取要求。2024年起,相关文件规定的部分“双一流”建设高校对考生的高考成绩要求须达到生源份普通类本科批次录取控制分数线;其他高校对考生的高考成绩

要求须达到生源省份普通类本科批次录取控制分数线的80%。

高考成绩公布后、志愿填报前,高校在高考文化课成绩达到录取要求的合格考生范围内,依据体育专业测试成绩,择优确定入围考生名单并按规定公示。围考生人数不得超过学校运动队 招生计划数。入围考生按要求填报运动队志愿。

高校按照公布的录取规则,根据入围考生填报的志愿和体育专业测试成绩择优录取。在录取考生中,高考文化课成绩不低于高校相关专业在我省本科批录取分数线下20分的考生,可录取至对应的普通专业;其余考生限定录取至体育学类专业。高校应在 普通本科批次录取结束前确定考生录取专业


19. 艺术类专业考试形式有哪些?

艺术类专业分为两类:一类是不组织专业考试的专业,包括 艺术史论、艺术管理、非物质文化遗产保护、戏剧学、电影学、戏剧影视文学、广播电视编导、影视技术等一类是可组织专业 考试的专业,包括音乐类、舞蹈类、表(导)演类、播音与主持 类、美术与设计类、书法类、戏曲类等,有关高校也可根据人才培养实际不对考生专业考试成绩提出要求。

艺术类专业考试分为省级统考(含省际联考)、高校校考。省级统考由我省统一组织(简称统考),音乐类、舞蹈类、表(导)演类、播音与主持类、美术与设计类、书法类等实行省级 统考,戏曲类实行省际联考;高校校考由招生院校组织实施。


20.  什么是艺术类专业考试“考评分离”?

“考评分离”指的是在条件统一的考试环境中对考生考试音 视频先进行采集,以音视频作为考生答卷,考试结束后再组织评委专家通过集中观看考生音视频进行评分的考试组织模式。我省音乐类(声乐、器乐、视唱科目)、舞蹈类、戏剧影视表演类,戏剧影视导演类(文学作品朗诵、命题即兴表演科目)、播音与主持类专业统考采取“考评分离”形式。

“考评分离”实现了考场基础条件统一、考场设备配备统一、核心录制设备型号统一、考试系统软件统一、音视频采集参数设 置统一、录制设备位置统一、考生应试区域统一、录制背景统 一。即考生录制环境完全一致,确保考试公平公正。



21. 什么是“定向就业招生”?

为了帮助边远地区、少数民族地区和工作环境比较艰苦的行业培养人才,委托培养单位与普通高校签订协议,高校拿出部分计划进行“定向就业招生”。考生填报了定向志愿即视为愿意到定向单位就业。


22.  什么是国家专项计划?

国家专项计划是指中央部门所属高校和省(区、市)所属重点高校面向原贫困地区定向招生专项计划。按照教育部规定,专项计划实施区域的贫困县脱贫后2024年仍可继续享受专项计划政策。


23.  什么是高校专项计划? 

高校专项计划是指教育部直属高校和其他试点高校单独招收农村学生专项计划。按照教育部规定,专项计划实施区域的贫困县脱贫后2024年仍可继续享受专项计划政策。


24. 什么是地方专项计划?             

地方专项计划是指我省省属骨干高校单独招收农村学生专项计划。我省实施区域为县〔含县级市和自2014年起经国务院批准的县(市)改区〕。


25. 什么是公费师范生?

公费师范生是师范院校师范生公费教育的简称。学生在校学习期间免交学费,免交住宿费,并补助生活费。公费师范生须成绩合格,毕业后到协议确定的地方中小学任教,有编有岗。

特别提醒考生,在享受优惠政策的同时也要注意义务和责任。公费师范生须按要求签订协议,承诺毕业后从事中小学教育,服务期6年。公费师范毕业生未履行协议的,要按规定退还 已享受的公费教育费用并缴纳违约金。具体事宜请考生在报考前 向录取院校或者教育行政部门咨询。


26.  什么是优师专项计划? 

优师专项计划属于公费师范生范畴,是为中西部欠发达地区 定向培养教师的专项计划,面向全省招生,分为国家优师专项(教育部直属师范大学承担)和地方优师专项(河北师范大学和河北民族师范学院承担),面向我省乡村振兴重点帮扶县(原集中连片特困县、国家扶贫开发工作重点县)中小学就业。考生被 优师专项计划录取后须签订协议,详情请咨询有关院校。

优师专项计划和公费师范生的区别:优师专项计划面向我省乡村振兴重点帮扶县中小学就业。公费师范生则由设区市有关部门及相关县(市、区)人民政府按照“择优、就近、急需”的原则落实工作岗位


27.  什么是免费医学定向计划?

免费医学定向计划是高校与基层医疗卫生机构(主要为乡镇 卫生院)达成合作协议,按培养要求培养医务人员的专项计划。学生在校学习期间免除学费、住宿费,并补助生活费。若考生入 学后不能正常毕业,要按规定退还已享受的减免教育费用。具体 2024年有关规定和招生计划。

学生在获取录取通知书之前,须与培养高校和当地县级卫生行政部门签署定向就业协议,承诺毕业后到有关基层医疗卫生机构服务不少于6年。具体事宜请向录取院校咨询。


28.  什么是对口升学考试?

对口升学考试是中等职业学校对口升学全省统一考试的简称,是指省内普通高校对口招收中等职业学校、中等专业学校及技工学校毕业学生的考试招生形式。对口升学考试包括文化考试和专业考试两部分,采取全省统一命题、统一考试、统一评定成绩的方法


29.  对口升学考试与普通高校招生考试能否兼报?

对口升学考试与普通高校招生考试两者不能兼报。第一,试卷不同。普通高校招生使用普通高考试卷,对口升学考试使用河北省对口招生试卷。第二,文化统一考试时间重合。考生不能同时参加普通高校招生考试与对口升学考试。第三,招生对象不同。普通高校招生的对象主要是高级中等教育学校毕业或具有同等学力者,对口升学考试的对象仅限于中等职业学校、中等专业学校及技工学校毕业学生。


30. 什么是高职单招?怎样报考? 

高职单招是普通高等职业教育单独考试招生的简称,是高职 院校分类考试招生的重要组成部分,是技术技能人才选拔的重要 渠道。按照国家有关规定,高职院校分类考试招生与普通高校考 试招生相对分开,更加突出职业教育人才培养规律,实行“文化 素质+职业技能”的评价方式。

(1)报考对象已通过2024年我省普通高校招生报名,且符合所报考院校招生条件的考生。

(2)报考时间

2024年2月26日9时至2月29日17时,考生(含免试考 )登录河北省教育考试院官方网站 (http://www.hebeea.edu.cn)  或河北省高职单招系统 (http://gzdz.hebeea.edu.cn) 进行网上报考,并依据相关要求和个人意愿选择考试类,交纳考试费。

(3)考试类的选择

2024年高职单招面向普通高中毕业生招生专业划分为考试一类到考试十类等10个考试类,其中,考试八类细分为艺术—音乐类、艺术—舞蹈及表演类、艺术一美术设计类、艺术一文化服务类共4个类别;面向中职学校毕业生招生专业划分为建筑类、机械类、农林类等10个专业类。

2024年高职单招招生计划,采取面向普通高中毕业生计划 (简称高中生计划)和面向中职毕业生计划(简称中职生计划) 分开编列。普通高中毕业生(含同等学力)可报考面向高中生计 划中的任一考试类,但不得报考面向中职生计划;中职学校毕业 (含中等师范、职业高中及其他中等专业学校、技工学校毕业 生等)可报考面向中职生计划,也可报考面向高中生计划,二者 选择其一。如报考面向中职生计划,须选择与高级中等教育阶段 所学专业相对应的专业类;如报考面向高中生计划,可任选考试 类报考,须与该类考生一起参加考试、录取。

各考试类(专业类)均设立牵头院校,具体各类牵头院校如下:

 

image.pngimage.png

(4)考试

实行“文化素质+职业技能”的考试方式,总分为750分。其中,文化素质考试满分300分,职业技能考试满分450分。具体考试内容按考生类别分为以下两类:

报考面向普通高中毕业生计划的考生。文化素质考试科目为语文、数学,每科150分。根据《河北省教育厅关于做好2024年 普通高等职业教育单独考试招生工作的通知》规定,所有考生均 须参加考试,普通高中毕业生文化素质不再使用高中学业水平合 格性考试成绩折算替代。

职业技能考试科目为专业基础和职业适应性测试,专业基础考试部分满分100分,职业适应性测试部分满分350分。其中,专业基础考试由牵头院校根据本考试类所含专业特点,从英语、 思想政治、历史、地理、物理、化学、生物等7个科目中选择1科。具体请考生查询牵头院校官方网站公布的考试说明,或直接  向牵头院校咨询。报考面向中职毕业生计划的考生。文化素质考试满分300分,职业技能考试满分450分。文化素质考试科目为语文、数学,每科150分。

职业技能考试包括专业能力测试和技术技能测试两部分, 业能力测试部分满分100分,技术技能测试部分满分350分。其 中,专业能力测试以教育部发布的中职专业教学标准中核心专业 知识为基本依据,重点考察综合专业能力;技术技能测试以教育 部发布的中职专业教学标准中核心技术技能为基本依据,充分体 现岗位技能、通用技术等内容。

①准考证打印

2024年3月28日9时至4月2日17时,考生可登录河北省教育考试院官方网站打印准考证。

②考试时间和地点

2024年高职单招考试时间为2024年4月2日,考生在高考 报名所在的县(市、区)参加考试(采取面试形式的考试八类音  乐类、舞蹈及表演类职业适应性测试由河北艺术职业学院组织,考试时间为2024年3月30日,考点设在河北艺术职业学院),具 体各科目考试时间如下:

 

image.png


③成绩查询

2024年4月中旬,考生可登录河北省教育考试院网站 (http:// www.hebeea.edu.cn) 或河北省高职单招系统 (http://gzdz. hebeea.edu.cn)    查询本人考试成绩。

对成绩有疑问的考生,可在规定时间内,到高考报名所在地 (市、区)招生考试机构提交书面成绩复核申请,并通过河北 省教育考试院官方网站查询成绩复核结果。

④志愿填报

高职单招实行平行志愿填报方式,设集中志愿和一 次征集志 愿。成绩公布后,考生在规定时间内登录河北省教育考试院官方 网站填报志愿,每次填报志愿可在本人所报考的考试类中选报10 所院校,每所院校最多填报6个专业和1个是否服从专业调剂选 项,不得跨类填报。

具体志愿填报时间请关注河北省教育考试院官方网站 (http://www.hebeea.edu.cn)  公告。

⑤录 

高职单招实行计算机远程网上录取。按照平行志愿投档原 则,即“分数优先、遵循志愿、  次投档、不再补档”,将各类 控制线上未录取的有志愿考生,结合高校要求,按高职单招考试 总成绩从高分到低分排序,遵循考生的志愿顺序依次投档,由高

校择优录取。

考生可在规定时间内登录省教育考试院官方网站查询本人录 取结果。

录取结束后,招生院校公布录取考生名单,向考生发放由校 长签发的录取通知书。已被高职单招录取的考生不再参加当年普 通高考、对口升学等考试及录取。

经河北省教育厅批准,2024年(www.hebeea.edu.cn) 公告。

image.pngimage.pngimage.png

院校具体招生专业以省教育厅批准的最终招生计划为准。在我省实施高职单招的省外院校,经省教育厅审核同意后在招生计划中予以公布。


31.  中等职业学校对口升学考试与高职單招考试有什么区别? 、

中等职业学校对口升学考试与高职单招考试属于两类不同的 考试类型,二者在报名方式、招生对象、考试时间、录取时间和 招生层次等方面均不同。第一,报名方式不同。报考对口升学考 试的考生,在高考报名时选择“对口”,并按规定时间向承办院 校进行对口升学专业考试报名交费、进行对口升学文化考试科目 确认交费;报考高职单招的考生须在完成高考报名后,按规定时 间参加高职单招的报考交费。第二,招生对象不同。对口升学考 试招生对象为中职生,即各类中等职业学校(含中等师范、职业 高中及其他中等专业学校)和技工学校毕业的学生;高职单招考 试除了中职生,普通高中生也可以报考。第三,考试时间不同。 对口升学考试专业考试在2月下旬至3月中下旬进行,文化考试 6月7日至8日进行;高职单招考试在4月初进行。第四,录 取时间不同。对口升学考试在7月上旬至8月中旬录取;高职单 招考试一般于4月底完成录取备案。第五,招生层次不同。对口 升学考试既有本科层次高校招生,也有专科层次招生;高职单招 则全部为高职专科层次招生。

提醒考生注意:已被高职单招录取的考生,不再参加当年我省普通高校招生统一考试(含普通高中学业水平选择性考试)和 对口升学考试及录取。



32.  保送生有哪些条件?

具备下列条件之一的我省考生可以申报保送生:

(1)高级中等教育阶段获得全国中学生奥林匹克竞赛决赛一 等奖并被中国科学技术协会遴选为参加国际数学、物理、化学、生物学、信息学奥林匹克竞赛国家队集训的应届高中毕业生。

(2)石家庄外国语中学推荐的部分应届高中毕业生。仅面向 高校外国语言文学类专业保送,2024年起,除北京外国语大学、上海外国语大学、外交学院可继续招收一定数量的外国语中学推 荐保送生安排在英语语种相关专业,其他高校招收的外国语中学推荐保送生均安排在除英语以外的小语种相关专业,鼓励高校培“小语种+”复合型人才。严禁高校以保送生招生形式将外国 语中学推荐保送的学生录取或调整到非外语类专业。推荐限额由  教育部确定。

(3)符合公安部、教育部印发的《关于进一步加强和改进公 安英烈和因公牺牲伤残公安民警子女教育优待工作的通知》(公政治〔2018〕27号)规定的公安烈士、公安英模和一级至四级因 公伤残公安民警的子女,可保送至公安院校学习深造。

(4)符合应急管理部办公厅、教育部办公厅印发的《关于国家综合性消防救援队伍英烈和因公伤残等人员子女报考中国消防 救援学院优待有关事项的通知》(应急厅〔2021〕16号)规定的 烈士、英雄模范和一级至四级因公伤残消防救援人员子女,可保送录取至中国消防救援学院学习。

(5)符合司法部、教育部印发的《关于监狱戒毒人民警察英 烈和因工伤残监狱戒毒人民警察子女保送司法行政系统警察院校  就读的通知》(司办通〔2024〕17号)规定的监狱戒毒人民警察烈士(生前系在职在编监狱戒毒人民警察)或被授予全国司法行 政系统一级、二级英雄模范称号的监狱戒毒人民警察英烈子女, 以及一级至四级因公伤残监狱戒毒人民警察子女,可保送至司法 行政系统警察院校学习深造。

(6)符合国家体育总局办公厅《关于做好2024年高校保送录 取优秀运动员有关事宜的通知》(体科字〔2023〕178号)中有关 高校保送录取运动员条件的考生,如:

①拥护中国共产党领导,拥护社会主义制度;无犯罪记录,无严重兴奋剂违规记录。

②符合2024年高考报名条件,并取得生源所在地高考报名号(考生号)。

③运动成绩优异,满足以下任一条件:

奥运项目破世界纪录或亚洲纪录或全国纪录(不含青年纪 );被授予国际级运动健将称号;足球、篮球、排球项目被授 予运动健将称号;除足球、篮球、排球外的其他奥运项目、围 棋、象棋、国际象棋、武术套路、武术散打项目的运动健将,应参加国家体育总局规定的赛事和小项的最高组别比赛,且取得世界 体育比赛前八名,或亚洲体育比赛前六名,或全国体育比赛前三名。

(7)根据教育部办公厅、国家体育总局办公厅、中国科协办

公厅《关于做好普通高校保送录取有关优秀华侨和港澳台学生 (运动员)的通知》(教学厅〔2020〕2号)规定,对符合内地(祖国大陆)高考或全国联招报名条件,且符合以下两种情况之 一的华侨和港澳台学生(运动员)予以保送录取:

①参加“国际数学奥林匹克竞赛 (IMO)”“国际物理奥林 匹克竞赛 (IPhO)”“国际化学奥林匹克竞赛 (IChO)”“国际 生物学奥林匹克竞赛 (IBO)”“国际信息学奥林匹克竞赛 (IOI)”

5项比赛中任意一项比赛并获奖的学生。

②获得中华人民共和国全国运动会、亚洲夏季运动会、亚洲冬季运动会、夏季奥林匹克运动会、冬季奥林匹克运动会前三名

的运动员。

(8)符合《教育部办公厅关于做好有关高校保送录取世界技能大赛获奖选手工作的通知》(教学厅〔2020〕3号)规定,凡在 世界技能组织主办的“世界技能大赛 (WorldSkillsCompeti-tion)”中获奖的中国国家代表队选手且符合以下条件者,具备保送至高校深造的资格:

①符合有关省(区、市)高考报名条件的中职毕业生,可保送至高校相应的高职或本科专业。

②符合有关省(区、市)专升本报名条件的高职毕业生,可保送至高校相应的本科专业。

中职或高职在校生在其应届毕业当年获得保送资格。

保送录取本科专业的高校限本科层次职业学校和应用型普通本科高校。






33.  什么是高校招生“阳光工程”?

为了进一步规范招生管理,增加招生工作透明度,更好地维 护广大考生的合法权益,确保招生工作的公平、公正,教育部在高校招生工作中实施“阳光工程”,建立和完善以“十公开”为主要内容的信息公开制度,即:

(1)招生政策公开。

(2)高校招生资格公开。

(3)高校招生章程公开。

(4)高校招生计划公开。

(5)考生资格公开。

(6)录取程序公开。

(7)录取结果公开。

(8)咨询及申诉渠道公开。

(9)重大违规事件及处理结果公开。

(10)录取新生复查结果公开。


34.  考生档案有哪些内容和作用? 

考生档案分电子档案和纸介质档案。

考生电子档案是高校录取新生的主要依据,其主要内容包括考生报名信息、体检信息、成绩信息、诚信记录(主要指考试违规以及在招生其他环节违规的简要事实及处理结果)、志愿信息、

普通高中应届考生的综合素质评价和学业水平考试成绩信息及报考军队、公安、司法等院校的政治考核(察)、面试结论信息等。

考生的高考纸介质档案为“河北省2024年普通高校录取新生综合信息登记表”,由考生凭本人高校录取通知书、居民身份证和准考证到当地招办或中学领取。

考生入学时应携带高考纸介质档案和中学学籍档案或人事档案等,交高校学籍管理部门。

未经有关部门同意,任何单位和个人不得对外泄露考生档案信息。


35. 香 港 、澳 门 的 高 校 如 何 招 生 ? 

目前,香港、澳门的21所高校在河北省招生。招生对象是参加当年全国普通高等学校招生统一考试的考生。香港院校有15所,名单如下:

 

image.png


澳门院校有6所,名单如下:

 

image.png


    香港、澳门的高校在河北省招生模式如下:

(1)香港高校招生模式

分为统招和单独招生两种模式。

一是统招模式。香港中文大学和香港城市大学等2所高校按照内地招生体制招生,即该两所高校在河北招生计划中公布招生计划,使用高考成绩,考生志愿填报在本科提前批,由高校择优 录取。香港珠海学院2023年起加入高考统招招生,在我省本科批次录取

二是单独招生模式。香港大学、香港理工大学、香港科技大学、香港浸会大学、岭南大学、香港教育大学、香港都会大学、香港演艺学院、香港树仁大学、香港恒生大学、香港高等教育科 技学院、东华学院等12所院校采用单独招生形式,招生计划不分 省。报考这12所香港高校的考生须按照高校的要求提交入学申 请,参加高校组织的考试或面试(具体报名时间以各高校公布的 时间为准),由学校根据考生高考成绩和面试表现等其他要求录 取考生。需要注意的是,凡被香港12所单招院校录取并经本人向 录取院校确认就读的考生,不再参加内地高校的统一录取。

(2)澳门高校招生模式

澳门大学、澳门科技大学、澳门理工学院、澳门旅游学院、澳门镜湖护理学院、澳门城市大学招生计划不分省,均使用高考成绩,考生可根据高校的分数要求,在规定时间内登录高校网站 办理报名及填报志愿手续,由澳门高校择优录取。其中澳门城市 大学和澳门科技大学除招收本科生外,还招收部分预科生。允许 澳门高校录取的考生与内地其他院校间重复录取,报考澳门高校不会影响考生报考内地高校。若同时被澳门高校和内地高校录取,考生可自行决定是在澳门高校还是在内地高校就读。

2024年参加普通高考的考生若有意报考上述院校,可登录相应高校的网站获取信息。



36. 高考各阶段考生密码有哪些重置办法? 

考生应牢记在高考报名时所设置的密码,并且注意保密。在后期报名信息、体检信息、高考成绩查询以及志愿填报时都须使用此密码进行登录。

若考生遗忘了此密码,可通过以下途径进行密码重置,重置后的默认密码为考生身份证号后8位。(1)通过手机微信重置密码。考生使用手机关“河北省教 育考试院”微信公众号 (hbsksy),进入“密码找回”页面,通过人脸识别自助进行密码重置。

(2)通过计算机重置密码。考生使用能够访问互联网并装有摄像头的计算机,通过人脸识别自助进行密码重置。此方法需要安装人脸认证组件,下载方式及使用说明请通过志愿填报或考生

信息查询系统考生登录页面右下角“找回密码”链接进入查看。

(3)到就近报名点重置密码。考生携带本人身份证到任一报名点,通过读取身份证、人脸识别完成密码重置。

 


37. 考生如何查看自己的高考报名及体检信息? 

考生可凭考生号(或身份证号)和密码登录省教育考试院

“普通高校招生考试信息管理与服务平台”(网址:http://gk.he-beea.edu.cn),进入“信息查询”模块,查看自己的高考报名及体检信息。




河北省2024年普通高校

招生工作新视点

 1.深化艺术类专业考试招生改革。按照教育部统一部署,2024年起,普通高校招生艺术类专业考试分为美术与设计类、音乐类(包括音乐表演、音乐教育两类,其中音乐表演类考试包括声乐、器乐两个方向)、舞蹈类、表(导)演类(包括戏剧影视表演、服装表演、戏剧影视导演三个方向)、播音与主持类、书法类、戏曲类等7个科类。其中,美术与设计类、音乐类、舞蹈类、表(导)演类、播音与主持类、书法类等6个科类实行省级统考;戏曲类实行省际联考。

2024年起,进一步提高文化课成绩要求,艺术史论、戏剧影视文学等部分艺术类专业,直接依据考生高考文化课成绩择优录取;美术与设计类、书法类、舞蹈类、表(导)演类、音乐类等5类省级统考的高考文化课成绩占比调整为50%,播音与主持类仍为70%;校考专业高考文化课成绩要求,调整为执行普通类专业批次录取控制分数线(有关破格录取的,按高校公布的考试招生办法执行)。严格控制校考范围,仅少数高校经教育部批准后,可在省级统考基础上组织校考。

2.改革高校高水平艺术团招生办法。2024年起,高校高水平艺术团不再从高校招生环节选拔,由相关高校从在校生中遴选培养。

3.深化高校高水平运动队考试招生改革。2024年起,进一步完善和规范高校高水平运动队考试招生工作,通过优化招生项目范围,严格报考条件和资格审核,改进考试评价方式,提高文化成绩要求,完善招生录取机制,选拔培养德智体美劳全面发展且具有较高体育竞技水平的学生,为奥运会、世界大学生运动会等重大体育比赛和国家竞技体育后备人才培养体系提供人才支撑。

4.高校选考科目优化调整。教育部印发的《普通高校本科招生专业选考科目要求指引(通用版)》,从2024年高考开始实施。这次高校选考科目优化调整,主要是强化了相关专业对高中学习物理、化学科目的基础性要求。学生可根据自身兴趣爱好及特长,统筹考虑国家和社会需要,结合拟报考高校及专业(类)的选考科目要求,合理确定选考科目。5.优化高职单招考试模式。2024年起,报考高职单招的考生在高考报名所在县(市、区)参加考试;普通高中毕业生文化素质不再使用高中学业水平合格性考试成绩折算替代,所有考生均须参加考试;高职单招报考缴费、打印准考证、查询成绩、填报志愿、查询录取结果等均须登录河北省教育考试院官方网站进行;志愿填报数量由5所院校增加至10所院校。

重要日程备忘

image.png


image.png


 


1. 哪些人员需要参加高考报名?

(1)参加普通高校招生全国统一考试及河北省普通高中学业

水平选择性考试的考生;

(2)参加对口招生全省统一考试的考生;

(3)参加保送生等提前单独招生的考生;

(4)报考运动训练、武术与民族传统体育专业的考生;

(5)参加高职单招、残障单招、消防单招、职教师资单招、“3+4”对口贯通等经教育部和我省批准的普通高校单独考试招生的考生;

(6)其他参加普通高校招生录取或备案的考生。

报考高校少年班的学生,待相关高校确定合格名单后,再单独组织报名。


2. 考生在我省参加高考报名须具备什么条件?

(1)普通高校招生全国统一考试及河北省普通高中学业水平选择性考试(简称统考)报名条件:

①具有河北省户籍或符合条件的港澳台地区、外省进城务工人员随迁子女、在华取得永久居留权的外国人等非我省户籍人员;

②遵守中华人民共和国宪法和法律;

③高级中等教育学校毕业(含应届生)或具有同等学力; ④身体状况符合相关要求。

(2)中等职业学校对口升学全省统一考试(简称对口升学考)报名条件:

①具有河北省户籍,或符合条件的港澳台地区、外省进城务

2. 考生在我省参加高考报名须具备什么条件?

(1)普通高校招生全国统一考试及河北省普通高中学业水平选择性考试(简称统考)报名条件:

①具有河北省户籍或符合条件的港澳台地区、外省进城务工人员随迁子女、在华取得永久居留权的外国人等非我省户籍人员;

②遵守中华人民共和国宪法和法律;

③高级中等教育学校毕业(含应届生)或具有同等学力; 

④身体状况符合相关要求。

(2)中等职业学校对口升学全省统一考试(简称对口升学考)报名条件:

①具有河北省户籍,或符合条件的港澳台地区、外省进城务






3. 哪些人员不能参加高考报名?

(1)具有普通高等学历教育资格的高校在校生,或已被普通高校录取并保留入学资格的学生;

(2)高级中等教育学校非应届毕业的在校生;

(3)在高级中等教育阶段非应届毕业年份以弄虚作假手段报名并违规参加普通高校招生考试(包括统考、对口和高校单独组织的招生考试)的应届毕业生;

(4)因违反国家教育考试规定,被给予暂停参加普通高校招生考试处理且在停考期内的人员;

(5)因触犯刑法已被有关部门采取强制措施或正在服刑者,其中,未成年人按相关法律规定执行;

(6)外省在我省借读以及在我省中等职业学校等学校就读但户籍不在我省的应届生(符合报考条件的随迁子女除外);

(7)以“大中专招生”等名义将户口迁入我省学校集体户,但没有户籍所在学校学籍和连续就读记录的人员;

(8)已在外省参加2024年普通高考报名或拟在外省参加普通 高考报名的人员;

(9)现役军人。


4. 外省进城务工人员随迁子女在我省参加高考报名须具备 哪些条件?

在我省就业的外省进城务工人员随迁子女(以下简称随迁子女)参加我省高考报名,除符合我省高考报名条件外,同时还须具备以下条件:

(1)在父亲或母亲或法定监护人经常居住地(居住证所在)接受高中段教育,至毕业时具有两年(含)以上连续就学记(不含高中毕业后复读时间)和学籍;

(2)父亲或母亲或法定监护人具有居住证、就业创业证(或就业创业证电子凭证或就业失业登记证)。

提醒在读的高一、高二随迁子女的家长,须按照《河北省居住证实施办法(试行)》的有关规定,提早到当地公安部门申领居住证,以免影响考生高考报名。


5. 考生报考“三个专项”计划,须具备哪些条件?

(1)国家专项计划申报条件:

①在户籍所在县(市、区)报名,并符合2024年统一高考报 名条件;

②本人具有实施区域当地连续3年以上户籍,其父亲或母亲或法定监护人具有当地户籍;

③本人具有户籍所在县(市、区)高中连续3年学籍并实际就读。

申报国家专项计划须同时具备以上3项条件。

(2)高校专项计划申报条件:

①在户籍所在县(市、区)报名,并符合2024年统一高考报名条件;

②本人及父亲或母亲或法定监护人户籍地在实施区域的农村,本人具有当地连续3年以上户籍;

③本人具有户籍所在县(市、区)高中连续3年学籍并实际就读。

申报高校专项计划须同时具备以上3项条件。有关高校可在此基础上提出其他报考要求,并在招生简章中明确,提醒考生报 考时注意。

(3)地方专项计划申报条件:

①在户籍所在县(市、区)报名,并符合2024年统一高考报名条件;

②本人及父亲或母亲或法定监护人的户籍在实施区域的农村,且本人具有当地连续3年以上户籍;

③本人具有户籍所在县(市、区)高中连续3年学籍并实际就读。

申报地方专项计划须同时具备以上3项条件。

根据教育部规定,从2023年高考招生起,往年被专项计划录取后放弃入学资格或退学的考生,不再具有专项计划报考资格。





6. “三个专项”计划在我省实施区域和范围有哪些? (1)国家专项计划实施区域

石家庄市:赞皇县、灵寿县、行唐县、平山县;

秦皇岛市:青龙满族自治县;

邯郸市:大名县、魏县;

邢台市:广宗县、威县、新河县、巨鹿县、平乡县、临城县;

保定市:涞水县、阜平县、唐县、涞源县、望都县、易县、曲阳县、顺平县;

张家口市:宣化区(仅限原宣化县区域)、万全区、崇礼区、 张北县、康保县、沽源县、尚义县、蔚县、阳原县、怀安县、赤城县、涿鹿县赵家蓬区;

承德市:承德县、平泉市、隆化县、滦平县、丰宁满族自治

县、围场满族蒙古族自治县;

沧州市:海兴县、南皮县、盐山县;

衡水市:武强县、饶阳县、武邑县、阜城县。

(2)高校专项计划实施区域

石家庄市:赞皇县、灵寿县、行唐县、平山县;

秦皇岛市:青龙满族自治县;

邯郸市:大名县、魏县、广平县、馆陶县、鸡泽县、肥乡区、涉县;

  邢台市:广宗县、威县、新河县、巨鹿县、平乡县、临城 县、南和区、任泽区、内丘县、临西县;

  保定市:涞水县、阜平县、唐县、涞源县、望都县、易县、曲阳县、顺平县、博野县;

  张家口市:宣化区(仅限原宣化县区域)、万全区、崇礼区、张北县、康保县、沽源县、尚义县、蔚县、阳原县、怀安县、赤城县、涿鹿县;

  承德市:承德县、平泉市、隆化县、滦平县、丰宁满族自治

县、围场满族蒙古族自治县、宽城满族自治县、兴隆县;

 沧州市:海兴县、南皮县、盐山县、献县、东光县、吴桥 县、孟村回族自治县、肃宁县;

 衡水市:武强县、饶阳县、武邑县、阜城县、故城县、枣强县;

  廊坊市:大厂回族自治县。

(3)地方专项计划实施区域

地方专项计划在我省实施区域为县〔含县级市和自2014年起 经国务院批准的县(市)改区〕。




7. 考生报考免费医学定向计划须具备哪些条件?

(1)在户籍地县(市、区)报名,并符合2024年统一高考报名条件;

(2)本人及父亲或母亲或法定监护人户籍地须在农村,本人具有当地连续3年以上户籍。

  申报免费医学定向计划须同时具备以上2项条件。


8. 考生报名时需要提供哪些材料?

(1)考生报名时应提供户口本、本人居民身份证、高中学业 水平考试准考证或高级中等教育毕业证书(含同等学力证件等有 效证件、证明)、思想政治品德考核鉴定等基本材料;

(2)残疾考生(含参加残障单招、优秀残疾人运动员免试入学的考生)须提供本人残疾人证,并准确填写本人残疾人证号。申请享受2024年高考合理便利的残疾考生还须按报名点要求,如实填写“残疾人报考河北省普通高等学校招生全国统一考试合理便利申请表”;

(3)在职职工应有单位介绍信;

(4)自主就业的退役士兵应有当地退役军人事务部门出具的介绍信;

(5)拟报考2024年面向退役士兵高职单招专项计划的退役士兵考生,应提供退出现役证及经当地县级退役军人事务管理部门认定盖章的“2024年高考报名退役士兵身份认定表”;

(6)在外省就读的我省户籍普通高中应届毕业生,回我省参加高考报名,应提供学籍所在地相关省级部门出具的高中学业水平考试成绩证明、综合评价信息材料;

(7)随迁子女考生须提供高中段学籍证明、毕业学校出具的连续学习记录证明、本人居民身份证、户口本、父亲或母亲或法定监护人在县级及以上的公共就业服务机构领取的就业创业证(或就业创业证电子凭证或就业失业登记证)和在公安派出所领 取的有效期内居住证;

(8)在我省长期居住并具备报名资格的港、澳地区居民,须提供香港或澳门居民身份证和港澳居民来往内地通行证(或中华人民共和国港澳居民居住证)。台湾地区居民须提供在台湾居住的有效身份证明和台湾居民来往大陆通行证(或中华人民共和国台湾居民居住证);

(9)在华取得永久居留权的外国人,须提供在有效期内的中华人民共和国外国人永久居留身份证。



9. 高考报名时间和地点是怎样规定的?

 2024年我省高考报名工作从2023年10月30日9时开始,至11月13日17时结束。报名结束后不再安排补报名。考生原则上在户籍所在地招生考试机构指定的报名点报名,

应届考生也可以在学籍所在地报名。社会考生(包括复读)因特殊情况确需跨市报名的,须考生就读地市级招生考试机构与户籍地市级招生考试机构协商同意,但由此引发的后果由考生自己 负责。

拟报考三个专项计划、免费医学定向计划和享受少数民族加分资格的考生,须在户籍地报名。

符合在我省报考条件的随迁子女,须在其父亲或母亲或法定监护人居住证所在市级招生考试机构指定的报名点报名。

在我省长期居住并具备报名资格的港、澳地区居民和台湾地区居民,在居住地市级招生考试机构指定的报名点报名。

在华取得永久居留权的外国人,在居住地市级招生考试机构指定的报名点报名。




10. 报名工作流程是怎样的?

2024年高考报名包括资格初审、网上申报、资格审核、现场确认、交费等环节。

(1)报名资格初审。考生应按照报名点安排,在规定时间 内,持有效证件(证明)到指定报名点进行报名资格初审。填写 “河北省普通高校招生考生报名登记表(草表)”“河北省普通高校招生考生报名有关资格申请表(草表)”等表格。报名点对通过报名资格初审的考生进行身份证基础信息(包括姓名、性别、民族、身份证号、出生年月和身份证照片等信息)采集。

(2)网上申报。完成报名资格初审的考生,在规定时间内登录河北省教育考试院“普通高校招生考试信息管理与服务平台(网址: http://gkhebeea.eclu.cn, 或由省教育考试院网站 http://

www.hebeea.edu.cn右侧导航栏的“普通高考信息服务”进入)进行网上申报。网上申报主要内容有:

①密码设置。考生首次登录须修改初始密码(初始密码为身份证号后8位)。密码设置要有一定强度,不要过于简单。考生密码不仅用于报名系统登录、报名信息查看,后续的成绩查询、志愿填报、录取结果查询等都需要使用密码登录。因此考生要妥善 保管,严防密码泄漏。考生遗忘密码可持本人身份证到报名点进行重置。

②基本信息填报。考生网上申报时,须按要求核对身份证信息,填报户籍信息、简历、就读信息、联系方式、录取通知书邮 寄地址及父亲、母亲或法定监护人的相关信息等。其中,随迁子 女考生还须如实填写父亲或母亲或法定监护人身份证号、居住证 编号等信息;残疾考生(含参加残障单招、优秀残疾人运动员免 试人学的考生)须如实填写本人残疾人证号及残疾类型,否则会影响普通高校招生全国统一考试合理便利的申请或单独招收残疾 考生高校的报考。

③考试信息填报。考试信息包含考试类型、报考类别、外语 语种等。其中考试类型分为“统考”和“对口”,统考报考类别分为普通、艺术、体育。

④相关资格申请。包括优惠加分、优先录取、三个专项计划、订单定向免费医学生计划、残疾考生高考合理便利等资格的申请。考生满足相关条件方可申请并按要求提供相关审核材料,未在网上申请或未按要求提供审核材料的视为放弃资格。

(3)报名资格审核和现场确认。考生资格初审和网上申报完 成后,在规定时间内持身份证等有效证件(证明)及相关材料到指定报名点进行报名资格审核并完成图像采集、信息确认等工 作。考生本人须认真核对“2024年河北省普通高校招生考生报名 登记确认表”并由本人签字确认(报名登记确认表不得由他人代 )。报名信息签字确认后不能更改。如因考生个人原因造成信息填报错误的,后果由考生本人承担。未进行信息确认的报名信息无效。

(4)交纳报名费。考生完成信息确认后,按要求网上支付报名费,支付成功即报名完成。未交纳报名费的考生,报名信息无效,已交纳的报名费不再退还。


 image.png



 



 


     11. 考生报名时如何选择考试类型和报考类别?

考试类型分为“统考”和“对口”。

(1)统考。参加普通高校招生全国统一考试及河北省普通高 中学业水平选择性考试的考生、参加经教育部批准的高校单独招生以及其他需要通过普通高校招生录取或备案的考生(如报考保送生、国家体育总局组织文化考试的体育单招、消防单招、残障 单招、职教师资班等)选择“统考”。

统考报考类别分为普通、艺术、体育。

①普通:拟报考非艺术、体育专业的考生报考类别应选普通。报考普通类的考生不能兼报艺术、体育类。

②艺术:拟报考艺术专业的考生报考类别应选艺术。未按艺术报名的考生不能参加本年度艺术类专业考试,同时不能录取至使用艺术专业考试成绩的相关艺术类专业。

③体育:拟报考体育专业的考生报考类别应选体育。未按体育报名的考生,不能参加本年度我省普通体育类专业测试及录取。体育专业考试实行全省普通体育类专业统一测试。按照教育部规定,2024年起,健美操、啦啦操等体育类项目 纳入体育类专业考试招生,不得通过艺术类专业考试方式进行 招生。

(2)对口。参加中等职业学校对口升学全省统一考试的考 生,考试类型选择“对口”。中等职业学校和技工学校的毕业考 生可以报考,其他学校〔包含普通高中、高中毕业同等学力、其他中等学历教育、高职(专科)学历教育、本科()以上学历 教育〕毕业的考生不能报考。

对口报考专业类分为旅游类、财经类、农林类、畜牧兽医 类、机械类、电子电工类、建筑类、计算机类、医学类、学前教 育类。考生只能选择与高级中等教育阶段所学专业相对应的对口专业类报考。报考机械类、农林类的考生还须选择测试项目。




12. 报考艺术类考生如何选择艺术类别?

高考报名时“报考类别”选择“艺术”的考生须从音乐表演 (声乐)、音乐表演类(器乐)、舞蹈类、美术与设计类、戏剧 影视表演类、服装表演类、戏剧影视导演类、播音与主持类、书 法类、戏曲类等10个“艺术类别”中选择1类报考,不可兼报。报考音乐表演类(声乐)的考生,可选择“兼考音乐教育类(声 乐主项)”,同时获得音乐教育类(声乐主项)报考资格;报考 音乐表演类(器乐)的考生,可选择“兼考音乐教育类(器乐主 )”,同时获得音乐教育类(器乐主项)报考资格。报考音乐 表演类(声乐)、音乐表演类(器乐)、舞蹈类的考生须在“声 乐、器乐、舞蹈艺术统考细项”中分别对应填报一项唱法、器 种、舞种。兼考音乐教育类的考生,还须分别对应填报一项“声 乐、器乐副项艺术统考细项”,主项选择声乐的考生,副项须选 择器乐;主项选择器乐的考生,副项须选择声乐。器种为“中西 打击乐”专业考生,还须具体填报两种不同打击乐器,其中至少一种为音高类打击乐器。

具体报考流程见以下图例。


image.png

 


13. 考生可以选择哪些外语语种?

普通高考外语语种有英语、俄语、日语、德语、法语和西班牙语6种,考生只能选择其中一种。对口招生外语语种为英语。


14. 享受优惠加分政策的考生如何申请加?

拟享受优惠加分的考生,须在规定期限内提供相应的证件、材料向相关主管部门提出申请,相关主管部门对所有拟享受高考优惠加分的考生进行资格审核。未向主管部门申报或未通过资格审核和信息公示的考生,录取时不予优惠加分。

优惠加分项目审核部门和审核登记所需材料如下:

(1)少数民族自治县(含民族县)的少数民族考生。

考生在报名时提出申请,由考生报名地县级招生考试机构负 责审核登记。审核所需材料包括:①户口本(含考生本人及父亲或母亲或法定监护人户籍);②身份证;③户籍所在县学籍学校 满3年学籍证明;④户籍所在县学籍学校3年实际就读证明等。

少数民族自治县(含民族县)享受加分的少数民族考生须在

户籍所在县参加高考报名,且考生本人具有户籍所在县高中阶段连续3年完整户籍、学籍并实际就读,其父亲或母亲或法定监护人具有当地户籍。

我省的少数民族自治县(含民族县)包括:宽城县、丰宁县、围场县、青龙县、大厂县、孟村县、滦平县、隆化县、平泉市。

(2)归侨、归侨子女、华侨子女和台湾省籍(含台湾户籍)考生。

考生在报名时提出申请。其中,归侨、归侨子女、华侨子女考生,要按照侨务部门规定的时间、地点和所需材料到户籍地县 级以上侨办进行资格审核。台湾省籍(含台湾户籍)考生,由户 籍地台湾事务办公室进行资格审核。

归侨考生审核所需材料包括:①户口本和身份证;

②县级以上侨务部门出具的考生归侨身份证明③“三侨”考生身份审查 表等。

归侨子女考生审核所需材料包括:①考生户口本和身份证;

 (母)户口本;③父(母)单位人事部门出具的归侨身份、 与考生关系的证明;④“三侨”考生身份审查表等。

华侨子女考生审核所需材料包括:①考生户口本和身份证; 

②父母一方护照复印件、定居证复印件;

③我驻外使领馆出具的 华侨身份认证;

④我驻外使领馆或国内单位人事部门出具的与考 生的亲属关系证明;⑤“三侨”考生身份审查表等。

台湾省籍(含台湾户籍)考生审核所需材料包括:①台湾省(不含台湾户籍)考生需提供户口本、身份证。

②台湾户籍考 生需提供有效身份证明和台湾居民来往大陆通行证(或中华人民 共和国台湾居民居住证)等。

(3)自主就业退役士兵考生。

考生在报名时提出申请,由档案所在地退役军人事务部门负责审核登记。主要审核查验退出现役证。

(4)烈士子女考生。

考生在报名时提出申请,由户籍地退役军人事务部门负责审核登记。审核所需材料包括:①烈士证明书;②烈士与考生关系证明等。

(5)服役期间荣立二等功(含)以上考生和在服役期间被战

(原大军区)以上单位授予荣誉称号的退役军人考生。

考生在报名时提出申请,由档案所在地退役军人事务部门负责审核登记。审核所需材料包括

①退出现役证;

②奖(勋)

章、立功受奖证书等。

(6)具有河北户籍农村独生子女考生。

由卫生健康部门负责审核。考生在规定时间内向户口所在地 卫生健康部门提出申请,审核所需材料包括:

①参加高考的证 明;②户口本;③身份证;④独生子女父母光荣证;⑤农村居民 户口状况证明;⑥违法生育的处理情况证明;⑦如实填写河北省 独生子女审定表(一式四份);⑧考生本人近期四张小二寸免冠 照片;⑨父母离婚、丧偶的,要按照卫生健康部门的要求提供相 关证明材料等。

审核通过的农村独生子女考生,按照报名点规定的时间,将 经县(市、区)、乡(镇、街道)两级卫生健康部门审核盖章的 河北省农村独生子女审定表交报名点,由报名点汇总后,报县级 招生考试机构。

2016年1月1日(含)以后出生的农村户口独生子女考生参加高考,不再享受加分。





15. 符合优先录取条件的考生如何申请?

(1)   符合优先录取条件的军人子女,不在报名时申请,以相关部门提供的名单为准。资格审核工作由省军区负资。

(2)符合优先录取条件的公安烈士、公安英模、因公牺牲和一级至四级因公伤残公安民警子女,不在报名时申请,以相关部门提供的名单为准。资格审核工作由公安部门负责。

(3)符合优先录取条件的残疾人民警察、退出部队现役考生在报名资格初审时携带伤残人民警察证、退役军人事务部门出具的身份证明和退出现役证等证件材料向报名点提出申请,并在网上报名时进行申请。

(4)符合优先录取条件的消防救援队伍人员及其子女,按照相关规定执行,以应急部门提供的名单为准。资格审核由应急管理部门负责。

(5)经共青团中央青年志愿者守信联合激励系统认定,获得5A 级青年志愿者考生,以相关部门提供的名单为准。




16. 报考普通体育类专业有哪些具体要求?

普通体育类专业包括体育教育、社会体育指导与管理等。符2024年河北省普通高等学校招生报名条件、年龄不超过22周岁的考生均可按普通体育类报名。教练员、体育教师、优秀运动员 (指省级以上优秀运动队的队员)可放宽到28周岁。一般要求男生 身高170 cm 以上,女生160 cm 以上;任何一眼裸视不低于4.7。

凡按普通体育类参加高考报名的考生均须参加河北省统一组 织的专业考试和文化考试。未按体育类报名的考生,不能参加本 年度我省普通体育类专业测试及录取。


17. “统考”和“对口”两类考试能否兼报?

“统考”和“对口”是两种考试类型,两者不能兼报。普通高中生只能选择“统考”,不能选择“对口”;各类中等职业学校 (含中等师范、职业高中及其他中等专业学校)和技工学校毕业考生既可选择“对口”也可选择“统考”,但只能选择一种类型。



18.报名和参加各种专业(专门)考试及文化统一考试时是否需要携带本人居民身份证?

所有报名参加高考的考生在报名、参加各种专业(专门)考试及文化统一考试时,必须交验本人居民身份证。因此,考生必 须在高考报名前及时办理好居民身份证。身份证遗失或无法读取 的考生应及时进行补办,以免影响高考报名和考试。高一和高二年级的学生也应尽早办理居民身份证。




19. 身份证、户口簿、学考及学籍的姓名、身份证号、民 族、出生年月等内容不一致的如何办理报名手续?

如果身份证、户口簿、学考及学籍的姓名、身份证号、民族、出生年月等内容不一致,会影响考生的考试、录取、学籍注册及毕业,必须按管理权限由相关主管部门核准更正一致后,方可参加高考报名。


20. 在外省就读回我省参加高考的考生,普通高中学生学业 水平考试和综合素质评价信息如何办理?

在校生须本人持转出省省级学业水平考试主管机构出具的学业水平考试成绩证明、考生居民身份证到我省现就读学校办理学业水平考试成绩认证手续;非在校生须本人持转出省省级学业水平考试主管机构出具的学业水平考试成绩证明、考生居民身份证 到户口所在地招生考试机构办理学业水平考试成绩认证手续。

学籍转回我省普通高中的,按中学学籍管理规定和程序,由转人学校登录“河北省普通高中学生综合素质评价电子平台”,采集其综合素质评价信息。

学籍未转回我省的,到户口所在地招生考试机构指定的报名点进行报名。报名点为普通高中的,由报名点在规定时间内登录“河北省普通高中学生综合素质评价电子平台”采集其综合素质 评价信息;报名点不是普通高中的,由户口所在地招生考试机构委托一所设立报名点的普通高中,登录“河北省普通高中学生综合素质评价电子平台”采集其综合素质评价信息。

转回学生的高中综合素质评价信息的采集和学业水平考试成绩转人工作,一般截至高考当年5月31


21.考生号的作用是什么

  所有参加普通高校招生(含对口招生)的考生,均须通过报名获得考生号。考生号是考生参加专业考试、文化课考试、录取和进行电子备案的重要信息之一,对每位考生来说是唯一的,用于编排考场、建立电子档案和录取备案。


22. 准考证的作用是什么?

准考证是考生参加高考的时间、考试地点、考场座位的通知单,是考生参加高考时进入考点、考场的重要证件,是新生入学报到的重要依据。

考生参加考试入场时,监考人员将与各市招生考试机构打的准考证存根进行逐一核验。


23. 考生个人信息表的使用范围有哪些? 

报考艺术、体育、对口、保送生、强基计划、高水平运动队、体育单招及院校单独招生等需要专业(专门)考试的考生,如高校要求考生提供个人信息表,考生可登录省教育考试院“普通高校招生考试信息管理与服务平台” (由省教育考试院网站http://www.hebeea.edu.cn 右侧导航栏的“普通高考信息服务” 进入),进入“信息查询”模块,自行查询打印“2024年河北省普通高等学校招生考生个人信息表”,报名点不再盖章。


24.  考生为什么要签订 《河北省普通高校招生考试考生诚信承诺书》? 

为了加强对考生的招生考试诚信教育,使每一名考生都阅读 和了解《考场规则》《国家教育考试违规处理办法》《中华人民共 和国教育法》《中华人民共和国刑法》及《最高人民法院最高人 民检察院关于办理组织考试作弊等刑事案件适用法律若干问题的解释》中涉考条款等法规、规章,自觉遵守考试纪律,杜绝招生 考试中徇私舞弊和腐败行为的发生,教育部要求考生应签订诚信 承诺书。考生在招生考试中违反诚信承诺书的内容,违规行为将 会被记录到考生的诚信档案中。



诚信承诺书内容:

河北省普通高校招生考试考生诚信承诺书

我是参加2024年普通高校招生考试的考生,本人作如下承诺,如有违反,自愿承担有关后果并接受处理。

1. 保证考试招生各环节所涉各项个人信息(包括报名信息、志愿信息等)为本人填报,确保真实、准确、有效,按规定接受 相关信息公示,保证报名后至录取结束前,身份证号、姓名、民族、联系方式等关键身份信息不再修改,我省户籍考生的户籍不迁出我省。同意授权招生考试机构将个人信息用于招生考试有关工作使用。

2. 自觉遵守《考场规则》《违规行为的认定与处理》《中华人民共和国教育法》《中华人民共和国刑法(修正案九)》及 《最高人民法院最高人民检察院关于办理组织考试作弊等刑事案件 适用法律若干问题的解释》有关内容等各项招生考试法律、规定。

3. 如实、准确申报优惠加分、优先录取、三个专项计划、免费医学定向计划等资格,按要求提交相关审核材料,按规定参加资格审核。

4. 按照教育部要求参加体检,如实填写既往病史,遵守我 省体检规定。

5. 保证按要求确定选考科目,并及时交纳考试费。

6.  保证本人参加考试,并接受考试期间安全检查和考生身份鉴别,除考试用品外,其他规定以外的任何物品(如资料、手机等通信工具、手表等计时设备)不得带入考试区域。

7. 保证本人按规定填报高考志愿,对所报志愿负责。

承诺人:               ;身份证号:                       20            

备注:本联由报名点保存1年

………请…………沿…………此…………线…………撕…………下………


 


 

 


致考生的一封公开信


同学:你好!

为顺利完成普通高校招生考试工作,特向你真诚提示:

1. 报名及资格审核环节,如实填报相关信息,并按要求提交真实材料。

2. 按时参加体检,如实填写既往病史,遵守我省体检规定。

3. 按要求确定选考科目,并及时缴纳考试费。

4. 做好考前准备:熟知《考场规则》和答题规定;牢记各科考试时间,记准所在考点、考场位置和行车路线;出发时要带齐准考证、身份证及考试必备用品。

5. 严禁携带下列物品进入考场:各种与考试内容有关的材 料、存贮与考试内容有关的电子设备,手机和具有接收或发送信息功能的设备,手表等计时设备,涂改液、修正带等。如将规定 以外的物品带进考场且没放在指定位置,均按考试违规处理。

6. 自觉接受违规物品检查:不要穿戴带有金属的衣物、金 属饰品。规定以外的物品不要带入考场,已带入的要放到指定位置。自觉维护考试秩序,服从考试工作人员管理。

7. 珍爱诚信,拒绝违规:我省所有考点配备智能安检门, 实行“2+1”安检模式(考试封闭区域入口、考场入口使用金属探测器对考生进行安检,考试封闭区域入口使用智能安检门对考  生进行安检),手机集中统一存放管理。考试期间,考场周边有 无线电管理部门利用技术手段对高考有害信号进行跟踪和干扰, 考场内装有信号屏蔽器。对所有考点、考场进行全时段监控 像,考试结束后省教育考试院将组织考场录像回放。总之,无论  是考场发现的违规,还是通过录像回放发现的违规,我省均按教 育部规定取消其高考成绩和录取资格。作弊情节严重的,三年内 不准参加高考。《中华人民共和国刑法(修正案九)》及《最高 人民法院最高人民检察院关于办理组织考试作弊等刑事案件适用法律若干问题的解释》,将组织作弊、买卖作弊设备、买卖考题、替考等作弊以及帮助作弊行为纳入刑法范畴。考生作弊情况 将记入教育部的考生诚信档案并接受社会查询,考生将为自己的 违规行为付出沉重的代价。

8. 考生本人填报志愿,对所报志愿负责。

9. 高考报名后至录取结束前,身份证号、姓名、民族、联系 方式等关键身份信息不得修改,我省户籍考生的户籍不得迁出我省。

10. 提高安全意识,注意网报密码保管,不向他人提供、泄 露网报登录密码。

当前国家正在推进诚信体系建设,诚信考试是每一名公民应  尽的义务。希望大家自觉遵守考试纪律和规定,做到诚信应考、安全应考,不要给自己留下人生遗憾!

最后,预祝你取得好成绩!


25.  如何确定考试科目和交纳考试费?

已参加高考报名的考生,在2024年4月26—29日,须按照规定时间登录省教育考试院“普通高校招生考试信息管理与服务平台”(网址: http://gk.hebeea.edu.cn,  或通过省教育考试院网

 http://www.hebeea.edu.cn    右侧导航栏的“普通高考信息服”进入),选择统一高考文化考试(含全国统考、普通高中学业水平选择性考试)考试科目,同时交纳考试费。对口文化考试科目确认及交费同期进行。未交纳考试费的考生不得参加相应科目考试。

考生应对选考科目认真检查,一经确认不能再修改。对口升学专业考试、体育专业考试、艺术专业考试、高职单招、“3+4”对口转段等其他考试费用的收取,按有关文件执行。


26. 报考选择性考试科目有什么要求? 

参加普通高等学校招生全国统一考试的考生均应参加选择性考试,其中普通高中在校学生应在相应科目高中学业水平合格性考试合格的基础上报考选择性考试科目。




河北省2024年普通高校

招生工作新视点

 1.深化艺术类专业考试招生改革。按照教育部统一部署,2024年起,普通高校招生艺术类专业考试分为美术与设计类、音乐类(包括音乐表演、音乐教育两类,其中音乐表演类考试包括声乐、器乐两个方向)、舞蹈类、表(导)演类(包括戏剧影视表演、服装表演、戏剧影视导演三个方向)、播音与主持类、书法类、戏曲类等7个科类。其中,美术与设计类、音乐类、舞蹈类、表(导)演类、播音与主持类、书法类等6个科类实行省级统考;戏曲类实行省际联考。

2024年起,进一步提高文化课成绩要求,艺术史论、戏剧影视文学等部分艺术类专业,直接依据考生高考文化课成绩择优录取;美术与设计类、书法类、舞蹈类、表(导)演类、音乐类等5类省级统考的高考文化课成绩占比调整为50%,播音与主持类仍为70%;校考专业高考文化课成绩要求,调整为执行普通类专业批次录取控制分数线(有关破格录取的,按高校公布的考试招生办法执行)。严格控制校考范围,仅少数高校经教育部批准后,可在省级统考基础上组织校考。

2.改革高校高水平艺术团招生办法。2024年起,高校高水平艺术团不再从高校招生环节选拔,由相关高校从在校生中遴选培养。

3.深化高校高水平运动队考试招生改革。2024年起,进一步完善和规范高校高水平运动队考试招生工作,通过优化招生项目范围,严格报考条件和资格审核,改进考试评价方式,提高文化成绩要求,完善招生录取机制,选拔培养德智体美劳全面发展且具有较高体育竞技水平的学生,为奥运会、世界大学生运动会等重大体育比赛和国家竞技体育后备人才培养体系提供人才支撑。

4.高校选考科目优化调整。教育部印发的《普通高校本科招生专业选考科目要求指引(通用版)》,从2024年高考开始实施。这次高校选考科目优化调整,主要是强化了相关专业对高中学习物理、化学科目的基础性要求。学生可根据自身兴趣爱好及特长,统筹考虑国家和社会需要,结合拟报考高校及专业(类)的选考科目要求,合理确定选考科目。5.优化高职单招考试模式。2024年起,报考高职单招的考生在高考报名所在县(市、区)参加考试;普通高中毕业生文化素质不再使用高中学业水平合格性考试成绩折算替代,所有考生均须参加考试;高职单招报考缴费、打印准考证、查询成绩、填报志愿、查询录取结果等均须登录河北省教育考试院官方网站进行;志愿填报数量由5所院校增加至10所院校。

重要日程备忘

image.png


image.png


 


1. 怎样进行思想政治品德考核?

思想政治品德考核主要是考核考生本人的现实表现。考生所在学校或单位应对考生的政治态度、思想品德作出全面鉴定,并对其真实性负责。无就读学校或工作单位的考生原则上由所属的乡镇、街道办事处鉴定。

报考军队院校、公安院校、司法院校、北京电子科技学院等 有特殊要求院校(专业)的考生,还应进行相应的政治考核或考察。


2. 哪些情况属于思想政治品德考核不合格?

有下列情形之一,且未能提供对错误的认识及改正错误的现实表现等证明材料的,应认定为思想政治品德考核不合格。

(1)有反对宪法所确定的基本原则的言行或参加邪教组情节严重的;

(2)触犯刑法、治安管理处罚法,受到刑事处罚或治安管理 处罚且情节严重、性质恶劣,尚在处罚期内的。


3. 报考军队院校的考生怎样进行政治考核?

政治考核由考生户籍所在地或经常居住地的县(含自治县、不设区的市、市辖区,下同)人民政府兵役机关,按照军队征集和招录人员政治考核有关规定组织实施。考生户籍所在地、经常居住地与高考报名所在地不在同一县的,政治考核工作由考生高考报名所在地的县级人民政府兵役机关负责,考生户籍所在地、经常居住地的县级人民政府兵役机关配合做好相关工作。

政治考核结论分为合格和不合格两种。具体事宜,以当年文件规定为准。



4. 报考公安院校的考生怎样进行政治考察?

凡志愿报考公安普通高等院校的考生,按照省公安厅规定时间,通过“河北省公安厅公安院校公安专业招生信息服务平台”(由河北省公安厅官方网站 http://gat.hebei.gov.cn飘窗进入)进行政治考察报名。按提示注册登录、填写报名信息。报名成功后,考生(或考生监护人)携带打印的“2024年河北省普通高  学校招生考生个人信息表”、考生身份证、居民户口簿、一张近期免冠小2寸证件照,由考生监护人办理的还需携带办理人身份证,到考生户籍所在地公安派出所现场提出政治考察申请,未现场提出申请的,无法进行政治考察。

考察的项目和标准,参照公安机关录用人民警察的有关规定 执行。考察实施机关为县级公安机关,审核机关为市级公安机 关,由省公安厅政治部对考察情况进行分析,综合作出考察结论。考生可通过“河北省公安厅公安院校公安专业招生信息服务平台”查询政治考察结论。政治考察合格的考生,登录“河北省 公安厅公安院校公安专业招生信息服务平台”,在政治考察结论 查询页面如实填写“公安院校公安专业招生患病经历申报表”。

具体事宜,以当年文件规定为准。


5. 报考司法类高校提前录取专业的考生怎样进行政治考察?

有的司法类高校提前录取专业须单独组织政治考察,有的司法类高校涉警专业须组织政治考察、面试、体检及体能测试等,具体安排详见高校招生章程或向有关高校咨询。



6. 报考军队飞行学员的考生怎样进行政治考核?

  报考军队飞行学员的考生(须全面检测合格),由军队招飞机构组织政治考核。具体时间以军队招飞机构相关安排为准。


7. 报考北京电子科技学院的考生怎样进行政审?

报考北京电子科技学院的考生,填报志愿后由保密机要部门

组织政审。具体要求以学校当年公布的招生章程为准。



8. 综合素质评价包括哪些内容?

综合素质评价是对学生全面发展状况的观察、记录、分析,是发现和培养学生良好个性的重要手段,是深入推进素质教育的一项重要制度。综合素质评价注重考查学生社会责任感、创新精神和实践能力。评价内容包括思想品德、学业水平、身心健康、艺术素养、社会实践等五个方面。

普通高中生的综合素质评价由中学负责,学校将学生的写实记录信息上传至全省统一的普通高中综合素质评价电子平台,最终形成普通高中学生综合素质评价电子档案。


9. 综合素质评价如何使用?

普通高中学生综合素质评价作为高校招生录取的参考依据,投档时,我省将考生的综合素质评价情况提供给招生高校。具体使用办法由招生高校在招生章程中予以明确。




河北省2024年普通高校

招生工作新视点

 1.深化艺术类专业考试招生改革。按照教育部统一部署,2024年起,普通高校招生艺术类专业考试分为美术与设计类、音乐类(包括音乐表演、音乐教育两类,其中音乐表演类考试包括声乐、器乐两个方向)、舞蹈类、表(导)演类(包括戏剧影视表演、服装表演、戏剧影视导演三个方向)、播音与主持类、书法类、戏曲类等7个科类。其中,美术与设计类、音乐类、舞蹈类、表(导)演类、播音与主持类、书法类等6个科类实行省级统考;戏曲类实行省际联考。

2024年起,进一步提高文化课成绩要求,艺术史论、戏剧影视文学等部分艺术类专业,直接依据考生高考文化课成绩择优录取;美术与设计类、书法类、舞蹈类、表(导)演类、音乐类等5类省级统考的高考文化课成绩占比调整为50%,播音与主持类仍为70%;校考专业高考文化课成绩要求,调整为执行普通类专业批次录取控制分数线(有关破格录取的,按高校公布的考试招生办法执行)。严格控制校考范围,仅少数高校经教育部批准后,可在省级统考基础上组织校考。

2.改革高校高水平艺术团招生办法。2024年起,高校高水平艺术团不再从高校招生环节选拔,由相关高校从在校生中遴选培养。

3.深化高校高水平运动队考试招生改革。2024年起,进一步完善和规范高校高水平运动队考试招生工作,通过优化招生项目范围,严格报考条件和资格审核,改进考试评价方式,提高文化成绩要求,完善招生录取机制,选拔培养德智体美劳全面发展且具有较高体育竞技水平的学生,为奥运会、世界大学生运动会等重大体育比赛和国家竞技体育后备人才培养体系提供人才支撑。

4.高校选考科目优化调整。教育部印发的《普通高校本科招生专业选考科目要求指引(通用版)》,从2024年高考开始实施。这次高校选考科目优化调整,主要是强化了相关专业对高中学习物理、化学科目的基础性要求。学生可根据自身兴趣爱好及特长,统筹考虑国家和社会需要,结合拟报考高校及专业(类)的选考科目要求,合理确定选考科目。5.优化高职单招考试模式。2024年起,报考高职单招的考生在高考报名所在县(市、区)参加考试;普通高中毕业生文化素质不再使用高中学业水平合格性考试成绩折算替代,所有考生均须参加考试;高职单招报考缴费、打印准考证、查询成绩、填报志愿、查询录取结果等均须登录河北省教育考试院官方网站进行;志愿填报数量由5所院校增加至10所院校。

重要日程备忘

image.png


image.png


 



1.考生为什么要体检?

考生体检信息是考生档案材料的重要组成部分,是高校录取考生时重点审核的内容之一。因此,所有考生(包括保送生、高职单招、体育单招等各类单招)均须在招生考试机构规定的时间内,到指定体检站参加体检。非招生考试机构指定的体检站为考 生作出的体检结果无效,未按招生考试机构要求参加体检,将影响考生正常录取备案,后果由考生本人负责。



2. 考生体检应注意什么?

(1)了解体检政策。《教育部、卫生部、中国残疾人联合会关于印发〈普通高等学校招生体检工作指导意见〉的通知》(教学〔2003〕3号)、《教育部办公厅、卫生部办公厅关于普通高等学校招生学生人学身体检查取消乙肝项目检测有关问题的通知》 (教学厅〔2010〕2号)、《河北省招生委员会、河北省卫生计生委关于做好河北省普通高等学校招生体检工作的通知》冀招委普2014〕11号〕和《关于转发<教育部办公厅卫生部办公厅关于普通高等学校招生学生人学身体检查取消乙肝项目检测有关问题 的通知〉的通知》(冀招委普〔2010〕9号)等文件是体检工作主  要依据,考生要认真学习、了解并掌握相关政策规定。

(2)做好充分准备。第一,要保证较充足的睡眠。睡眠不足会导致免疫力下降,加上过度疲劳和考前的紧张,很容易患上感 冒,感胃可能导致转氨酶一过性增高,从而影响体检结果。所以体检前考生要合理利用时间,保证充分休息、充足睡眠。第二,要注意饮食,不吃辛辣、油腻和不易消化的食物,不饮酒,不做 剧烈运动等。第三,建议体检前尽量不服药物、不吃保健品。避免因药物引起转氨酶一过性增高影响了体检结果。若因疾病遵医 嘱不能停药时还可以继续服用,体检时应向医生讲明。第四,体 检前还要做好个人卫生,尽量穿宽松的衣服,便于体检,勿带贵重物品,以免丢失。抽血检查一般在上午进行,考生应空腹抽血,禁早餐。

(3)克服紧张心理。有些考生在体检时往往因心情过于紧张而导致血压升高、心率加快、口吃、视力下降、对色觉检查图谱 不能正确描述等,使体检结果不能如实反映身体状况。如体检量 血压时,有的考生缺乏常识,在精神没有平稳时急于检查,影响 了血压的稳定,出现血压升高的情况。又如,检查眼科有两方面的内容,即视力和色觉。在做这两方面检查时应注意:①在查视 力时不要用遮盖物压迫眼球,以免影响正常结果。②在色觉检查 时,医生要求受检者在指定时间内辨认色盲本内的数字或图形,因此,在受检时不要一翻开色盲本就立即回答,而应看准后再回答,但不要犹豫不决,超过规定时间才回答会被误定为色盲或色弱,从而影响某些专业的录取。

(4)坚持实事求是。考生体检时,要与体检医生积极配合,如实回答医生提出的问题,如实填写既往病史。不得由他人替检,不得弄虚作假。视力不好的考生,体检时不要戴隐形眼镜, 佩戴义眼或佩戴助听器的考生体检时向体检医生讲明。高校入学  体检复查时,被查出隐瞒病史或弄虚作假的将取消入学资格,请考生不要存有侥幸心理。

(5)重视体检结果。体检结束后,考生会得到《河北省普通高等学校招生考生体检结果通知单》。通知单内容包含考生体检 信息以及根据考生体检信息和《普通高等学校招生体检工作指导意见》得出的志愿填报指导参考,其对考生准确了解自身身体状况和填报志愿有非常重要的作用。体检结果是招生院校录取的重要依据,因此考生应特别重视体检结果通知单中的相关信息,拿到通知单后要认真核对,无误后签字确认。如果对体检信息有异议,可在规定时间内申请复检。

特别提醒考生,填报志愿时一定要认真对照《普通高等学校招生体检工作指导意见》及高校招生章程中的有关要求慎重选择专业,避免因为身体原因被高校退档而失去录取机会。






3. 高考体检有哪些项目?

高考体检共分眼科、耳鼻喉科、口腔科、外科、内科、化验 X 线摄影几部分。眼科主要检查视力、色觉和眼病;耳鼻喉科 主要检查听力、嗅觉、耳鼻咽喉疾病;口腔科检查唇腭、牙齿等;外科检查身高、体重、皮肤、颈部、四肢、面部、关节等;内科主要检查血压、心脏和腹部器官;化验主要检验转氨酶 (A.L.T) (一般在早晨进行,要求考生空腹)等X线摄影主要检查 胸、肺等器官。


4. 高考体检是怎样进行的?

考生体检在全封闭的体检站内进行,医务人员由体检领导小组负责聘请和培训,一律佩戴标志上岗。考生由中学教师带队,凭打印的带有考生照片的体检表或体检证进入体检站参加体检。无关人员不得进入体检站。体检结束后,打印体检信息核对表交由考生确认签字。

报考军队院校、公安院校、司法类院校提前录取专业及民航飞行技术专业的考生,还须参加有关部门另行组织的体检。


5. 对体检结果有异议的考生怎样申请复查?

对体检结果有异议的考生,应当场提出复检申请,或在得到本人体检结果5天内向当地招生考试机构申请复检,由各地复检医院进行复检,体检结果以复检结果为准。如考生对复检结果仍有异议,应于接到复检结果15日内提出终检申请,由终检医院对有异议的复检结果作出最终裁定。


6.《普通高等学校招生体检工作指导意见》的具体内容是什么? 

一、患有下列疾病者,学校可以不予录取

1. 严重心脏病(先天性心脏病经手术治愈,或房室间隔缺损分流量少,动脉导管未闭返流血量少,经二级以上医院专科检 查确定无须手术者除外)、心肌病、高血压病。

2. 重症支气管扩张、哮喘,恶性肿瘤、慢性肾炎、尿毒症。 

3. 严重的血液、内分泌及代谢系统疾病、风湿性疾病。

4. 重症或难治性癫痫或其他神经系统疾病;严重精神病未治愈、精神活性物质滥用和依赖。

5. 慢性肝炎病人并且肝功能不正常者(肝炎病原携带者但肝功能正常者除外)。

6. 结核病除下列情况外可以不予录取。

(1)原发型肺结核、浸润性肺结核已硬结稳定,结核型胸膜炎已治愈或治愈后遗有胸膜肥厚者;

(2)一切肺外结核(肾结核、骨结核、腹膜结核等)、血行性播散型肺结核治愈后1年以上未复发,经二级以上医院(或结 核病防治所)专科检查无变化者;

(3)淋巴腺结核已临床治愈无症状者。

二、患有下列疾病者,学校有关专业可不予录取

1. 轻度色觉异常(俗称色弱)不能录取的专业:以颜色波长作为严格技术标准的化学类、化工与制药类、药学类、生物科  学类、公安技术类、地质学类各专业,医学类各专业;生物工程、生物医学工程、动物医学、动物科学、野生动物与自然保护区管理、心理学、应用心理学、生态学、侦察学、特种能源工程与烟火技术、考古学、海洋科学、海洋技术、轮机工程、食品科学与工程、轻化工程、林产化工、农学、园艺、植物保护、茶  学、林学、园林、蚕学、农业资源与环境、水产养殖学、海洋渔业科学与技术、材料化学、环境工程、高分子材料与工程、过程  装备与控制工程、学前教育、特殊教育、体育教育、运动训练、

运动人体科学、民族传统体育各专业。

   2. 色觉异常Ⅱ度(俗称色盲)不能录取的专业:除同轻度色觉异常外,还包括美术学、绘画、艺术设计、摄影、动画、博物馆学、应用物理学、天文学、地理科学、应用气象学、材料物理、矿物加工工程、资源勘探工程、冶金工程、无机非金属材料工程、交通运输、油气储运工程等专业。专科专业:与以上专业 相同或相近专业。

3. 不能准确识别红、黄、绿、蓝、紫各种颜色中任何一种颜色的导线、按键、信号灯、几何图形者不能录取的专业:除同轻度色觉异常、色觉异常Ⅱ度两类列出专业外,还包括经济学类、管理科学与工程类、工商管理类、公共管理类、农业经济管理类、图书档案学类各专业。不能准确在显示器上识别红、黄、 绿、蓝、紫各颜色中任何一种颜色的数码、字母者不能录取到计算机科学与技术等专业。

4. 裸眼视力任何一眼低于5.0者,不能录取的专业:飞行技术、航海技术、消防工程、刑事科学技术、侦察。专科专业:海洋船舶驾驶及与以上专业相同或相近专业(如民航空中交通管 )。

5. 裸眼视力任何一眼低于4.8者,不能录取的专业:轮机工程、运动训练、民族传统体育。专科专业:烹饪与营养、烹饪艺等。

三、患有下列疾病不宜就读的专业

1. 主要脏器:肺、肝、肾、脾、胃肠等动过较大手术,功  能恢复良好,或曾患有心肌炎、胃或十二指肠溃疡、慢性支气管炎、风湿性关节炎等病史,甲状腺功能亢进已治愈一年的,不宜就读地矿类、水利类、交通运输类、能源动力类、公安学类、体育学类、海洋科学类、大气科学类、水产类、测绘类、海洋工程类、林业工程类、武器类、森林资源类、环境科学类、环境生态类、旅游管理类、草业科学类各专业,及土木工程、消防工程、 农业水利工程、农学、法医学、水土保持与荒漠化防治、动物科学各专业。专科专业不宜就读烹饪工艺、西餐工艺、面点工艺、 烹饪与营养、表演、舞蹈学、雕塑、考古学、地质学、建筑工程、交通土建工程、工业设备安装工程、铁道与桥梁工程、公路与城市道路工程、公路与桥梁工程、铁道工程、工业与民用建筑

工程专业。

2. 先天性心脏病经手术治愈,或房室间隔缺损分流量少,动脉导管未闭返流血量少,经二级以上医院专科检查确定无须手

术者不宜就读的专业同第三部分第一条。

3. 肢体残疾(不继续恶化),不宜就读的专业同第三部分第一条。

4. 屈光不正(近视眼或远视眼,下同)任何  眼矫正到 4.8、镜片度数大于400度的,不宜就读海洋技术、海洋科学、测 控技术与仪器、核工程与核技术、生物医学工程、服装设计与工程、飞行器制造工程。专科专业:与以上相同或相近专业。

5. 任何一眼矫正到4.8、镜片度数大于800度的,不宜就读 地矿类、水利类、土建类、动物生产类、水产类、材料类、能源 动力类、化工与制药类、武器类、农业工程类、林业工程类、植 物生产类、森林资源类、环境生态类、医学类、心理学类、环境 与安全类、环境科学类、电子信息科学类、材料科学类、地质学  类、大气科学类及地理科学、测绘工程、交通工程、交通运输、 油气储运工程、船舶与海洋工程、生物工程、草业科学、动物医 学各专业。专科专业:与以上相同或相近专业。

6. 一眼失明另一眼矫正到4.8、镜片度数大于400度的,不 宜就读工学、农学、医学、法学各专业及应用物理学、应用化学、生物技术、地质学、生态学、环境科学、海洋科学、海洋技术、生物科学、应用心理学等专业。

7. 两耳听力均在3米以内,或一耳听力在5米另一耳全华的,不宜就读法学各专业、外国语言文学各专业以及外交学、新闻学、侦察学、学前教育、音乐学、录音艺术、土木工程、交通 运输、动物科学、动物医学各专业、医学各专业

8. 嗅觉迟钝、口吃、步态异常、驼背、面部疤痕、血管瘤、黑色素痣、白瘢风的,不宜就读教育学类、公安学类各专业以及外交学、法学、新闻学、音乐表演、表演各专业。

9. 斜视、嗅觉迟钝、口吃不宜就读医学类专业。

此部分内容供考生在报考专业志愿时参考。学校不得以此为依据,拒绝录取达到相关要求的考生。

未列入专业目录或经教育部批准有权自定新的学科专业,学校招生时可根据专业性质、特点,提出学习本专业对身体素质、生理条件的要求,并在招生章程中明确刊登,做好咨询解释工作。







7. 报考军队院校军检是如何进行的?

军检包括面试、体格检查和心理检测,由省军区统一组织。

省军区根据考生报考志愿和报考基本条件,区分院校、首选 科目、男女生,按照各院校招生计划数的一定比例,从高分到低 分的顺序,确定参加军检对象。

全省在部分市设立军检站,由所在地军分区(警备区)负责 组织实施。体格检查标准按照《军队选拔军官和文职人员体检标 准》(军后卫〔2023〕116号)执行,区分通用标准和选拔军官补充标准两个部分,拟报考特种作战、通信导航、电子对抗、防化、机降、伞降、医疗卫生、舰艇、潜艇、潜水、装甲、油料、测绘、导弹等特殊岗位招生专业的考生,在符合通用标准的基础上,还须达到相应的补充标准,具体详见当年招生计划专业备注中体检标准要求。

面试主要考察了解考生的报考动机、形象气质、逻辑思维和 语言表达等方面的基本素质,通常采取目测、口令调整和语言交流等方法进行。

具体事宜以当年文件为准。


8. 军队院校招收学员体格检查标准具体内容是什么? 

通用标准

外科项目

第一条男性身高低于162 cm,女性身高低于158 cm,不合格。

第二条 体重指数(BMI)在下列范围的,不合格。

(一)男性体重指数低于17 .5或者30以上,女性体重指数低

17或者24以上;

(二)男性体重指数17.5以上且低于30、女性体重指数17以上且低于24,空腹血糖7.0 mmol/L以上;

(三)男性体重指数28以上且低于30,糖化血红蛋白6.5%

以上。

第三条 颅脑损伤,颅骨畸形,颅骨缺损或者颅盖凹陷,颅内异物存留,颅骨及颅内手术史,脑外伤后综合征,不合格。

第四条 颈部运动功能受限,斜颈,Ⅲ度单纯性甲状腺肿,甲状腺切除术后,不合格。可自行矫正的斜颈,合格。

第五条 脊柱、骨盆、胸廓的畸形或者骨折及骨折史,脊柱、骨盆、胸廓的手术史,腰椎间盘突出症及病史,强直性脊柱 炎及病史,其他明显影响脊柱功能的疾病及病史,不合格。

下列情况合格:

(一)轻度胸廓畸形;

(二)可自行矫正的脊柱侧弯;

(三)无合并伤的肋骨单纯性骨折,2根以下,治愈1年以 X线片显示骨折线消失,无功能障碍及其后遗症。

第六条 骨、关节畸形或者残缺骨、关节、滑囊疾病或者

损伤及其后遗症,关节习惯性脱位,重度扁平足,肘关节过伸大15度,肘关节外翻大于20度,肘关节过伸或者外翻虽未超过 前述规定但存在功能障碍,影响肢体功能的腱鞘疾病,周围神经 损伤影响功能,不合格。

下列情况合格:

(一)四肢骨单纯性骨折,治愈1年以上 X线片显示骨折线消失,复位良好,无功能障碍及其后遗症;

腋臭术后超过6个月,无并发症,无腋臭或者轻度腋臭,合格。

第十一条  脉管炎,动脉瘤,中、重度下肢静脉曲张,不合格。

第十二条  泌尿生殖系统疾病或者损伤及其后遗症,生殖器官畸形或者发育不全,临床型Ⅲ度精索静脉曲张,不合格。

下列情况合格:

(一)无自觉症状的轻度非交通性精索鞘膜积液,不大于健侧宰九;

(二)无自觉症状的宰九鞘膜积液,包括睾丸在内不大于健侧睾丸1倍;

(三)包茎,包皮过长,轻度急性包皮炎,阴囊炎;

(四)精索静脉曲张手术半年以上,无复发,无后遗症;

(五)交通性鞘膜积液手术半年以上,无复发,无后遗症;

(六)隐睾经手术下降至阴囊,术后半年以上,无并发症,后遗症;

(七)附宰囊肿,无自觉症状,经专科诊断无需手术。

第十三条 胸、腹腔手术史,疝,不合格。

下列情况合格:

(一)闲尾炎手术半年以上,无后遗症;

(二)腹外疝手术半年以上,无后遗症;

(三)胆石症微创手术(保留胆囊)半年以上,无后遗症;

(四)胸腔镜下交感神经链切断术半年以上,无后遗症。

第十四条 脱肛,肛痿,肛旁脓肿,重度陈旧性肛裂,环状 痔,混合痔,不合格。

下列情况合格:

(一)混合痔2个以内,且长径均1cm 以下;

(二)低位单纯性肛痿(只有 一 个痿管),术后1年以上,无复发。

第十五条 梅毒、淋病、非淋菌性尿道炎、尖锐湿疣、生殖器疱疹、软下痞、性病性淋巴肉芽肿,以及其他性传播疾病及其并发症、后遗症,不合格。

第十六条 恶性肿瘤及病史,恶性肿瘤术后,不合格。

面颈部长径超过1cm的良性肿瘤、囊肿,其他体表部位长径超过3cm 的良性肿癌、囊肿,或者虽未超出前述规定但影响功能和训练的,不合格。

颅脑、纵隔、心脏的良性肿癌、囊肿,不合格。

甲状腺、乳腺、眼、耳、鼻、咽、喉、口腔等其他部位的良性肿瘤、囊肿,无需手术治疗或者已行手术符合恢复时限,术后无并发症、后遗症,不影响功能和训练,合格。

内科项目

第十七条  高血压,器质性心脏病(含先天性心脏病术后),严重的血管疾病,右位心脏,不合格。

下列情况合格:

(一)听诊发现心律不齐、心脏杂音,经检查系生理性;

(二)心脏射频消融术后痊愈半年以上,无复发,无并发症,无需服药,心电图及心脏彩色多普勒超声检查正常。

第十八条  收缩压低于90 mmHg  或者140 mmHg  以上,舒张压低于60 mmHg 或者90 mmHg 以上,不合格。

第十九条 心率低于50次/分或者高于100次/分,不合格。

心率高于100次/分且在110次/分以下,排除病理性原因,合格。

第二十条 慢性支气管炎,支气管哮喘,支气管扩张,肺大泡,肺纤维化,气胸及气胸史,以及其他呼吸系统慢性疾病,不合格。

第二十一条 严重的消化道溃疡、慢性胰腺炎、溃疡性结肠炎、克罗恩病、肠结核等严重的消化系统疾病,不合格。

第二十二条  泌尿、血液、内分泌系统疾病,代谢性疾病免疫性疾病,以及经确诊的以上慢性严重性疾病治愈未达稳定年限,不合格。

下列情况合格:

(一)急性膀胱炎、急性肾孟肾炎、急性前列腺炎,治愈半 年以上,无复发,无并发症;

(二)单纯性缺铁性贫血,血红蛋白大于90 g/L;

(三)巨幼细胞贫血治愈5年以上,血常规检查正常;

(四)儿童青少年时期患过敏性紫癜,治愈10年以上,无复 发,无并发症,血、尿常规等相关检查正常;

(五)亚急性甲状腺炎治愈1年以上,甲状腺功能(甲功五 )正常,无需服药,无症状和体征。

第二十三条  鼠疫,霍乱,艾滋病,病毒性肝炎,结核病, 传染性肺炎,人感染高致病性禽流感,新型冠状病毒、腺病毒、 诺如病毒感染,细菌性和阿米巴性痢疾,伤寒和副伤寒,沙门菌  肠炎,脊髓灰质炎,麻疹,风疹,流行性出血热,流行性乙型脑炎,流行性脑脊髓膜炎,流行性腮腺炎,急性出血性结膜炎,麻  风病,白喉,猩红热,狂犬病,布鲁菌病,疟疾,炭疽,钩端螺旋体病,血吸虫病,丝虫病,黑热病,水症等具有传染性的疾  病,不合格。

下列情况合格:

(一)急性甲型、戊型病毒性肝炎,治愈半年以上,无症状 和体征,实验室检查正常,无复发;急性乙型、丙型、丁型病毒性肝炎,治愈1年以上,无症状和体征,实验室检查正常,无复发;

(二)肺结核、浅表淋巴结结核、结核性胸膜炎无胸膜肥厚 或者粘连、结核性腹膜炎无腹膜肥厚或者粘连、结核性脑膜炎无后遗症,治愈3年以上,无复发(泌尿生殖系统结核除外);

(三)细菌性痢疾、阿米巴性痢疾、伤寒、副伤寒、沙门菌 肠炎,治愈半年以上,无复发;

(四)布鲁菌病、疟疾、黑热病、血吸虫病、钩端螺旋体病、 流行性出血热,治愈1年以上,无复发,无后遗症;

(五)丝虫病,治愈半年以上,无后遗症;

(六)腺病毒感染,治愈1年以上,无后遗症;

(七)诺如病毒感染,治愈3个月以上;

(八)其他传染性疾病,已达到临床治愈标准且病原体被 清除。

第二十四条  癫痫、脑血管疾病、脱髓鞘性疾病、运动障碍 性疾病、骨骼肌肉疾病、中枢神经系统感染性疾病以及其他神经 系统疾病及后遗症,不合格。

轻型流行性乙型脑炎已治愈,无后遗症,合格。

第二十五条  晕血,不合格。

第二十六条  影响正常表达的口吃,不合格。

耳鼻喉科项目

第二十七条  双耳中至少一耳耳语听力达到5米,且另一耳 耳语听力3米以上,合格。

第二十八条  眩晕症,不合格。

第二十九条  明显的耳廓畸形及先天性耳畸形,外耳道锁,全耳廓再造术后,外耳道胆脂瘤,耳扉及外耳道良、恶性肿 瘤,不合格。

下列情况合格:

(一)耳前瘿管,耳廓及外耳道良性肿瘤,经专科明确无需手术或者治疗,无不适症状,不影响功能和穿戴;

(二)耳前痿管,外耳道胆脂瘤,耳廓及外耳道良性肿瘤,术后痊愈,无并发症,无后遗症。

第三十条  鼓膜穿孔,慢性分泌性中耳炎,化脓性中耳炎,耳源性颅内、外并发症,中耳炎后遗症,中耳胆脂瘤,耳硬化  症,中、内耳术后,中耳肿瘤,以及其他难以治愈的耳病,不合格。鼓膜穿孔修补术后半年以上,鼓膜修复良好,合格。 

第三十一条 嗅觉丧失,不合格。

第三十二条 严重的鼻畸形,鼻中隔穿孔,鼻骨骨折及骨折 史,中、重度变应性鼻炎,急性鼻实炎,严重的慢性鼻实炎,鼻息肉,鼻部肿瘤以及其他严重影响鼻功能的慢性鼻病,不合格。

下列情况合格:

(一)鼻中隔穿孔术后半年以上,鼻中隔修补完整,无复发,

无后遗症;

(二)单纯性鼻骨(粉碎性骨折除外)骨折,术后3个月以

上,复位良好,无并发症,无后遗症;

(三)鼻骨无错位性骨折3个月以上,无并发症,无后遗症。

第三十三条  Ⅲ度肿大的慢性扁桃体炎,阻塞性睡眠呼吸暂 停低通气综合征,慢性喉炎,鼻咽血管纤维瘤,咽部恶性肿瘤, 喉肿瘤,以及影响吞咽、发音、呼吸功能或者其他难以治愈的

咽、喉疾病,不合格。

下列情况合格:

(  )慢性喉炎治愈后(无声音嘶哑,声带无充血、肥厚、

萎缩,声带闭合良好);

(二)鼻咽血管纤维瘤术后半年以上,无并发症,无后遗症。

眼科项目

第三十四条 双眼中任何一眼裸眼视力小于4.5,不合格。

下列情况合格:

(一)双眼中任何一眼裸眼视力小于4.8时,需进行矫正视力 检查,双眼中任何一眼矫正视力均4.8 以上且矫正度数均在600度以下;

(二)双眼中任何一眼行激光手术(有晶体眼人工晶体植入 术除外),术后时间在半年以上,手术眼裸眼视力4.8以上,无并发症,且眼底检查正常,

(三)双眼中任何一眼行激光手术(有晶体眼人工晶体植入


7. 单纯的 QRS 电轴偏移在-30度至+120 度;

8. 单纯逆钟向或者顺钟向转位;

9. 左心室高电压(不伴 ST 段压低、T 波低平、双相);

10.R 波显示, RV1V2高,但肢体导联 QRS  波电压无变 化, QRS  电轴无明显右偏,右胸导联无 ST-T 改变;

11. 室上嵴型QRS (V1 rsr'型,r>r',I、V5 导联无

s 波或者s 波在正常范围内);

12. 不完全性右束支阻滞,经心脏彩超检查排除心脏器质性 病变;

13. 每分钟5次以下的偶发早搏,若为室性偶发早搏需起卧 活动20次后复查,复查后早搏消失[早搏呈二、三联律除外,室 性早搏呈 Ron-T  多源性(含双源性)、多形性(含双形性)、特 宽型 (QRS时间在0.16 s 以上)、特矮型 (QRS  振幅小于1.0 mV)  除外];

14. 心室早复极,心率较慢时以R 波为主导联J 点抬高,ST 段呈凹面向上型抬高小于0.1 mV;

15. 以 R 波为主导联 ST  段呈水平型压低0.05 mV  (aVL  导联可压低0.1 mV), 或者呈上斜型压低小于0.1 mV;

16.T  波显示,Ⅱ导联直立,电压大1/10R 波 ,aVF导联 低平,Ⅲ导联倒置;在V1 V2 导联大于V5 V6(TV5 V6 大 1/10R );

17.U 波明显,但未高于T波。

第五十二条 腹部超声检查发现恶性征象,肝肾弥漫性实质 损害,肾孟积水,单肾,结石,胰腺病变,内脏反位,以及其他 病变和异常的,不合格。

下列情况合格:

(一)轻、中度脂肪肝;

(二)胆囊息肉样病变,数量5个以下,且最大一个息肉的最 大径小于0.6 cm;

  (三)肝脾肾囊肿和血管瘤单脏器数量3个以下,且最大一个 长径3 cm 以下;

(四)肝、脾内钙化灶,肝内串珠样钙化灶性质稳定;

(五)肾实质钙化灶数量3个以下,且最大一个长径1.0 cm 以下;

(六)肾错构瘤数量2个以下,且最大一个长径1.0 cm 以下;

(七)肾盂宽1.5 cm 以下,输尿管不增宽;

(八)副脾;

(九)脾脏长径12.0 cm 以下,厚度4.0 cm以下;脾脏长径 虽大于12.0 cm或者厚径大于4.0 cm,但脾面积测量(0.8×长径 ×厚径)在38 cm² 以下,排除器质性病变。

第五十三条 妇科超声检查发现子宫肌癌,附件区不明性质 包块,盆腔不明性质包块,以及其他病变和异常的,不合格。

下列情况合格:

(一)不伴其他异常的盆腔积液深度2.0 cm以下;

(二)单发附件区、卵巢囊肿最大径3.0cm以下。

第五十四条  器质性心脏病,不合格。

医学检验

第五十五条 血液、尿液常规检查应当结合临床及地区进行综合判定,除血红蛋白可作为贫血诊断指标、血小板计数可作为 血小板疾病诊断参考指标,其他检查项目原则上不作单项淘汰。

粪便常规检查,在地方性寄生虫病和血吸虫病流行地区为必检项目,在体检期间未发现流行趋势的其他地区不做检查。

第五十六条 血清丙氨酸氨基转移酶大于80 U/L,不合格。

男性血清丙氨酸氨基转移酶大于50 U/L  且在80 U/L以下, 女性血清丙氨酸氨基转移酶大于40 U/L 且在80 U/L  以下,除外 病理性因素,合格;有轻、中度脂肪肝,但未发现标准内其他相关疾病,合格。

第五十七条 血清肌酐酶法检测结果:男性59~104μmol/L,

女性45~84 μmol/L,格。

血清肌酐苦味酸速率法检测结果:男性62~115 μmol/L,    

男性53~97 μmol/L,   合格。

血清肌酐检测结果大于正常参考区间上限,通常判定不合 格;复查正常或者轻微增高,临床除外肾脏功能受损,通常判定合格。

第五十八条  血清尿素正常值参考区间:2.9~8.2 mmol/L。

血清尿素检测结果大于正常参考区间上限,肌酐正常时,应当结合临床进行综合判定。

第五十九条  空腹血糖7.0 mmol/L  以上,不合格。

第六十条 糖化血红蛋白6.5%以上,不合格。

第六十一条  乙型肝炎表面抗原阳性,不合格。

第六十二条  人类免疫缺陷病毒抗体阳性,不合格。

第六十三条 梅毒螺旋体特异性抗体和非特异性抗体均为阳性,不合格。

第六十四条 尿液毒品检测阳性,不合格。

第六十五条  尿液妊娠试验阴性,合格。

尿液妊娠试验阳性,但血清绒毛膜促性腺激素 (HCG结果 在参考区间内,合格。

选拔军官补充标准

外科项目

第六十六条 装甲类岗位,身高超过182 cm, 不合格。

航海类中的舰艇、潜艇岗位,身高超过185 cm, 不合格。

防化类岗位,航海类中的潜水岗位,身高低于168 cm 或者超 185 cm,不合格。

航空类中的伞降、机降岗位,特战类岗位,男性身高低于 168cm,女性身高低于165 cm,不合格。

中央警卫团岗位,男性身高低于170 cm,女性身高低于165 cm,

不合格。

   解放军仪仗司礼大队岗位,男性身高低于180 cm, 女性身高 低于173 cm, 不合格。

第六十七条 航海类中的舰艇、潜艇、潜水岗位,单纯性甲状腺肿,不合格。

第六十八条 特战类岗位,航海类中的潜水岗位,航空类中的伞降、机降岗位,轻度胸廓畸形,不合格。

特战类岗位,无合并伤的肋骨单纯性骨折,2根以下, X 线 检查显示骨折线消失,无功能障碍及其后遗症,治愈后不足3年,不合格。

第六十九条  特战类岗位,上肢单纯性骨折,复位良好, X 线片显示骨折线消失,无功能障碍及其后遗症,治愈后不足3年,不合格。

特战类岗位,航空类中的伞降、机降岗位,发生在14岁前的下肢单纯性骨折,复位良好, X 线片显示骨折线消失,无功能障 碍及其后遗症,治愈后不足3年,不合格。

特战类岗位,中度以上扁平足,不合格。

特战类岗位,航海类中的舰艇、潜艇、潜水岗位,航空类中的伞降、机降岗位,大骨节病(含仅指、趾关节稍粗大,无自觉症状,无功能障碍),不合格。

第七十条 特战类岗位,航空类中的伞降、机降岗位,膝内翻股骨内髁间距离大于4cm, 膝外翻胫骨内踝间距离大于4 cm, 并腿下蹲(双足间距不超过肩宽)膝后夹角大于0度,不合格

第七十一条  中央警卫团岗位、解放军仪仗司礼大队岗位,面颈部、着军队制式短袖体能训练服其他裸露部位的文身,非裸 露部位长径大于3 cm 的文身,不合格。

第七十二条  中央警卫团岗位、解放军仪仗司礼大队岗位, 面颈部的白癜风,不合格。

第七十三条 装甲类岗位,航海类中的舰艇、潜艇、潜水岗 位,腋臭,不合格;腋臭术后6个月以上,无并发症,无腋奥,合格。

第七十四条  特战类岗位,航空类中的伞降、机降岗位,下 肢静脉曲张,不合格。

第七十五条 特战类岗位,航空类中的伞降、机降岗位,临 床型Ⅱ度以上(含Ⅱ度)精索静脉曲张,不合格。

航空类中的伞降、机降岗位,精索鞘膜积液,睾九鞘膜积 液,不合格。

第七十六条  特战类岗位,航空类中的伞降、机降岗位,疝 术后者,不合格。

内科项目

第七十七条 特战类岗位,航海类中的舰艇、潜艇、潜水岗 位,航空类岗位,心率101次/分以上,不合格。

航海类中的潜水岗位,心率59次/分以下,不合格。

第七十八条  特战类岗位,航海类中的舰艇、潜艇、潜水岗 位,航空类中的伞降、机降岗位,心脏射频消融术后,心脏杂 音,不合格。

第七十九条 航海类中的潜水岗位,有肺结核病史,结核性 胸膜炎、腹膜炎、脑膜炎病史者,不合格。

耳鼻咽喉科项目

第八十条  通信导航类岗位,特战类岗位,航海类中的舰 艇、潜艇、潜水岗位,航空类岗位,双耳中任一耳耳语听力低于 4米,不合格。

第八十一条  特战类岗位,航海类中的舰艇、潜艇、潜水岗 位,航空类岗位,严重的外耳湿疹或者真菌病,不合格。

第八十二条  特战类岗位,航海类中的潜水岗位,航空类中 的伞降、机降岗位,鼓膜中度以上内陷,鼓膜瘢痕或者钙化斑, 鼓膜穿孔修补术后,不合格。

第八十三条  防化类岗位,后勤类的医疗卫生、油料岗位,特战类岗位,航海类中的舰艇、潜艇、潜水岗位,唤觉迟钝,不

合格。

第八十四条  防化类岗位,后勤类的医疗卫生、油料岗位,特战类岗位,航海类中的舰艇、潜艇、潜水岗位,航空类岗位, 萎缩性鼻炎,慢性鼻窦炎,病理性鼻中隔偏曲,不合格

前款规定岗位,鼻中隔偏曲矫正术后1个月以上,无并发症, 无后遗症,合格。

眼科项目

第八十五条  特战类岗位,航海类中的潜水岗位,航空类中的伞降、机降岗位,任何一眼裸眼视力小于4.8,任何屈光不正 手术史,不合格。

第八十六条  防化类,导弹类,电子对抗类,通信导航类, 测绘类,后勤类中的医疗卫生、油料岗位,特战类岗位,航海类 中的舰艇、潜艇、潜水岗位,航空类中的伞降、机降岗位,经色 觉图谱检查判定为色弱者,不合格。

第八十七条 航海类中的潜水岗位,共同性内、外斜视(含 15度以下),不合格。

口腔科项目

第八十八条    航海类中的潜水岗位,超矜超过5 mm, 开  超过3 mm, 上下颌牙咬合到对颌牙龈的深覆拾、反矜,重度牙 列不齐等严重错拾畸形,不合格。

航海类中的舰艇、潜艇、潜水岗位,重度牙龈炎,中度牙周 炎,不合格。

第八十九条 特战类岗位,航海类中的舰艇、潜艇、潜水岗 位,航空类的伞降、机降岗位,颌面部骨折术后(未取出钛板钛钉等固定物),不合格。

特战类岗位,航海类中的潜水岗位,唇裂术后不合格。

医学影像

第九十条 特战类岗位,航空类中的伞降岗位,航海类中的 潜水岗位,钙化点大于3个,肺纹理增强,胸膜轻度增厚、一侧肋膈角轻度变钝,不合格。

第九十一条 特战类岗位,航海类中的潜水岗位,航空类中 的伞降、机降岗位, 一度房室阻滞,不合格。

第九十二条 特战类岗位,航海类中的潜水岗位,航空类中 的伞降、机降岗位,胆囊息肉样病变,肝、脾、肾囊肿及血管 瘤,肾错构瘤,肾孟宽大于1.0 cm, 不合格。

以上内容仅供参考,具体标准以省军区公布的文件为准。



 



 


 9. 报考公安院校面试体检和体能测评是如何安排的?

面试体检和体能测评工作由省公安厅具体组织实施,一般安排在高考志愿填报前进行。政治考察合格的考生凭准考证、身份证参加面试体检(无须空腹)和体能测评。考生通过“河北省公安厅公安院校公安专业招生信息服务平台”查询本人参加公安院校招生测试的具体时间段,严格根据时间段安排参加测试。如有变化,以当年相关文件为准。


10. 公安院校招生对面试体检和体能测评有哪些要求?

面试体检标准参照公安机关录用人民警察的有关规定执行, 详见《公务员录用体检通用标准(试行)》(人社部发〔2016〕 140号)、《公务员录用体检特殊标准(试行)》(人社部发2010〕 82号)。同时,还应符合下列条件:

(1)身高:男性170厘米及以上,女性160厘米及以上。

(2)体重:男性体重指数(单位:千克/米²)在17.3至27.3之间(含本数,计算时四舍五入保留小数点后一位,下同),女性在17.1 25.7之间。

(3)视力:任何一眼裸眼视力均为4.8及以上,无明显视功 能损害眼病。

(4)色觉:无色盲、色弱。

(5)外观:无少白头,无胸廓畸形,无脊柱侧弯、驼背,膝内翻股骨内髁间距离不超过7厘米,膝外翻胫骨内髁间距离不超7厘米,无足底弓完全消失的扁平足,身体无影响功能的瘢痕面颈部无瘢痕,无下肢静脉曲张,无腋奥,共同性内、外斜视不 超过15度,无唇、腭裂或唇裂术后有明显瘢痕。

(6)其他:无影响面容且难以治愈的皮肤病(如白癜风、银 屑病、血管癌、斑痣等);外观无存在明显疾病特征(如五官畸形、不能自行矫正的斜颈等);无三度单纯性甲状腺肿;无文身; 嗅觉不迟钝;两耳听力正常。

体能测试项目和标准按照《国家学生体质健康标准(2014年 修订)》的有关规定执行,具体如下:

(1)50米跑。可测次数:1次。合格标准:男性≤9.2秒,女性≤10.4秒;

(2)立定跳远。可测次数:3次。合格标准:男性≥2.05米,女性≥1.5米;

(3)1000米跑(男)/800米跑(女)。可测次数1次。合格标准:男性≤4分35秒,女性≤4分36秒;

(4)引体向上(男)/仰卧起坐(女)。可测次数:1次。 合格标准:男性≥9次/分钟,女性≥25次/分钟。

以上4个项目应当全部进行测评。其中,有3个及以上达标的,体能测评结论为合格。

以上内容仅供参考,具体标准以当年文件为准。



11. 司法类高校提前录取专业的面试、体能测试是如何安 ?

        有的司法类高校提前录取专业仍须单独组织面试、体能测试,详见高校招生章程或向有关高校咨询。



12. 司法类高校提前录取专业对身体条件有哪些要求?

司法类高校提前录取专业对考生的身体条件除执行《普通高等学校招生体检工作指导意见》外,还参照教育部高校学生司、司法部法规教育司联合下发的《中国政法大学、西南政法大学、中南财经政法大学、华东政法学院、西北政法学院和中央司法警官学院提前批录取专业招生办法》 (教学〔2003〕16号),具体要求:

五官端正、体形匀称,无各种残疾,心理健康;

双侧裸眼视力均不低于4.7,无色盲、色弱;

男性身高不低于1.70米,体重不低于50千克;女性身高不 低于1.60米,体重不低于45千克;

体能测试成绩合格。

image.png

以上内容仅供参考,具体标准以当年公布为准。


13. 报考军队飞行学员体检是如何安排的?

军队招收飞行学员对身体的要求更加严格,除参加普通体检外,还须参加招飞机构组织的初选、复选、定选三个阶段的选  拔。相关安排及要求按军队招飞机构规定执行。

初选。主要包括宣传动员、个人报名、学校推荐、身体初步检测。

复选。在招飞选拔中心设置的检测站进行,主要包括体格检查、心理选拔、文化摸底测试和政治考核。

定选。在空军招飞局检测基地进行,主要包括体格检查、心理选拔和审批录取。

招飞选拔实行单项淘汰,有一项不合格即终止检测


14. 报考民航飞行学员面试体检是如何安排的?

民航招收飞行学员对身体有特殊要求,除参加普通体检外,还须参加民航招飞单位组织的初检面试、招飞体检鉴定(含飞行职业心理学检测)。具体详询相关民航招飞单位。




15. 普通高校为什么对新生进行身体复查?

  为确保新生质量,进一步了解掌握学生的身体状况,以便有目的、有计划地组织教学,开展体育活动,按照国家教育、卫生 行政部门要求,各普通高校在新生报到后应组


16. 新旧视力对照表

image.png





17. 血压原用计量单位与法定计量单位换算表

image.pngimage.pngimage.png 



 



河北省2024年普通高校

招生工作新视点

 1.深化艺术类专业考试招生改革。按照教育部统一部署,2024年起,普通高校招生艺术类专业考试分为美术与设计类、音乐类(包括音乐表演、音乐教育两类,其中音乐表演类考试包括声乐、器乐两个方向)、舞蹈类、表(导)演类(包括戏剧影视表演、服装表演、戏剧影视导演三个方向)、播音与主持类、书法类、戏曲类等7个科类。其中,美术与设计类、音乐类、舞蹈类、表(导)演类、播音与主持类、书法类等6个科类实行省级统考;戏曲类实行省际联考。

2024年起,进一步提高文化课成绩要求,艺术史论、戏剧影视文学等部分艺术类专业,直接依据考生高考文化课成绩择优录取;美术与设计类、书法类、舞蹈类、表(导)演类、音乐类等5类省级统考的高考文化课成绩占比调整为50%,播音与主持类仍为70%;校考专业高考文化课成绩要求,调整为执行普通类专业批次录取控制分数线(有关破格录取的,按高校公布的考试招生办法执行)。严格控制校考范围,仅少数高校经教育部批准后,可在省级统考基础上组织校考。

2.改革高校高水平艺术团招生办法。2024年起,高校高水平艺术团不再从高校招生环节选拔,由相关高校从在校生中遴选培养。

3.深化高校高水平运动队考试招生改革。2024年起,进一步完善和规范高校高水平运动队考试招生工作,通过优化招生项目范围,严格报考条件和资格审核,改进考试评价方式,提高文化成绩要求,完善招生录取机制,选拔培养德智体美劳全面发展且具有较高体育竞技水平的学生,为奥运会、世界大学生运动会等重大体育比赛和国家竞技体育后备人才培养体系提供人才支撑。

4.高校选考科目优化调整。教育部印发的《普通高校本科招生专业选考科目要求指引(通用版)》,从2024年高考开始实施。这次高校选考科目优化调整,主要是强化了相关专业对高中学习物理、化学科目的基础性要求。学生可根据自身兴趣爱好及特长,统筹考虑国家和社会需要,结合拟报考高校及专业(类)的选考科目要求,合理确定选考科目。5.优化高职单招考试模式。2024年起,报考高职单招的考生在高考报名所在县(市、区)参加考试;普通高中毕业生文化素质不再使用高中学业水平合格性考试成绩折算替代,所有考生均须参加考试;高职单招报考缴费、打印准考证、查询成绩、填报志愿、查询录取结果等均须登录河北省教育考试院官方网站进行;志愿填报数量由5所院校增加至10所院校。

重要日程备忘

image.png


image.png


 


1. 普通高校招生文化考试时间如何安排? 

普通高校招生全国统一考试和普通高中学业水平选择性考试 6月7日至9日进行,详见下表:

 image.png



2. 普通高校招生文化课考试科目是如何设置的?

河北省2024年普通高校招生考试包括全国统一考试和普通高中学业水平选择性考试,满分750分。

统一高考科目为语文、数学、外语3门,使用原始成绩计入考生高考文化总成绩,每门满分150分。其中,外语分英语、俄语、日语、法语、德语和西班牙语等6个语种,考生任选其中一个语种参加考试。所有语种均进行听力考试。

选择性考试科目为思想政治、历史、地理、物理、化学、生6门,考生须从历史、物理2门首选科目中选择1门,再从思想政治、地理、化学、生物4门再选科目中选择2门参加考试。其中,历史、物理使用原始成绩计入考生高考文化总成绩,每门满分100分;思想政治、地理、化学、生物按等级赋分后的成绩计入考生高考文化总成绩,每门满分100分。普通高中在校学生应在相应科目高中学业水平合格性考试合格的基础上报考选择性考试科目。

统一高考科目试题由教育部命制,选择性考试科目试题由我省自主命制。




3. 思想政治、地理、化学、生物4门再选科目考试成绩是如何进行等级赋分的?

等级赋分是按统一规则,由原始成绩进行等级划定后,再由 等级转换而来的分数。转换后赋分成绩满分为100分,赋分起点 30分。先将每门再选科目考生的原始成绩从高到低划分到 A、BCDE 5个等级,各等级人数所占比例分别约为15%、35%、35%、13%和2%。再将A 至E等级内的考生原始成绩,依照等比例转换原则,分别对应转换到100~86、85~71、70~56、55~41和40~30五个分数区间,得到考的赋分成绩。

高考成绩公布时,考生查询到的再选科目成绩是等级赋分后的成绩,不需要考生自行转换。


4. 中等职业学校对口升学考试科目及考试时间如何安排?

中等职业学校对口升学考试分文化考试和专业考试两部分,满分750分。

对口文化考试科目为语文、数学、英语(不进行听力考试),每科试卷满分均为120分,与全国统一高考同期进行,详见下表:

 

image.png

对口专业考试(含专业理论考试和专业技能考试)按专业类别采取全省统一命题、统一考试、统一评定成绩的办法,由有关高校承办,安排在2月下旬至3月中下旬进行,满分390分。旅游类、财经类、农林类、畜牧兽医类、机械类、电子电工类、建筑类、计算机类、医学类等9类,专业理论满分240分,专业技能满分150分。学前教育类只考教育理论,采取笔试形式,满分390分。




              5. 考点、考场是如何安排的?

考点一般设在县级以上(含县级)人民政府所在地,按照国家教育考试标准化考点规范要求建设的中学等办学机构。考场内考生单人、单桌、单行,每场不超过30人。为加强考场管理,由省以考区为单位按一定规则用计算机随机编排考场。


6. 普通高考外语听力考试如何进行?

普通高考英语听力考试采用由考点统一使用有线广播系统或考场播放器进行放音的方式,其他语种采用考场播放器放音的方式。外语科目考试,考生须于考前30分钟进入考场,考前15分钟禁止迟到考生进入考。考前10分钟,开始试听,试听结束时将有“试音到此结束”的提示音。考试正式开始后先进行外语听力考试。听力考试期间,考生须保持安静。听力部分结束后,会播出“听力部分到此结束”的提示音。


7. 残疾考生如何申请高考合理便利?

申请高考合理便利的残疾考生,报名时须如实填写本人残疾人证号,提供本人第二代中华人民共和国残疾人证(以下简称残疾证)、身份证,如实填写“残疾人报考河北省普通高等学校招生全国统一考试合理便利申请表”并交当地招生考试机构。招生 考试机构组织专家组进行现场综合评估:

(一)专家组成员由本级招生考试机构、残联、卫健委等部门相关人员及班主任组成。根据残疾考生申请的类型,专家组成员须包含相应医疗学科的副高级职称以上专家。

(二)专家组对考生身份及残疾情况进行现场确认,结合残疾考生的残疾程度、日常学习情况、提出的合理便利申请以及考试组织条件等因素进行综合评估,并将评估意见填写在“残疾人报考河北省普通高等学校招生全国统一考试合理便利申请表”中,由专家组全体成员签字确认。

市招生考试机构根据专家组意见提出审核意见,经省招生考 试机构复核同意后,下发《河北省普通高等学校招生全国统一考试残疾考生申请合理便利结果告知书》(以下简称《告知书》)。

当地招生考试机构应及时将《告知书》送达考生或法定监护人确认、签收,如考生或法定监护人拒收则视为自动放弃,则由送达人写明情况并签字。

残疾考生对《告知书》内容如有异议,可从收到《告知书》之日起5个工作日内,向省教育行政部门提出书面复核申请。合理便利审核结果仅作为普通高校招生统一高考和选择性考试提供相应合理便利的依据。如考生需在艺术、体育专业统考以及对口  专业考试、高职单招考试中申请合理便利,须在考试前10个工作日向考试承办院校或当地招生考试机构(美术与设计类专业统考  考生须在考试前5日向市级招生考试机构)提出申请,按要求提供《告知书》和残疾人证等有关证明材料,院校或当地招生考试 机构在保证正常组考的前提下,为考生提供力所能及的合理便利,具体便利措施以院校或当地招生考试机构答复为准。




8. 艺术类专业统考是如何安排的?

2024年起,河北省普通高校招生艺术类专业考试分为美术与 设计类、音乐类(包括音乐表演、音乐教育两类,其中音乐表演 类考试包括器乐、声乐两个方向)、舞蹈类、表(导)演类(包 括戏剧影视表演、服装表演、戏剧影视导演三个方向,以下简称 戏剧影视表演类、服装表演类、戏剧影视导演类)、播音与主持 类、书法类、戏曲类等7个科类。其中,美术与设计类、音乐类、 舞蹈类、表(导)演类、播音与主持类、书法类等6个科类实行 省级统考;戏曲类按教育部要求执行,实行省际联考。报考艺术 类专业考试的考生,须在7个科类中选择1个科类报考,不可跨 类兼考、兼报。

音乐类考生须在音乐表演类(声乐)或音乐表演类(器乐) 中选择其一进行填报。报考音乐表演类(声乐)的考生,可选择“兼考音乐教育类(声乐主项)”,同时获得音乐教育类(声乐主)报考资格;报考音乐表演类(器乐)的考生,可选择“兼考音乐教育类(器乐主项)”,同时获得音乐教育类(器乐主项)报考资格。报考表(导)演类的考生须在戏剧影视表演类、服装表演类和戏剧影视导演类中选择1类报名参加考试,不可兼考、

兼报。考生须按高考报名时选择的艺术类别参加相应的艺术统考。

美术与设计类专业统考由河北师范大学具体组织实施,考点设在各市;音乐类专业统考由河北经贸大学具体组织实施;舞蹈 类专业统考由衡水学院具体组织实施;戏剧影视表演类专业统 考、戏剧影视导演类专业统考由邯郸学院具体组织实施;服装表演类专业统考由河北科技大学具体组织实施;播音与主持类专业统考由河北地质大学具体组织实施;书法类专业统考由河北大学 具体组织实施。

美术与设计类、书法类、音乐类(乐理、听写科目)、戏剧 影视导演类(叙事性作品写作科目)专业统考采取笔试形式;服  装表演类专业统考采取“现场面试评分”形式,评委根据考生现场表现进行评分;音乐类(声乐、器乐、视唱科目)、舞蹈类、戏剧影视表演类、戏剧影视导演类(文学作品朗诵、命题即兴表演科目)、播音与主持类专业统考采取“考评分离”形式,即在条件统一的考试环境中对考生考试音视频先进行采集,以音视频作为考生答卷,考试结束后再组织评委专家通过集中观看考生音 视频进行评分的考试组织模式。

省级统考各科类的科目和分值、内容和形式、考试要求、考 查范围等要求见《河北省普通高等学校艺术类专业省级统考考试说明》《河北省普通高等学校艺术类专业省级统考考试说明解读》(考生可登录河北省教育考试院官网 http://www.hebeea.edu.cn/

音乐类考生须在音乐表演类(声乐)或音乐表演类(器乐) 中选择其一进行填报。报考音乐表演类(声乐)的考生,可选择“兼考音乐教育类(声乐主项)”,同时获得音乐教育类(声乐主)报考资格;报考音乐表演类(器乐)的考生,可选择“兼考音乐教育类(器乐主项)”,同时获得音乐教育类(器乐主项)报考资格。报考表(导)演类的考生须在戏剧影视表演类、服装表演类和戏剧影视导演类中选择1类报名参加考试,不可兼考、

兼报。考生须按高考报名时选择的艺术类别参加相应的艺术统考。

美术与设计类专业统考由河北师范大学具体组织实施,考点设在各市;音乐类专业统考由河北经贸大学具体组织实施;舞蹈 类专业统考由衡水学院具体组织实施;戏剧影视表演类专业统 考、戏剧影视导演类专业统考由邯郸学院具体组织实施;服装表 演类专业统考由河北科技大学具体组织实施;播音与主持类专业 统考由河北地质大学具体组织实施;书法类专业统考由河北大学 具体组织实施。

美术与设计类、书法类、音乐类(乐理、听写科目)、戏剧影视导演类(叙事性作品写作科目)专业统考采取笔试形式;服装表演类专业统考采取“现场面试评分”形式,评委根据考生现场表现进行评分;音乐类(声乐、器乐、视唱科目)、舞蹈类、戏剧影视表演类、戏剧影视导演类(文学作品朗诵、命题即兴表演科目)、播音与主持类专业统考采取“考评分离”形式,即在 条件统一的考试环境中对考生考试音视频先进行采集,以音视频作为考生答卷,考试结束后再组织评委专家通过集中观看考生音视频进行评分的考试组织模式。

省级统考各科类的科目和分值、内容和形式、考试要求、考查范围等要求见《河北省普通高等学校艺术类专业省级统考考试说明》《河北省普通高等学校艺术类专业省级统考考试说明解读》(考生可登录河北省教育考试院官网 http://www.hebeea.edu.cn/学确定该专业与省级其他统考科类的对应关系。

(3)考试安排。

①缴费办法。

2023年11月22日至11月25日14时,美术与设计类专业统 考考生登录河北师范大学招生信息网(网址:http://zsjyc.hebtu. edu.cn/zsw/)“专业测试”栏目查询缴费办法并缴纳专业考试 费。逾期未缴费考生视为自愿放弃专业考试,不能参加本年度美术与设计类专业统考。缴费后未在规定时间参加专业测试,不予退费。

②下载准考证。

2023年11月26日14时至12月2日,已缴费考生登录河北 师范大学招生信息网(网址:http://zsjyc.hebtu.edu.cn/zsw/)

“专业测试”栏目下载打印准考证。

③考试时间、地点。

 image.png


考试地点:高考报名所在地的市,具体地点见准考证。

考生持准考证和身份证原件等材料,按准考证规定的时间、地点及要求参加专业考试。

④河北师范大学咨询电话:0311-80786666。

(二)音乐类专业统考

(1)音乐类专业统考分音乐表演、音乐教育两类,其中音乐 表演类考试包括器乐、声乐两个方向,总分为300分。音乐表演 类考试包括乐理(15分)、听写(30分)、视唱(15分)、器乐/ 声乐(240分)四个科目。音乐教育类考试包括乐理(15分)、听写(30分)、视唱(15分)、主项(165分)、副项(75分)五个科目,其中主项选择声乐的考生,副项须选择器乐;主项选择器乐的考生,副项须选择声乐。兼考音乐表演类和音乐教育类的 考生,相同科目只考一次,成绩互认。如考生兼考音乐教育类,但未参加相应的副项考试,其音乐教育类考试总成绩(及综合成

)为零。

(2)考生报考以下对音乐专项技能有专业考核要求且符合教育部艺术类专业设置规定的专业,均应参加相应的音乐类专业统考。

音乐表演类本科专业:音乐表演、音乐学、流行音乐等专业。

音乐表演类专科专业:音乐表演、现代流行音乐、音乐制 作、钢琴伴奏、钢琴调律、音乐传播等专业。

音乐教育类本科专业:音乐学、作曲与作曲技术理论、音乐 治疗、音乐教育、录音艺术等专业。

音乐教育类专科专业:音乐制作、钢琴伴奏、钢琴调律、音 乐传播、作曲技术、音像技术、录音技术与艺术、音乐教育等专业。

(3)考试安排。

①缴费办法。

2023年12月11日至12月17日,音乐类专业统考考生凭考 生号和身份证号登录河北经贸大学考试报名缴费(网址: https://hbyytk.hueb.edu.cn:8083) 或通过河北经贸大学财务信  息网(网址: https://cw.hueb.edu.cn) 登录考试报名缴费平台, 核对显示的“考生号、姓名”无误后,按“二选一”要求填报器/声乐测试信息(根据系统提示填报器乐/声乐两首测试曲目及相关信息,待测试日候考时随机抽取其中一首用于测试,中西打击乐专业考生无须进行曲目抽取,测试时直接演奏所填报两首不同打击乐器的作品,其中至少1首为音高类打击乐器作品)并在网上缴纳专业考试费。逾期未缴费考生视为自愿放弃专业考试

不能参加本年度音乐类专业统考。缴费后未在规定时间参加专业测试,不予退费。

②下载准考证。

2023年12月25日至12月29日,已缴费考生登录河北经贸大学考试报名缴费平台(网址: https://hbyytk.hueb.edu.cn:

8083)下载打印准考证。

③考试时间、地点。

乐理、听写考试时间、地点

音乐表演类(声乐)〔及兼考音乐教育类(声乐主项)〕考生考试时间:2023年12月30日13:30  14:00 测试乐理,14:00—14:30测试听写。

音乐表演类(器乐) 〔及兼考音乐教育类(器乐主项)〕考生考试时间:2023年12月30日 16:30—17:00 测试乐理,17:00—17:30测试听写。

考试地点:河北经贸大学(石家庄市学府路47号),具体测试地点见准考证。

器乐、声乐、视唱考试时间、地点

考试时间:考生自2023年12月31日起开始测试,具体测试时间见准考证。

考试地点:河北经贸大学(石家庄市学府路47号),具体测试地点见准考证。

考生持准考证和身份证原件等材料,按准考证规定的时间、地点及要求参加音乐类专业统考。对拒不按“二选一”规则参加 测试的考生(如测试时所报器乐/声乐二首测试曲目经专家鉴定 为同一曲目;或拒不按候考时随机抽取的曲目参加测试的),其器乐/声乐科目测试成绩按0分处理。

④河北经贸大学咨询电话:0311-87655611。

(三)舞蹈类专业统考

(1)舞蹈类专业统考总分为300分,包括舞蹈基本功(总分120分,其中身体条件测试50分、技术技巧测试70分),舞蹈表测试,不予退费。

②下载准考证。

2023年12月25日至12月29日,已缴费考生登录河北经贸大学考试报名缴费平台(网址:https://hbyytk.hueb.edu.cn:8083)下载打印准考证。

③考试时间、地点。

乐理、听写考试时间、地点

音乐表演类(声乐)〔及兼考音乐教育类(声乐主项)〕考生考试时间:2023年12月30日13:30 14:00 测试乐理,14:00—14:30测试听写。

音乐表演类(器乐)〔及兼考音乐教育类(器乐主项)〕考生考试时间:2023年12月30日 16:30—17:00 测试乐理,17:00—17:30测试听写。

考试地点:河北经贸大学(石家庄市学府路47号),具体测试地点见准考证。

器乐、声乐、视唱考试时间、地点

考试时间:考生自2023年12月31日起开始测试,具体测试时间见准考证。

考试地点:河北经贸大学(石家庄市学府路47号),具体测试地点见准考证。

考生持准考证和身份证原件等材料,按准考证规定的时间、地点及要求参加音乐类专业统考。对拒不按“二选一”规则参加 测试的考生(如测试时所报器乐/声乐二首测试曲目经专家鉴定为同一曲目;或拒不按候考时随机抽取的曲目参加测试的),其 器乐/声乐科目测试成绩按0分处理。

④河北经贸大学咨询电话:0311-87655611。

(三)舞蹈类专业统考

(1)舞蹈类专业统考总分为300分,包括舞蹈基本功(总分 120分,其中身体条件测试50分、技术技巧测试70分),舞蹈表考生持准考证和身份证原件等材料,按准考证规定的时间、地点及要求参加舞蹈类专业统考。

④衡水学院咨询电话:0318-6016287(咨询考务问题),0318-6016569(咨询缴费问题)。

(四)表(导)演类专业统考

(1)戏剧影视表演类专业统考

①戏剧影视表演类专业统考总分为300分,包括文学作品朗(100分)、自选曲目演唱(50分)、形体技能展现(50分)、命题即兴表演(100分)四个科目。

②考生报考以下对戏剧影视表演专项技能有专业考核要求且符合教育部艺术类专业设置规定的专业,均应参加戏剧影视表演类专业统考。

本科专业:表演(戏剧影视表演)、戏剧教育、曲艺、音乐剧等专业。

专科专业:表演艺术、戏剧影视表演、歌舞表演、曲艺表演、音乐剧表演、现代废术设计与表演、民族表演艺术等专业。

根据教育部《普通高等学校艺术类本科考试招生专业目录(试行)》文件精神,曲艺、音乐剧等本科专业,表演艺术、歌舞表演、曲艺表演、音乐剧表演、民族表演艺术等专科专业,相关招生高校可对应戏剧影视表演类统考科类,也可根据人才培养需要,跨科类科学确定该专业与省级其他统考科类的对应关系。

③考试安排。

A. 缴费办法

2023年11月24日至11月30日,戏剧影视表演类专业统考考生登录邯郸学院艺术统考信息网(网址: https://ystk.hdc.

edu.cn),按照提示要求缴纳专业考试费。逾期未缴费考生视为自愿放弃专业考试,不能参加本年度戏剧影视表演类专业统考。缴费后未在规定时间参加专业测试,不予退费。

B. 上传形体技能展现科目伴奏音乐、下载准考证

2023年12月15日至12月21日,已缴费考生登录邯郸学院艺术统考信息网(网址:https://ystk.hdc.edu.cn),按照系统提 示要求上传形体技能展现科目伴奏音乐、下载打印准考证。伴奏

音乐上传后可预览试听、更换,提交确认后,伴奏音乐将锁定不得更换。

C. 考试时间、地点

考试时间:考生自2023年12月31日起开始测试,具体测试时间见准考证。

考试地点:邯郸学院东校区(邯郸市邯山区学院北路530号,滏河南大街与学院北路交叉口东行200米路北)。

考生持准考证和身份证原件等材料,按准考证规定的时间、地点及要求参加戏剧影视表演类专业统考。

D. 邯郸学院咨询电话:0310-7090228。

(2)戏剧影视导演类专业统考

①戏剧影视导演类专业统考总分为300分,包括文学作品朗(50分)、命题即兴表演(50分)、叙事性作品写作(200分) 三个科目。

②考生报考戏剧影视导演专业、广播影视节目制作专业等对戏剧影视导演专项技能有专业考核要求且符合教育部艺术类专业 设置规定的专业,均应参加戏剧影视导演类专业统考。

③考试安排。

A. 缴费办法

2023年11月24日至11月30日,戏剧影视导演类专业统考考生登录邯郸学院艺术统考信息网(网址: https://ystk.hdc.

edu.cn), 按照提示要求缴纳专业考试费。逾期未缴费考生视为自愿放弃专业考试,不能参加本年度戏剧影视导演类专业统考。缴费后未在规定时间参加专业测试,不予退费。

B. 上传形体技能展现科目伴奏音乐、下载准考证

2023年12月15日至12月21日,已缴费考生登录邯郸学院

艺术统考信息网(网址:https://ystk.hdc.edu.cn), 按照系统提示要求上传形体技能展现科目伴奏音乐、下载打印准考证。伴奏 音乐上传后可预览试听、更换,提交确认后,伴奏音乐将锁定不 得更换。

C.  考试时间、地点

叙事性作品写作科目考试时间:2023年1230日9:00 11:30。

文学作品朗诵、命题即兴表演科目考试时间:考生自2023年12月31日起开始测试,具体测试时间见准考证。

考试地点:邯郸学院东校区(邯郸市邯山区学院北路530号,滏河南大街与学院北路交叉口东行200米路北)。

考生持准考证和身份证原件等材料,按准考证规定的时间、地点及要求参加戏剧影视导演类专业统考。

D.  邯郸学院咨询电话:0310-7090228。

(3)服装表演类专业统考

①服装表演类专业统考总分为300分,包括形体形象观测 (150分)、台步展示(120分)、才艺展示(30分)三个科目。

②考生报考表演(服装表演)专业、时尚表演与传播专业等 对服装表演专项技能有专业考核要求且符合教育部艺术类专业设 置规定的专业,均应参加服装表演类专业统考。

③考试安排。

A.  缴费办法

2023年12月13日至12月18日,服装表演类专业统考考生 登录河北科技大学校园统一支付平台

(网址: http://shoufei.hebust.edu.cn/login.aspx) 缴纳专业考试费。逾期未缴费考生视为

自愿放弃专业考试,不能参加本年度服装表演类专业统考。缴费 后未在规定时间参加专业测试,不予退费。

B.  下载准考证

2024年1月15日至1月24日,已缴费考生登录河北科技大学招生办公室网站(网址: http://bkzs.hebust.edu.cn/)“服装 表演类专业统考管理系统”下载打印准考证。

 C. 考试时间、地点

考试时间:考生自2024年1月25日起开始测试,具体测试 时间见准考证。

考试地点:河北科技大学新校区(石家庄市裕华区裕翔街26)。

考生持准考证和身份证原件等材料,按准考证规定的时间、地点及要求参加服装表演类专业统考。

D. 河北科技大学咨询电话:0311-81668135。

(五)播音与主持类专业统考

(1)播音与主持类专业统考总分为300分,包括作品朗读 (100分)、新闻播报(100分)、话题评述(100分)三个科目。

(2)考生报考以下对播音与主持专项技能有专业考核要求且 符合教育部艺术类专业设置规定的专业,均应参加播音与主持类 专业统考。

本科专业:播音与主持艺术、播音与主持等专业。

专科专业:播音与主持专业。

(3)考试安排。

①缴费办法

2023年12月12日至12月22日(每天9时  18时),播音 与主持类专业统考考生登录河北地质大学普通高考招生信息网

(网址: https://zsxxw.hgu.edu.cn/) “播音与主持类专业统考管

理系统”,按照提示要求缴纳专业考试费。逾期未缴费考生视为 自愿放弃专业考试,不能参加本年度播音与主持类专业统考。缴 费后未在规定时间参加专业测试,不予退费。

②下载准考证

2023年12月27日至2024年1月2日,已缴费考生登录河北 地质大学普通高考招生信息网(网址: https://zsxxw.hgu.edu.

/)   “播音与主持类专业统考管理系统”下载打印准考证。 ③考试时间、地点

考试时间:考生自2024年1月3日起开始测试,具体测试时

间见准考证。

考试地点:河北地质大学校本部(石家庄市裕华区建华南大

 街与槐安东路交叉口南行200米路东)。

 考生持准考证和身份证原件等材料,按准考证规定的时间、

 地点及要求参加播音与主持类专业统考。

④河北地质大学咨询电话:0311-87207400(报名缴费及下载 准考证期间)、0311-87208410(考试期间)。

(六)书法类专业统考

(1)书法类专业统考总分为300分,包括书法临(150分,两幅作品各75分)、书法创作(150分,两幅作品各75分)两个 科目。

(2)考生报考书法学专业、书画艺术专业等对书法专项技能有专业考核要求且符合教育部艺术类专业设置规定的专业,均应参加书法类专业统考。

根据教育部《普通高等学校艺术类本科考试招生专业目录(试行)》文件精神,书画艺术等专科专业,相关招生高校可对应书法类统考科类,也可根据人才培养需要,跨科类科学确定该 专业与省级其他统考科类的对应关系。

(3)考试安排。

①缴费办法

2023年12月3日至12月9日,书法类专业统考考生登录河 北大学艺术类专业测试管理系统(网址: http://zhaoban.hbu. cn/ysxk/),     按照提示要求缴纳专业考试费。逾期未缴费考生视为自愿放弃专业考试,不能参加本年度书法类专业统考。缴费后未在规定时间参加专业测试,不予退费。

②下载准考证

2023年12月25日至12月30日,已缴费考生登录河北大学艺术类专业测试管理系统(网址:http://zhaoban.hbu.cn/ysxk/)下载打印准考证,打印准考证前须认真阅读考试须知和注意事项。

③考试时间、地点

考试时间:2023年12月31 9:00—12:00。其中,书法临摹科目考试时间为9:00- 10:30,书法创作科目考试时间为 10:30—12:00。

考试地点:河北大学七一路校区(地址:保定市莲池区七一东路2666号)或五四路校区(地址:保定市莲池区五四东路180),具体考试地点见准考证。

考生持准考证和身份证原件等材料,按准考证规定的时间、地点及要求参加书法类专业统考。

④河北大学咨询电话:0312-5079698。



 









 
















9. 考生如何查询艺术类专业统考成绩?

(1)根据统考工作安排,我省适时公布艺术类专业省统考成绩并确定专业统考合格标准。届时艺术类专业统考考生登录河北省教育考试院官网 (http://www.hebeea.edu.cn/) 查询省统考成绩。

(2)省统考成绩公布后,考生如对本人当次考试成绩有异议,须在规定时间内,携带本人居民身份证、准考证等材料到高考报名点申请成绩复核,填写成绩复核确认表,逾期不再受理。

复核内容包括考生个人相关信息是否正确,考生答卷(含答卷、答题卡、视频音频等,下同)是否与本人信息一致,答卷是否漏评、漏统(登),各科目(小题)得分合成后是否与提供给考生的成绩一致。评分标准和评分细则的宽严问题不在复核范围之内。成绩复核不提供考生查看答卷服务。

成绩复核结束后,考生凭考生号和密码登录河北省教育考试院普通高校招生考试信息管理与服务平台(网址:https://gk.he-beea.edu.cn), 在“信息查询”栏目中查询成绩复核结果。

(3)我省不发放艺术类专业考试报考证和艺术类专业统考合格证。如需要报考证,考生可登录河北省教育考试院普通高校招生考试信息管理与服务平台(网址: https://gk.hebeea.edu.cn)

“信息查询”模块,自行查询打印《2024年河北省普通高等学校招生考生个人信息表》。




10. 戏曲类专业省际联考是如何安排的?

(1)按照教育部统一部署,戏曲类实行省际联考,考试成绩用于戏曲类专业在各省的招生录取。戏曲类本科专业实行省际联考后,招生高校不再组织其他考试。

(2)实行省际联考的高校戏曲类本科专业包括音乐表演(戏曲音乐)、作曲与作曲技术理论(戏曲音乐)、表演(戏曲表演)、戏剧影视导演(戏曲导演)等。每个专业或剧种(方向)在全国 设置一个考点,各省相关考生均可向组考高校申请参加考试。

戏曲类省际联考一般安排在当年12月至次年2月间举行。考 生须按照我省有关规定和组考高校有关要求,参加当年高考报名 和省际联考的专业考试报名,按照组考高校要求参加考试。组考 高校将根据工作安排发布省际联考专业考试须知,考生可登录院校网站查询或咨询院校。

根据戏曲类专业招生院校招收剧种情况,相关专业(剧种)考试考点如下:

中国戏曲学院:戏曲导演、戏曲作曲、戏曲表演(京剧、昆曲、豫剧、蒲剧)、戏曲音乐(京剧、昆曲、豫剧、蒲剧)。浙江音乐学院:戏曲表演(越剧)、戏曲音乐(越剧)。 吉林艺术学院:戏曲表演(吉剧)、戏曲音乐(吉剧)。

辽宁大学:戏曲表演(东北地方戏)。

兰州文理学院:戏曲表演(秦腔、陇剧)、戏曲音乐(秦腔,陇剧)。

内蒙古艺术学院:戏曲表演(二人台表演)。

   (3)戏曲类省际联考,由组考高校负责发布考生专业考试成绩,各专业(剧种)合格人数原则上不得超过专业(剧种)年度 本科计划数的4倍。成绩查询方式详见组考高校通知。

联考成绩公布后,考生如对本人成绩有异议,须在规定时间内,由考生本人向组考高校提出申请,组考高校按程序进行复核 后及时反馈。成绩复核内容包括:考生试卷或考试信息是否与本人信息一致,是否漏评、漏统(登),各科目得分合成后是否与提供给考生的成绩一致,不提供考生查卷(视频)服务


11. 院校艺术类专业校考是如何安排的?

院校组织校考的时间、地点及要求,校考专业成绩评定、发布及复核,校考专业合格标准确定等有关工作,由高校组织实施,考生可登录相关院校网站查阅或直接向院校咨询。


12. 我省对艺术类专业校考的相关规定有哪些?

(1)按照教育部要求,原则上高校不再组织校考,直接使用省级统考成绩。少数专业特色鲜明、人才培养质量较高的艺术院校,对考生艺术天赋、专业技能或基本功有较高要求的高水平艺术类专业,经教育部批准后,可按程序申请在省级统考基础上组织校考。按照教育部规定,具体校考高校和专业,考生可查询相关省级教育行政部门、招生考试机构网站或相关高校网站。

(2)2024年起,所有高校艺术类专业校考工作均在学校所在地组织,不再跨省设置校考考点。

   (3)拟参加艺术类专业校考的考生,均须参加相应的省级艺术类专业统考且合格。

(4)对于可授予艺术学学士学位的艺术教育、服装设计与工程、风景园林、文化产业管理等4个非艺术类本科专业,院校须编制分省分专业计划,安排在普通类专业批次录取。院校若对考生有艺术专业基础要求,须在招生章程中明确告知考生应参加的 专业考试科类及录取要求。我省不统一组织此类专业考试。



3. 普通体育类专业考试有哪些要求?

(1)身体条件建议:参加体育专业考试的考生应充分评估自己身体状况,根据自己身体条件情况参加专业考试。不建议有心脏疾病等隐患的考生参加考试。考生赴考前,应到正规医院进行心电图及脑电图等相关项目检查,并根据检查结果及医生建议决定是否赴考,以防考试时发生意外,危及身体健康。为防止考生在考试过程中发生意外伤害,根据教育部和国家体育总局有关文件精神,建议考生在参加考试前购买人身意外伤害保险(含往返 交通)。对在考试期间因自身原因发生的意外伤害事故,责任由 考生自负。

(2)签订承诺书:按照教育部有关文件精神参加河北省普通体育类专业考试的考生,须签订“同意接受兴奋剂检查和不使用兴奋剂的承诺书”,拒不签订承诺书或拒绝接受兴奋剂检查的考生,视为主动放弃考试资格。

(3)报到要求:考生须按考前公告要求按时完成网上缴费和网上报到手续,逾期未缴费或未报到的考生视为放弃专业测试资格,不安排补缴费或补报到。不接受未办理高考报名手续或没有 居民身份证的考生参加考试。

(4)考试内容:100米跑、立定三级跳远、原地推铅球、800米跑等四项。

(5)考试成绩:每项100分,满分400分。录取时,专业成绩四舍五入保留两位小数。

(6)申诉办法:考生每完成一个项目,现场公布该项目成绩,并由考生本人签字确认。如对成绩或判罚存有异议,考生须当场提出书面申诉申请,过期不再受理申诉。

(7)相关规定:一是测试过程中如因特殊情况(极端天气)中断测试,考点将根据具体情况调整测试时间并公告。调整后的时间一般为正常测试全部结束后的第二天,具体时间见考点招生网,调整测试时间的考生不再缴费。二是凡因犯规、缺考等原因造成测试项目无成绩者不得再次测试该项目。

三是测试开始 后因自身原因无法继续测试的考生不允许再次测试该项目。

四是凡已取得成绩(含单项)的考生,不允许再测试。五是测试时间 一经确定,原则上不予调整。因与其他测试时间冲突或身体等客观原因无法按时测试的考生,需本人提前向考点申请,并出具有关证明,经考点同意后,通过调整测试时间方式解决。

普通体育专业考试的具体安排可通过省教育考试院或承办高校的官网、官微查看,或向有关承办高校进行咨询。




14. 普通体育专业考试是怎样安排测试顺序的?

普通体育专业考试采取循环递进方式安排各市考生测试顺序。具体办法为:每2市为一组,按往年测试顺序将14个市分为7个组,逐年按组循环递进。2024年测试顺序依次为:

 

image.png



15. 河北体育学院少数民族传统体育项目测试是怎样安排的?

河北体育学院少数民族传统体育项目测试采取单独测试的办法进行,具体要求见河北体育学院招生信息网《河北体育学院2024年社会体育指导与管理专业(面向少数民族传统体育项目培)招生简章》。


16.  对口专业考试是如何安排的? 

对口专业考试分专业理论和专业技能考试,专业理论满分240分,专业技能满分150分,专业理论考试与专业技能考试安排在2024年2月下旬至3月中下旬进行。学前教育类专业考试采 取笔试形式,满分390分。

对口专业考试分为建筑类、财经类、医学类、机械类、电子电工类、旅游类、畜牧兽医类、农林类、计算机类、学前教育类 等十个专业类别,各专业类具体考试安排如下:

(一)建筑类、财经类

1. 缴费时间及办法

考生须于2024年1月3日至1月10日通过微信缴费,具体 缴费方式如下:在微信中向下滑动,在“小程序”界面中搜索 “农行微缴费”,左上角位置图标选择“邢台市”,点击顶部搜索 框搜索“河北科技工程职业技术大学”,然后选择“河北科技工 程职业技术大学考试费”,输入缴费号和姓名,即可缴费。缴费 号为个人14位考生号,缴费咨询电话:0319-2272042、2273676。 请考生务必注意:逾期未缴费考生视为自动放弃专业考试,不能 参加本年度对口升学建筑类、财经类专业考试,缴费结束后不再 安排任何方式的补缴费,已缴费考生不予退费。特别提醒:本年 度不受理集体缴费。

2. 考试时间安排

(1)专业理论考试时间

建筑类:2024年3月16日上午。

财经类:2024年3月17日上午。

(2)专业技能考试时间

建筑类:2024年3月16日下午、3月17日至3月18日全天 (如有特殊情况技能考试时间顺延)。

财经类:2024年3月17日下午。

(3)准考证打印时间

所有已缴费考生于2024年3月11日至3月15日登录河北科 技工程职业技术大学招生信息网(网址:http://zs.xpc.edu.cn/)

点击“通知公告中的准考证打印”栏目,打印准考证、考生须知。

3.考试地点

(1)建筑类

所有考生在河北科技工程职业技术大学参加考试。

(2)财经类

所有已缴费考生于2024年3月1日登录河北科技工程职业技 术大学招生信息网(网址:http://zs.xpc.edu.cn/) 查看。

4. 联系电话

建筑类、财经类:0319-2273053、2273675、2271766。 

5. 成绩查询

考生于2024年3月29日至3月31日登录河北科技工程职业技术大学招生信息网(网址: http://zs.xpc.edu.cn/)查询考试

成绩。如考生对成绩有异议,请于2024年4月1日前向河北科技 工程职业技术大学招生办公室提出成绩复核申请。

(二)医学类

1. 缴费时间及办法

(1)考生须于2024年1月3日8:00至1月9日18:00登 “河北省中等职业学校对口升学医学类专业考试网” (网址: https:///dkweb.hebeinu.edu.cn/dkweb/index.html),点击“报 名缴费”按钮进行缴费。

(2)考生须于缴费24小时后再次点击“报名缴费”按钮,查 询本人是否已经缴费成功。已经成功缴费的考生系统会显示“× ×同学您已缴费成功,可以正常参加‘河北省2024年中等职业学 校对口升学专业考试(医学类)’”。未缴费或缴费未成功考生 会显示“您未缴费,请在规定时间内尽快缴费”。

(3)考生须于规定时间内完成缴费并按照上述要求及时查询 缴费结果。请考生务必注意:逾期未缴费考生视为自动放弃专业 考试,不能参加本年度对口升学医学类专业考试,缴费结束后不 再安排任何方式的补缴费,已缴费考生不予退费

注:如缴费过程意外中断,请等待20分钟后再次登录网址重新进行缴费,不要连续操作。遇到缴费问题,考生可在缴费期间每日8:00至18:00拨打缴费咨询电话进行咨询:18931316892。

2. 考试时间安排

(1)专业理论考试时间

2024年3月16日上午。

(2)专业技能考试时间

2024年3月16日下午至3月22日下午(如有特殊情况技能 考试时间顺延)。

(3)准考证打印时间

所有已缴费考生于2024年3月6日后登录“河北省中等职业学校对口升学医学类专业考试网” (网址: https://dkweb.he-

beinu.edu.cn/dkweb/index.html 点击“准考证下载”按钮打印

准考证和《考生参加考试承诺书》。

3. 考试地点

(1)专业理论考试地点:石家庄地区(包括辛集)生源考生 在石家庄市参加考试,其他地区生源考生在张家口市参加考试。

(2)专业技能考试地点:所有地区生源考生均在张家口市参 加考试。

具体考试地点详见准考证。

4.联系电话

0313-4026123。

5. 成绩查询

考生于2024年4月10日后登录“河北省中等职业学校对口 升学医学类专业考试网”(网址: https://dkweb.hebeinu.edu.cn/

dkweb/indexhtml) 点击“成绩查询”按钮查询考试成绩。如考 生对成绩有异议,请于2024年4月10日至4月12日点击网站 “成绩复核”按钮,根据要求提交复核申请。考生可于4月19日 后点击网站“复核成绩查询”按钮自主查询复核结果。

(三)机械类、电子电工类、放游类

1. 缴费时间及办法

考生于2024年1月3日下午14:00至1月7日上午10:00登录河北师范大学招生信息网(网址:http://zsjyc.hebtu.edu.cn/ zsw/)    “专业测试”栏目查询《2024年河北省中等职业学校对口升学机械类、电子电工类、旅游类专业考试缴费通知》(以下简称《通知》),并完成以下流程:

(1)考试注册(建议使用电脑端)

按《通知》要求,考生登录考试注册系统,输入本人信息(考生姓名、身份证号以及考生号)后仔细阅读弹窗内容(考试说明、考生须知以及考试承诺书),并勾选已阅读后,完成考试 注册,同时获取考生缴费编号(缴费编号为缴费主要凭证,请考生务必牢记)。

(2)缴费

第一步,获取缴费编号后扫描或识别页面下方二维码,考生 凭缴费编号和身份证号通过中国建设银行缴费平台,网上缴纳考

试费。

第二步,输入和确认个人信息,并支付费用。

①在对应位置输入缴费编号、身份证号,点击“下一步”;

②点击“未缴费”,核对订单显示的“缴费日期、应缴金额” 等信息;

③点击订单最后的“详情(或V)”,核对备注中“姓名”无误后,再点击“去支付(或缴费)”,即可通过“网上银行支付、账号支付、扫码支付”缴费(三种支付方式任选其一即可), “网上银行支付、账号支付”可以使用考生本人或他人的建行卡 账号;“扫码支付”可以使用建行龙支付或者微信支付;

④支付成功的考生,可以点击“已缴费”查询缴费结果;

⑤特别注意:如果因缴费过程中未完成缴费而关闭页面、再 次登录后出现无“支付”按钮的情况,请在30分钟后再次登录缴费即可。

请考生务必注意:逾期未进行考试注册或者完成注册未缴费 的考生均视为自动放弃专业考试资格,不能参加本年度对口升学 机械类/电子电工类/旅游类专业考试,缴费结束后不再安排任何 方式的补缴费;考生需个人自行缴费并阅读考试须知;已缴费考 生不予退费。

2. 考试时间安排

(1)专业理论考试时间

机械类:2024年3月23日

电子电工类:2024年3月16日

旅游类:2024年3月16日

(2)专业技能考试时间

机械类:2024年3月23日至3月27日

电子电工类:2024年3月16日至3月18日

旅游类:2024年3月16日至3月18日

具体考试时间详见准考证。

(3)准考证打印时间

所有已缴费考生于2024年3月4日至3月15 日登录系统(网址: https://kwgl.hebtu.edu.cn/examManage/qiantai/index.

html)   输入考生信息,打印准考证。

注:考生必须在系统  (https://kwgl.hebtu.edu.cn/exam-Manage/qiantai/index.html)中完成登录流程,在弹窗处勾选已 阅读并缴费后才可以打印准考证,以上步骤缺一不可。

3. 考试地点

河北师范大学(石家庄市南二环东路20号)。

4. 联系电话

(1)机械类、电子电工类:0311-80787908。

(2)旅游类:0311-80786104。

(3)河北师范大学招生办公室:0311-80786666。

5. 考生参加专业技能考试自备工具

(1)机械类专业:着工装、工帽、工作鞋,并自备工量刃具(见附表)。

(2)电子电工类专业:万用表、电烙铁及常用焊接组装工具、电工工具各一套;穿工作服和绝缘鞋。

(3)旅游类专业:着工装(不带学校标志)和平底鞋。

附表:机械类考生自备工量刃具

image.png

 image.png

 

6. 成绩查询

机械类、电子电工类和旅游类考生请于2024年4月7日后登 录系统 (https://kwgl.hebtu.edu.cn/examManage/qiantai/in-dex.html) 查询专业考试成绩,如考生对成绩有异议,请于4月 10日前在系统中提出成绩复核申请。成绩复核联系电话:0311- 80786666(兼传真)。

(四)畜牧兽医类、农林类、计算机类

1. 缴费时间及办法

考生须于2024年1月3日至1月8日通过微信扫描“河北科 技师范学院对口专业考试智能缴费系统”二维码办理缴费手续,缴费咨询电话:0335-8076333。请考生务必注意:逾期未缴费考生视为自动放弃专业考试,不能参加本年度对口升学畜牧兽医/农林类/计算机类专业考试,缴费结束后不再安排任何方式的补缴费,已缴费考生不予退费。特别提醒:学校不受理集体缴费。缴费流程如下:

(1)用微信“扫一扫”功能,扫描“河北科技师范学院对口

专业考试智能缴费系统”二维码,进入缴费登录界面,输入14位 考生号,点击“确定”;

(2)进入个人缴费信息界面,核对本人信息无误后,点击 “去缴费”;

(3)进入缴费明细界面,核对本人缴费明细信息,如缴费信  息无误,勾选“我已阅读并同意《缴费系统服务说明》”,点击 “立即缴费”;

(4)输入个人微信支付密码,进行缴费即可;

(5)缴费成功后,可再次扫描二维码,点击“缴费记录”查 询缴费结果。

微信截图_20240605212136.png         “河北科技师范学院对口专业考试 智能缴费系统”二维码


2. 考试时间安排

(1)专业理论考试时间

畜牧兽医类:2024年2月28日上午。

农林类:2024年2月28日下午。

计算机类:2024年3月16日上午。

(2)专业技能考试时间

畜牧兽医类:2024年2月29日上午开始。

农林类:2024年2月29日上午开始。

计算机类:2024年3月16日下午开始。

具体考试时间安排请于2024年1月25日后登录河北科技师 范学院本科招生网“通知公告”栏查询。

(3)准考证打印时间

考生于每类考试开始前5日登录河北科技师范学院本科招生 “通知公告”栏下载打印准考证(具体考试时间和地点详见准考证)。

    3. 考试地点安排

畜牧兽医类、农林类、计算机类具体考试地点安排请于2024 1月25日后登录河北科技师范学院本科招生网“通知公告”栏查询。

4. 成绩查询

考生于2024年4月8日后登录河北科技师范学院本科招生网

“录取查询/对口成绩查询”模块查询考试成绩。如考生对成绩有异议,请于4月10日17:00前向河北科技师范学院招生办公室提 出成绩复核申请。

5. 如遇特殊原因考试时间等发生变化,将通过河北科技师范学院本科招生网站进行通知,请随时关注。

6. 联系电话0335-8076333;河北科技师范学院本科招生网址: http://zhaosheng.hevttc.edu.cn/

(五)学前教育类

1. 缴费时间及办法

考生须于1月3日至1月9日凭考生姓名和身份证号通过 “银联校园安心付缴费平台”网上缴费。请考生务必注意:逾期 未缴费考生视为自动放弃专业考试,不能参加本年度对口升学学 前教育类专业考试,缴费结束后不再安排任何方式的补缴费,已 缴费考生不予退费。特别提醒:本年度不受理集体缴费。缴费方 式如下:

(1)手机安心付客户端缴费流程:

第一步:下载校园安心付手机APP,成功下载后点击进入登 录界面,选择“登录/注册”按钮,点击后会弹出一个填写“手 机号码”的界面。

第二步:按照系统指示完成账号注册和登录密码设置,账号 注册登录成功后即可进入校园安心付主界面。

第三步:成功登录绑定考生信息后,点击主页面右上的“缴 ”选项,进入缴费项目界面,可清楚看到个人信息和缴费明细,确认后点击“去支付”。

第四步:进入核对订单界面,请再次核实确认订单信息,无 误后选择支付方式:银联在线支付,完成缴费。

第五步:缴费成功后,可通过已缴费栏目进行查询,或进入 个人中心,查看本机的交易记录。

如遇问题,请联系校园安心付客服热线:400-028-1024。

〔服务时间:9:00—20:00(平时);9:00—18:00(周六、 周日)〕。

考生可扫描二维码下载校园安心付手机客户端


   微信截图_20240604141200.png 

(2)考生也可关注“石家庄幼儿师范高等专科学校”公众号 进行缴费,方法如下:

第一步:关注“石家庄幼儿师范高等专科学校”公众号。

第二步:点击导航栏中“校园服务”栏目找到“缴费操作指 ”,点开并认真阅读。

第三步:按“缴费操作指引”的说明,选择网页缴费或微信 小程序缴费。

2. 考试时间安排

(1)2024年3月17日上午。

(2)考生于2024年3月12日以后登录石家庄幼儿师范高等 专科学校网站(网址:http://www.sjzysgz.com/) 查看“2024 年河北省学前教育类对口专业考试准考证打印”,打印准考证 (具体考试时间、地点详见准考证)。

3.  联系电话

0311-839325(01。

4.成绩查询

考生于2024年3月28日后登录石家庄幼儿师范高等专科学 校网站(网址:http://www.sjzysgz.com/) 查询考试成绩,如考生对成绩有异议,请于2024年3月29日上午11时前向石家庄幼儿师范高等专科学校提出成绩复核申请。

考生注意事项:

(1)考生凭准考证、身份证等材料,按规定时间和地点参加 考试。参加专业理论考试时,请自带黑色签字笔、2B 铅笔、橡皮、小刀、尺子等文具。其中,财经类考生参加专业技能考试还需自备红色、黑色签字笔,空白名章,印台;建筑类考生还需自 带三角板。

(2)考生要遵守考场规则。所有考场内均装有视频监控设备,考试期间全程录像。考点均配备手机智能安检门,实行“2+1”安检模式。考生不得携带规定以外的物品参加考试,按要求接 受安全检查。考生参加专业技能考试时,不允许穿校服或有所在学校标识的服装,不得佩戴校徽。自觉服从监考员等考试工作人 员管理,不得扰乱考场及其他考试工作地点的秩序。如不遵守考场规则,有违规行为的,严格按照《中华人民共和国教育法》以 及《国家教育考试违规处理办法》(教育部令第33号)执行,涉嫌违法的,将移送司法机关,依照《中华人民共和国刑法》等追究法律责任。

(3)经审核通过的残疾考生,如需在对口专业考试期间享受合理便利,须在考试前10个工作日向承办院校提出申请,同时提供《河北省普通高等学校招生全国统一考试残疾考生申请合理便利结果告知书》和残疾人证等有关证明材料。高校在保证正常组 考的前提下,为考生提供力所能及的合理便利,具体便利措施以高校答复为准。

(4)考生按公布的时间及方式查询专业考试成绩,对考试成 绩有异议的,在承办院校规定时间内提出成绩复核申请。

(5)收费标准按照河北省发展和改革委员会、河北省财政厅核准标准执行。

(6)有关专业考试未尽事宜,考生可直接与承办院校联系。 



 17. 考生如何参加外语口试?

拟报考外语专业和有外语口试要求专业的考生,应参加全省统一组织的外语口试。其中俄语、日语、法语、德语、西班牙语安排在6月17日进行,俄语口试由河北师范大学负责,日语口试由河北大学负责,法语、德语、西班牙语口试由河北外国语学院 负责。英语口试安排在6月17日至18日进行,由各市招生考试机构负责。考生要按规定的时间、地点,携带准考证和居民身份证参加口试,逾期不再补试。口试所需费用由考生自理。

口试前,给考生一定的准备时间阅读口试试题。一般由两名 教师对考生进行口试或采用人机对话方式。首先是通过朗读外语单词、词组、句子以及短文,考查考生语音、语调以及朗读技巧。其次是通过用外语回答问题,来考核考生的思维能力、听力、口头表达和模仿能力。

口试成绩满分为150分,不计考生高考文化总成绩,仅在录取时供对外语口试成绩有要求的专业(类)参考使用。英语口试成绩由市级招生考试机构发布,日语、俄语、法语、德语、西班牙语口试成绩由相关承办院校发布。


18.《考场规则》的主要内容有哪些?

考生应遵守如下考场规则:

(一)自觉服从监考员等考试工作人员管理,不得以任何理由妨碍监考员等考试工作人员履行职资,不得扰乱考场及其他考试工作地点的秩序,不得危害他人的身体健康和生命安全。

(二)凭准考证、身份证等材料,按规定时间和地点参加考 试。应主动接受监考人员和考试工作人员按规定进行的身份验证和对随身物品等进行的必要检查。按要求存放手机、包等非考试品。佩戴口罩的考生,在接受检查时须摘下口罩。因装有心脏起搏器等不宜进行金属探测器检查的考生,应在检查前声明,并 出示县级以上医院出具的证明,以免影响考试。

(三)除2B 铅笔、黑色字迹的钢笔或签字笔、直尺、圆规、三角板、无封套橡皮、透明文具袋、无字垫板等规定的考试用品外,其他任何物品,如手机、手表、智能产(智能手表、手环、眼镜等)、无线耳机等具有通讯功能的电子产品、无线电作弊器材、考试内容相关材料、管制器具等,不得带考试封闭区域。

(四)场后,对号人座,将准考证和身份证放在桌面左上角以便核验。考生要认真核对考试科目、考场号、座位号是否正 确。领到答题卡和试卷后,应在指定位置和规定时间内准确、清楚地填写姓名、考生号、考场号、座位号等内容。凡漏填、错填或书写字迹不清的答卷,影响评卷结果的,责任由考生自负。

遇试卷、答题卡分发错误及试题字迹不清、重印、漏印或缺页等问题,应举手询问,在开考前报告监考人员;开考后,再行报告、更换的,延误的考试时间不予延长;涉及试题内容的疑问,不得向监考人员询问。听力考试期间,不得向监考人员询问并保持安静。

(五)开考信号发出后方可开始答题。

(六)开考前30分钟(第一科为考前40分钟)考生开始入场,开考15分钟后迟到考生不得进入考点(有听力考试的外语科14:45以后迟到考生不准进入考场)参加当次科目考试。考试结束后方可交卷出场。考试时间内非经监考人员允许不得离场。交卷出场后不得再进场续考,也不得在考场附近逗留或交谈。

(七)在与题号相对应的答题区域内答题,写在草稿纸上或非题号对应的答题区域的答案一律无效。不得用规定以外的笔和纸答题,不得在答卷上做任何标记。

(八)在考场内须保持安静,不得吸烟,不得喧哗,不得交头接耳、左顾右盼、打手势、做暗号,不得夹带、旁窥、抄袭或有意让他人抄袭,不得传抄答案或交换试卷、答卷、草稿纸,不得传递文具、物品等,不得将试卷、答卷或草稿纸带出考场。如 体出现异常情况,应立即报告考试工作人员。

(九)考试结束信号发出后,立即停笔并停止答题。在监考人员依序收齐答卷、试卷、草稿纸后,根据监考人员指令依次退出考场。

(十)如不遵守考场规则,不服从考试工作人员管理,有违规行为的,按照《中华人民共和国教育法》《国家教育考试违规 处理办法》确定的程序和规定严肃处理,并记人国家教育考试诚 信档案;涉嫌犯罪的,按照《中华人民共和国刑法》《最高人民法院、最高人民检察院关于办理组织考试作弊等刑事案件适用法律若干问题的解释》等法律规定,移送司法机关追究法律责任。






19.考生如何配合考试工作人员进行安全检查?

为确保高考安全平稳进行,我省使用金属探测器、智能安门对进入考点考试封闭区域、考场的考生进行“2+1”安全检查。即:2次金属探测器和1次智能安检门安检。考生进入考点  

考试封闭区域前,应将手机、手表、包等非考试用品交给带队教(家长)或放在考点规定放置点,然后接受工作人员使用金属探测器进行1次人工安检,通过后经过智能安检门1次安检,然 后进入考场时接受监考人员使用金属探测器再次进行人工安检。 严防考生携带手机、手表、包、智能产品(智能手表、手环、眼镜等)、无线耳机等无线收发装置及其他考试规定以外的物品进考场。

考生参加考试时,尽量不要穿戴带有金属的饰品和衣物,进 考点考试封闭区域前应主动将违规物品(如手机等通讯工具、手表等计时设备)放到指定位置。接受检查时应双臂张开,配合工作人员进行检查。身体正面至背面,从上至下,包括头部、躯干、四肢(含脚部)等,以及考生的透明笔袋,均要进行检查。佩戴眼镜的考生要取下眼镜接受检查。检查脚部时,要抬起脚,离地20厘米左右。如在检查过程中出现报警声,考生应按工作人员要求进行解释、出示相关金属物品。检查完毕确定无违规物品后,考生方可进入考试封闭区域和考场。因身体原因,不能进行检查的考生(如装有心脏起搏器等),应说明情况,并出具县级以上医院的证明。

进入考试封闭区域或考场后因特殊原因离开后返回时,仍要对考生再次检查。对以各种形式阻挠或拒绝接受检查的考试一律不准进入考试封闭区域和考场;对各类扰乱考场秩序、危害考试安全的人员,除依照教育部《国家教育考试违规处理办法》予以处理外,情节严重的,公安机关还要依照《中华人民共和国治安管理处罚法》予以处罚;构成犯罪的,依法追究刑事责任。



20. 考生考前需做好哪些心理准备?

每位考生都希望考出自己的实力、考出好的成绩,相信这也是考生家长的期望。要实现这一目标,除扎实的功底和健康的身体外,充分做好心理准备必不可少。

(1)适量练习,保持活力。好多同学都有这样的感觉,几天不做数学题后再考试,审题迟疑缓慢,入手不顺,运算不畅且易出错。所以必须坚持每天做适量的练习,特别是重点和热点题型,保持思维的灵活和思路畅通。做题时要限时完成,否则容易 养成做题拖拉的毛病,临场时缺少思维激情,控制不好时间,发挥不出应有水平。

(2)合理饮食起居,调节身心。每年高考临近,都有一些考 生精神特别紧张,甚至病倒。我们提醒大家,要注意防止两个极 端的做法:一是彻底放松,破坏了长期形成的生物钟,会适得其反;二是挑灯夜战,加班加点,导致考前极度疲劳,临场提不起 精神。我们建议考生,休息调整是必要的,但是必须微调,特别要把兴奋状态逐步调整到高考考试时间,这当然离不开家长与考生的共同努力。高考前还要注意饮食的科学性和规律性,不能大吃大喝。考生考前的饮食宜清淡又要保证全面的营养,每天要摄人足够的含丰富淀粉的食物,保证用脑的需要。总之,生活要有节奏,张弛有度,保持心态平稳。

(3)充满自信,沉着应战。高考不仅是知识的比赛和智力的 竞争,也是思维品质的考查和心理素质的较量。考试前保持必胜的信念是十分重要的,走进考场要信心百倍,即使遇到难题也不 要慌张,因为大家拿到的考题是一样的。另外,进入考场后适度的紧张是正常的也是必要的,因为适度紧张有利于良好的临场发挥,千万不要因此而引起不必要的慌张。只要大家精心准备、充满自信、沉着应战,就一定能取得满意成绩。



21. 考生考前应注意哪些主要事项?

(1)考生应于6月6日16:00至17:00到考点熟悉环境,但不得进入考试封闭区域。考生应对考点的各类设施做到心中有数,包括考场、候考区、考点办公室、医务室、饮水处、自行车 放置处、厕所、应急疏散通道及场所等所在位置都要清楚。特别 是要熟悉并牢记6月9日再选科目的考试时间、考场号、座位号,避免记错时间,进错考场,坐错座位。

(2)建议距离考点较远的考生要提前在考点附近安排好食宿。

(3)准考证、身份证等进入考场必需的有效证件、材料,必须妥善保存。

(4)准备好考试用具,如0.5毫米的黑色字迹签字笔、直尺、圆规、三角板、2B铅笔、橡皮等。

(5)确认未携带违规物品。


22. 考生遇有特殊情况应如何处理?

(1)因交通原因不能按时到场。考生在考前应到考点实地看一看,由此计算时间和路线。路上所需时间要留有余地,以防迟到。万一发生交通堵塞或交通事故,考生可以出示准考证,求得公安交警的帮助。

(2)丢失或忘带准考证、身份证。每科考试,监考老师都会 按规定查验准考证、身份证。考生万一丢失或忘带,在入场时向考点报告,由监考老师会同考点主考核对准考证存根、进行人脸 识别等,确定无误后可以先参加考试。准考证到当地招生考试机 构办理补发,身份证到公安派出所办理,在下一场考试前,要交 验准考证、身份证。有带队老师的,考生可请带队老师帮助。

(3)考场上突然身体不适,如呕吐、腹泻等。出现这种现象多半是考生精神过于紧张、饮食不洁或休息不好所致。一旦发生这种情况,应及时向监考老师示意,监考老师会及时与考点办公室联系,由考点医务人员进行治疗。若考生中途需离开考场,须工作人员陪同,治疗后可继续应考,但耽误时间不补。

(4)试卷漏印或字迹不清。考生如遇试卷漏印或字迹不清应及时向监考老师声明,申请更换。

遇到突发性事件,首先要保持镇静,切忌过度紧张,手忙脚乱。要进行有效的自我心理调适,通过积极的心理暗示或做深呼 吸等方法消除紧张情绪,尽快将精力集中到所答的题目上来。



23. 答题过程中考生需要注意哪些事项?

开考信号发出后,方可答题。答题时要将答案填涂在答题卡规定的地方。

在考场内须保持安静,不准吸烟,不准喧哗,考试中不准交头接耳、左顾右盼、打手势、做暗号,不准夹带、偷看、抄袭或 有意让他人抄袭,不准传抄答案或交换试卷、答题卡。如遇到试 卷字迹不清、污染等问题可举手询问。开考15分钟后迟到考生不 准再进入考点,其中有听力考试的外语科开考前15分钟后禁止迟到考生进入考场。考试结束前不得离开考场。在整个答题过程中,尽自己的能力答好题,不要分散自己的注意力。

考生应严格遵守考场纪律,不要存有一丝一毫的侥幸心理,以免因小失大,追悔莫及。如有其他考生违规干扰自己答题时,应及时向监考老师反映,以免出现雷同试卷时被处罚。

考试结束时间到,考生应立即停笔,并将自己的试卷按答题卡、试卷、草稿纸的顺序整理好后、两臂下垂将手置于桌下待

收。试卷收齐后,考生应依次走出考场在考点内等候,待试卷全部清点无误后,由考点主考通知门卫开门方可外出。


24. 针对网上评卷,考生答题时要注意哪些问题?

(1)考生要仔细阅读答题卡上的注意事项,并按注意事项上的规定认真执行。

(2)认真填写考生信息(姓名、考生号等),字迹要清晰工整。

(3)在指定位置粘贴条码,条码一律横贴,即条码上的考生号和姓名为从左向右的方向;条码必须贴在指定的红色条码框中。要保持条码的整洁和完整,不要在条码上面或条码周围乱写 乱画。

(4)不要填写缺考标记,缺考考生的缺考标记由监考老师 填写。

(5)注意书写用笔,选择题必须使用正规厂家生产的2B  笔填涂(使用劣质铅笔填涂可能会影响计算机扫描识别),涂写要均匀、饱满、轻重适度,修改时务必要用橡皮擦干净;非选择题最好使用0.5毫米的黑色字迹签字笔书写,以确保字迹清晰。

(6)作图题直接用0.5毫米的黑色字迹签字笔绘制,也可先 用铅笔绘制,确认后再用0.5毫米黑色字迹签字笔描清楚。

(7)严禁在答题卡上的图像定位点(黑方块)周围作任何涂写和标记。

(8)答非选择题时,字迹要工整、清楚,不要写得太细长,字距适当,答题行距不宜过密。严格按照答题要求在相应题号指定的答题区域内第一行开始答题,并在规定区域内书写,切不可超出黑色边框,超出黑色矩形边框的答案无效。

(9)答题时如需对非选择题答案进行修改,可用修改符号 “——”将原书写内容划去,然后紧挨着在其上方或下方写出新的答案,修改内容书写时与正文一样不能超过该题答题区域的黑色矩形边框,否则修改的答案无效。修改选择题答案可使用橡皮擦,但应注意不要造成答题卡破损。禁止使用涂改胶带纸改错或用透明胶带纸粘扯欲修改的内容。

(10)规范作答选考题。按要求选好题目后作答,用2B 铅笔 将所选题目对应的题号右侧方框涂黑。评卷时按所涂题号进行评分,多涂、多答均按所涂的首题进行评分,不涂则按本选考题的首题进行评分。

(11)保持卡面清洁,不要折叠答题卡。




25. 网上评卷有哪些流程?

计算机网上评卷是对传统考试评卷方式的一场革命。评卷时,通过高速扫描仪快速扫描考生的答题卡,选择题部分由计算机对比标准答案自动给分;非选择题部分按照事先划定的范围切割成一个个的图片,存入计算机中,然后由评卷老师在计算机上评阅。

非选择题的评卷工作是分科、分题进行的,按各科试卷的试题多少,分若干组,采取流水作业的办法评阅全卷。在正式评卷前,对参加评卷的教师进行培训,学习研究试题答案及评分标准、评分办法、评卷纪律、注意事项等。在此基础上,由组长组织评卷员对考生的答卷进行试评,通过试评,在评分标准规定的范围内,制定各题的评分细则,形成文字材料,报学科领导小组审查同意后执行,以确保全组正确掌握评分标准。

非选择题实行“三评加终评”的评卷流程。评卷开始后,网 上评卷系统将事先分割好的、隐去考生所有个人信息的考生答题图片随机分发到两位评阅相应题目的评卷员的计算机上,评卷员根据评分标准在计算机上对考生的答题给出分数。当评阅同一考 生同一题目的两位评卷员所给的分数之差小于事先规定的差值限时,计算机自动取两个人的平均分作为该考生该题目的最终得分。如果出现两位评卷员所给的分数之差大于事先规定的差值阈限的情况,计算机自动将该考生该题目的图片发给该题目的题长,由题长进行三评。三评结束后,计算机自动对三位评卷员所给的分数进行两两对比,如果某两位评卷员所给的分数之差小于 事先规定的差值阈限,计算机自动求取平均分;如果都大于规定 的差值阈限,计算机会将该考生该题目图片发给学科评卷领导小 组裁定,由学科组长组织评卷专家给出最终分数。选择题和非选择题评卷完毕后,由计算机汇总生成考生的成绩。


26. 网上评卷有何优点?

一是减少评卷误差。评卷中的误差是考生和家长最为关心的 问题,也是传统评卷中始终难以解决的问题。由于网上评卷系统对每道题都设置一个差值阈限,这样使误差得到了有效的控制。写分、加分、登分的误差减少到最小。在评卷过程中,评卷员直接敲击键盘打分,减少了笔误。另外,考生成绩得分由计算机自动记录、加分、汇总,不会因字迹潦草而在加分、登分时错认数字,使登分、加分失误导致成绩差错的现象得到了有效的控制。

二是方便过程监控。传统的高考评卷凭借复查和抽查答卷来 对评卷过程监控,但这不能实现连续的、全面的动态监控,所以一直没能真正有效地解决监控问题。网上评卷系统可使组长对每 个评卷员进行实时动态监控,对评卷质量进行追踪监控,通过对 各个评卷员工作情况的比较,发现个别评卷员存在的问题,及时作出调整。

三是优化评卷流程。实行网上评卷取消了答卷流通的人工管理。以前在开始评卷之前,各评卷院校必须将答卷取走,评卷完毕后再送到省登分点,现在直接通过专线传输,解决了答卷交接 中的安全问题。评卷结束后,考生的成绩自动合成,免去了许多 烦琐的数据计算和统计。

四是确保评卷公正。评卷员凭本人用户名和密码进入评卷系统进行独立评阅,有效地防止了评卷员间的相互影响,评卷员在 评卷过程中看到的答卷永远都是新的,既不知道这份答卷是初评 还是复评,也不知道别人的评卷结果,这有利于评卷员进行独立判断、评分。而在传统评卷中,老师们直接在答卷上批改,一道题往往由多个老师来评,复评老师在评分时难免会受到初评老师的影响。网上评卷系统避免了这种现象的发生。




27. 考生如何查询高考成绩?

预计6月下旬,考生可登录省教育考试院网站(网址: http:// www.hebeea.edu.cn) 省教育考试院微信公众号 (hbsksy)教育考试院APP客户端或河北招生考试信息服务网(网址:http://www.hebeeb.com) 查询考试成绩。其中,选择性考试再选科目成绩为等级赋分后的成绩



河北省2024年普通高校

招生工作新视点

 1.深化艺术类专业考试招生改革。按照教育部统一部署,2024年起,普通高校招生艺术类专业考试分为美术与设计类、音乐类(包括音乐表演、音乐教育两类,其中音乐表演类考试包括声乐、器乐两个方向)、舞蹈类、表(导)演类(包括戏剧影视表演、服装表演、戏剧影视导演三个方向)、播音与主持类、书法类、戏曲类等7个科类。其中,美术与设计类、音乐类、舞蹈类、表(导)演类、播音与主持类、书法类等6个科类实行省级统考;戏曲类实行省际联考。

2024年起,进一步提高文化课成绩要求,艺术史论、戏剧影视文学等部分艺术类专业,直接依据考生高考文化课成绩择优录取;美术与设计类、书法类、舞蹈类、表(导)演类、音乐类等5类省级统考的高考文化课成绩占比调整为50%,播音与主持类仍为70%;校考专业高考文化课成绩要求,调整为执行普通类专业批次录取控制分数线(有关破格录取的,按高校公布的考试招生办法执行)。严格控制校考范围,仅少数高校经教育部批准后,可在省级统考基础上组织校考。

2.改革高校高水平艺术团招生办法。2024年起,高校高水平艺术团不再从高校招生环节选拔,由相关高校从在校生中遴选培养。

3.深化高校高水平运动队考试招生改革。2024年起,进一步完善和规范高校高水平运动队考试招生工作,通过优化招生项目范围,严格报考条件和资格审核,改进考试评价方式,提高文化成绩要求,完善招生录取机制,选拔培养德智体美劳全面发展且具有较高体育竞技水平的学生,为奥运会、世界大学生运动会等重大体育比赛和国家竞技体育后备人才培养体系提供人才支撑。

4.高校选考科目优化调整。教育部印发的《普通高校本科招生专业选考科目要求指引(通用版)》,从2024年高考开始实施。这次高校选考科目优化调整,主要是强化了相关专业对高中学习物理、化学科目的基础性要求。学生可根据自身兴趣爱好及特长,统筹考虑国家和社会需要,结合拟报考高校及专业(类)的选考科目要求,合理确定选考科目。5.优化高职单招考试模式。2024年起,报考高职单招的考生在高考报名所在县(市、区)参加考试;普通高中毕业生文化素质不再使用高中学业水平合格性考试成绩折算替代,所有考生均须参加考试;高职单招报考缴费、打印准考证、查询成绩、填报志愿、查询录取结果等均须登录河北省教育考试院官方网站进行;志愿填报数量由5所院校增加至10所院校。

重要日程备忘

image.png


image.png


 


1. 考生填报志愿的作用是什么?

考生填报志愿的作用有两个:一是表达自己的升学愿望;二,是作为普通高校录取的依据,无志愿考生不能参加录取。


2. 填报志愿的时间是如何安排的?

本科提前批、本科批及对口本科批的集中填报志愿时间安排6月下旬;专科提前批、专科批及对口专科批的集中填报志愿在本科批填报最后一次征集志愿时进行。各批次(段)集中填报志愿和征集志愿具体填报时间印制在考生准考证背面,并通过省教育考试院官方网站、微信公众号(hbsksy) 等渠道向社会公布。

军队院校填报志愿安排在6月下旬,在本科提前批、本科批及对口本科批的集中填报志愿开始前进行。具体要求详见当年军队院校招生相关文件。


3. 考生填报志愿方式和流程是怎样的?

考生填报志愿采取网上填报方式进行。考生在规定时间,使用计算机(系统不支持手机、平板等移动终端设备)登录省教育考试院网站(网址:http://www.hebeea.edu.cn,或直接访问https://gk.hebeea.edu.cn),通过志愿填报系统进行志愿填报.须使用IE11、Chrome、Edge 浏览器,其他类型及版本不予支持, 以免因浏览器类型及版本使用不当造成无法填报志愿或填报错误等后果。

具体填报流程如下页图所示:


image.png

(1)考生使用用户名和密码登录系统,用户名为本人的考生号或身份证号,密码为高考报名时所设置的密码。

(2)首次登录时,考生须认真阅读《2024年河北省普通高校招生志愿填报承诺书》,并确认承诺。

(3)考生在志愿填报规定时间内,点击“填报”按钮,进入“2024年河北省普通高校招生志愿填报表”页面。

(4)点击所要填报的批次(段),进入该批次(段)志愿填 报页面。依次选择计划性质、科类(填报高校专项计划、高水平运动队的考生还须选择获得的资格类型),然后输人院校代号、专业代号(顺序志愿还须选择是否“服从专业调剂”),逐条完 成填报后点击“保存”按钮。保存成功后返回到“2024年河北省普通高校招生志愿填报表”页面,所报志愿会在“所填志愿”栏中显示出来。

   在填报平行志愿(不含对口)时,需要提醒的是:

①志愿填报系统提供了多种调整志愿顺序的方法:清空、插入、上移、下移、删除、调整、拖动。顺序调整完成后,须点击“保存”按钮,否则调整志愿顺序无效。

②志愿填报页面左侧的“志愿导航”栏可以拖动,方便考生快速定位及志愿保存。

(5)考生在填报过程中如需修改志愿信息,可在“批次入口选择”栏中点击相应批次(段)名称,进入志愿修改页面,在规定时间内进行志愿修改。在当次志愿填报截止时间前,考生可多 次登录系统进行志愿修改和保存,最终以其网上最后一次修改并 保存成功的志愿为准。

(6)考生志愿核查无误后,点击“安全退出”按钮安全退出系统。

(7)每次志愿填报规定时间截止,系统将关闭约3小时(以实际操作时间为准)进行系统维护。维护完成系统重新开放后,考生可登录系统查看自己所填报的志愿信息。



 4. 考生在网上填报志愿时应注意哪些事项? 

(1)志愿填报应由考生本人完成,不要让他人代为填报,以免影响录取。

(2)考生应在规定时间内填报志愿,未在规定时间内填报志愿的,视为自动放弃。考生应尽早填报,避免临近系统关闭时间因网络拥堵、断电等突发情况影响志愿填报,甚至错失填报机会。

(3)考生填报志愿时须“一个批次(段)一保存”。每批次(段)填报志愿后,须点击“保存”按钮进行志愿保存,“保存”后即填报成功,未保存的志愿无效。志愿修改后,如未点击“保”按钮,则修改无效,仍以上一次保存的志愿为准。

(4)考生须按照志愿顺序号依次填报平行志愿。填报志愿 时,所填志愿序号必须连续,中间不能留空,否则系统无法保存。例如未填序号1(第一行)的院校及专业,而跳过去直接填 了序号2(第二行)的院校及专业,志愿填报系统不予支持。

(5)考生在上网填报之前应先在纸上填好《2024年河北省普通高校招生考生志愿填报草表》,尽量不要在网上一边思考一边填报。这样既可以提高网上填报志愿的效率和准确性,同时又避免造成网络拥堵。如果考生登录系统后20分钟内无任何操作,再次进行操作时,系统会进行“考生未登录”提示,或被强制退出系统且未进行保存过的志愿不会保存,考生须重新登录系统进行填报。

(6)考生在填报志愿中遇到系统给出填报有误的提示时,应从批次(段)、报考科类、计划性质、院校代号、专业代号、选科要求、性别要求,器种、唱法或舞种要求和本人情况等方面逐一查找出错原因,一定要在修改信息后再进行确认,否则可能导 致志愿信息无效或不能体现本人最终意愿。

(7)多名考生使用同一台计算机填报志愿时,每位考生填报志愿后一定要点击“安全退出”按钮,并关闭所有显示本人信息的页面,否则,后一位考生所填报的志愿信息可能会覆盖前面考生的志愿信息。同时还要注意,不要同时打开两个或两个以上填报页面,以免发生错误。

(8)考生要高度重视密码的设置与保管,密码设置不要过于 简单,更不要将密码告诉他人,以防被他人篡改志愿,影响录取。如果忘记登录密码可进行密码重置,具体办法请参考第一章“高考各阶段考生密码有哪些重置办法?”。


5. 高校专业(类)对考生选考科目有哪些要求? 

2024年我省普通高考继续实行“3+1+2”模式,除语文、数学、外语3门统考科目外,考生首先在物理、历史科目中选择1门作为首选科目,再从化学、生物、思想政治、地理科目中选2门作为再选科目。高校提出的选考科目要求可分为四类。第一类是对首选科目和再选科目都未提出要求的,即“不提科目要求”,表示首选科目为物理或历史的考生均可报考。

第二类是只对首选科目提出要求的,如“物理(1门科目考生必须选考方可报考)”,表示首选科目为物理、再选科目为任2科的考生方可报考。

第三类是只对再选科目提出要求的,如“化学(1门科目考生必须选考方可报考)”,表示首选科目为物理或历史、再选科目选考化学的考生方可报考;如“化学,生物(2门科目考生均 须选考方可报考)”,表示首选科目为物理或历史、再选科目选 考化学和生物的考生方可报考。

第四类是对首选科目和再选科目均提出选考要求的,如“物理,化学(2门科目考生均须选考方可报考)”,表示首选科目为物理、再选科目选考化学的考生方可报考;如“物理,化学,生(3门科目考生均须选考方可报考)”,表示首选科目为物理、再选科目选考化学和生物的考生方可报考。


6. 如何查询招生院校对选考科目的要求? 

考生可以登录拟报考高校官方网站进行查询,也可参考河北省教育考试院官方网站公布的《2024年拟在河北招生的普通高校招生专业选考科目要求》。2024年在我省实际招生的高校、专业及选考科目要求,以及高校相关专业是否在我省分别安排物理科目组合和历史科目组合招生计划,以招生院校当年在我省公布的招生计划为准。


7.2024年各录取批次及志愿是如何设置的

(1)普通类

普通类录取批次分为本科提前批、本科批、专科提前批、专科批等四个批次。

①本科提前批。分为 A、B、C 三段,分段依次录取。

本科提前批A段。包括有政治考察、面试、体检等特殊要求的国家专项计划、军队、公安、司法等本科专业。实行以学校为单位的顺序志愿模式。设1次集中填报志愿和1次征集志愿,每次可填报1所学校,每所学校设6个专业志愿和1个专业服从调剂选项。

本科提前批B段。包括除本科提前批 A 段以外的其他国家专项计划、公费师范生和免费医学定向生等本科专业。实行以“专(类)+学校”为单位的平行志愿模式,1个“专业(类)+学校”为1个志愿。设1次集中填报志愿和1次征集志愿,每次最多可填报96个志愿

本科提前批C 段。包括高水平运动队、高校专项计划、定向就业招生等特殊类型招生本科专业。实行以学校为单位的顺序志愿模式。只设1次集中填报志愿,不进行志愿征集。

②本科批。包括未列入本科提前批的普通类本科专业、地方专项计划、本科预科班等。实行以“专业(类)+学校”为单位的平行志愿模式,1个“专业(类)+学校”为1个志愿。设1次集中填报志愿和2次征集志愿,每次最多可填报96个志愿。

③专科提前批。包括有面试、体检等特殊要求的公安、司法等专科专业。实行以学校为单位的顺序志愿模式。设1次集中填 报志愿和1次征集志愿,每次可填报1所学校,每所学校设6个专业志愿和1个专业服从调剂选项。

④专科批。包括未列入专科提前批的普通类专科专业。实行“专业(类)+学校”为单位的平行志愿模式,1个“专业(类)+学校”为1个志愿。设1次集中填报志愿和2次征集志 愿,每次最多可填报96个志愿。

(2)艺术类

艺术类招生分本科提前批和专科提前批两个批次。每次志愿填报时(集中填报志愿或征集志愿),同一批次(段)只能选择 一个科类填报,各科类不能同时兼报。兼考音乐表演类和音乐教育类的考生,同一批次(段)可兼报两类专业志愿。考生报考艺术类专业须符合高校招生专业(类)选考科目要求。

①本科提前批。分为A、B两段,每段为一个独立的小批次,分段依次录取。

本科提前批A 段。包括少数经批准开展校考的艺术类本科专业和戏曲类省际联考本科专业。实行以学校为单位的顺序志愿模 式。设1次集中填报志愿,允许填报1所学校,设6个专业志愿 1个专业服从调剂选项,一般不再志愿征集。

本科提前批B段。包括使用河北省艺术统考成绩作为专业成 绩进行录取的艺术类本科专业。实行以“专业(类)+学校”为单位的平行志愿模式,1个“专业(类)+学校”为1个志愿。1次集中填报志愿和1次征集志愿,每次最多可填报70个志愿。

②专科提前批。包括艺术类专科专业,分平行志愿和顺序志 愿两种模式。其中,使用河北省艺术统考成绩作为专业成绩进行录取的,实行以“专业(类)+学校”为单位的平行志愿模式,1个“专业(类)+学校”为1个志愿,设1次集中填报志愿和1 次征集志愿,每次最多可填报70个志愿;使用戏曲类省际联考成  绩作为专业成绩进行录取的,实行以学校为单位的顺序志愿模式,设1次集中填报志愿和1次征集志愿,每次可填报1所学校,每所学校设6个专业志愿和1个专业服从调剂选项。

(3)体育类

体育类招生分本科提前批和专科提前批两个批次。①本科提前批。分为 A B两段,分段依次录取。

本科提前批A段。包括河北体育学院少数民族传统体育项 目。实行以学校为单位的顺序志愿模式。设1次集中填报志愿和1次征集志愿。

本科提前批B段。包括使用河北省普通体育类专业考试成绩 作为专业成绩进行录取的体育类本科专业。实行“专业(类)+学校”为单位的平行志愿模式,1个“专业(类)+学校”为1个志愿。设1次集中填报志愿和1次征集志愿,每次最多可填报

②专科提前批。包括使用河北省普通体育类专业考试成绩作 为专业成绩进行录取的体育类专科专业。实行“专业(类)+ 学校”为单位的平行志愿模式,1个“专业(类)+学校”为1个志愿。设1次集中填报志愿和1次征集志愿,每次最多可填报70个志愿。

(4)对口类

对口类分对口本科批和对口专科批。均设1次集中填报志愿 1次征集志愿,均实行平行志愿投档。每次最多可填报5所学校,每所学校设6个专业志愿和1个专业服从调剂选项。对口本科批随本科提前批 B 段填报志愿、录取,对口专科批随专科提前批填报志愿、录取。







8. 征集计划是怎样产生的? 

征集计划是某批次(段)集中填报志愿录取结束后的院校缺额和新增计划,由三部分组成:一是部分学校专业(类)因生源不足未录取满额,产生缺额;二是部分学校专业(类)因考生填报志愿不当或因身体条件不符合录取要求等原因被退档,形成缺额;三是平行志愿投档后,又有部分学校专业(类)向我省新增计划。未被录取的考生可在规定时间内,按照公布的征集计划填报征集志愿。


9.  考生填报志愿的主要依据和参考资料有哪些?

一是招生计划。2024年我省普通类、体育类按历史科目组合 和物理科目组合分别编制招生计划,艺术类不分历史科目组合和 物理科目组合,统一编制招生计划。内容主要包括学校名称、学校代号、专业名称、层次、选考科目要求、计划数、学制、学费和备注等。考生填报志愿时,各项信息都要以招生计划公布的内容为准。

二是招生章程。主要内容包括高校全称、校址、层次(本科、专科)、办学类型、办学主体、专业教学培养使用的外语语种,身体健康状况要求,进档考生录取规则(如对考生加分成绩

的使用、投档成绩相同考生的处理、进档考生的专业安排办法及 专业调剂录取办法等),学费标准,家庭经济困难学生资助政策 及有关办理程序,颁发学历证书的学校名称、证书种类等信息, 联系电话,网址,以及其他须知等。考生可登录教育部“阳光高 ”信息平台(网址:http://gaokao.chsicom.cn)、各高校的官 方网站查看院校招生章程。如发现有关内容与招生计划不一致,要及时向相关院校咨询核实。

特别提醒:考生初步确定报考高校专业(类)后, 一定要仔 细阅读其招生章程,如高校招生专业(类)对外语语种、身体条件、文化课或专业课成绩等有特殊要求的,会在招生章程中予以 明确。受篇幅和字段限制,志愿填报系统、《2024年河北省普通高等学校招生计划》以及《2024年河北省普通高等学校招生章 程》中的信息是不完备的,有些高校报送招生计划时仅摘录了招 生章程中的部分内容,考生填报志愿前务必登录教育部“阳光高”信息平台(网址: http://gaokao.chsi.com.cn)或高校的官 方网站查阅招生章程。

三是院校网站公布的信息。考生通过高校网站可了解高校基本情况、专业设置、地理位置等方面信息。

四是《普通高等学校招生体检工作指导意见》。其中规定了对患有某些疾病或有生理缺陷者不宜报考的专业。考生应根据体检结果,对照体检工作指导意见和有关高校的招生章程,避开自己不宜报考的专业。

五是各类考生成绩统计表。高考成绩公布后,省教育考试院将在官网、官微公布当年”河北省普通高校招生各类考生成绩统计表”,考生可在填报志愿时参考。

六是其他参考资料。包括《全国普通高校在河北招生录取分数分布统计》《2024年河北省普通高校招生报考指南》《2024年河北省普通高等学校招生艺术类报考指南》《河北省普通高等学校招生专业选报指南》。其中,《全国普通高校在河北招生录取分数分布统计》主要内容为近几年全国普通高校在河北省招生录取 情况,包括分学校、分专业的计划数、录取数、最高分、最低分、平均分、差值及各专业录取分数分布统计;《2024年河北省 普通高校招生报考指南》和《2024年河北省普通高等学校招生艺 术类报考指南》介绍了我省高考录取的相关政策规定、操作程序和注意事项,并收集整理了近年来高校录取的相关数据和资料,《2024年河北省普通高等学校招生专业选报指南》主要介绍了普通本专科各专业的培养目标、主要课程和就业方向,以及分专业录取情况统计、与专业选择相关的心理测试理论、方法和工具等内容。这些资料都可以为考生填报志愿提供参考。






10. 填报平行志愿有哪些建议?

根据平行志愿录取规则和志愿填报的方法步骤,提供以下建议供参考:

第一步,确定选择范围。考生在填报志愿时一般是综合考虑 自己的兴趣爱好、专业特长、成绩位次、身体条件等各方面的因素,结合国家和社会需求、高校层次、专业特色、地理位置等内 容,明确自己立志攻读的专业(类)方向和院校的范围。

第二步,确定初选志愿。认真阅读《2024年河北省普通高等 学校招生计划》《2024年河北省普通高等学校招生章程》,充分了 解拟报考院校的招生政策、录取规则,对考生身体条件、选考科 目、单科成绩要求、专业特色、培养方向、就业情况等要求,此外,还要了解院校办学性质、学制、学费等。然后通过研究分析  相关数据,根据自己的成绩位次,结合意向院校专业(类)往年的招生计划、录取最高分、最低分和平均分对应的位次等情况,与自己的成绩位次相比较,按照“冲一冲、稳一稳、保一保”的策略,分三个梯度初步确定足够数量院校专业(类)作为备选。每个梯度要尽量多选择一些院校专业(类),这个过程一般不会一次完成,需要反复斟酌、筛选增删,确定一定数量初选志愿。 

第三步,确定拟报志愿。在这一步,要再次认真研读《2024年河北省普通高等学校招生计划》《2024年河北省普通高等学校招生章程》或向院校咨询,要特别注意拟报考院校专业(类)的专业备注,尤其是对考生的要求,如选考科目要求、身体条件、外语语种、单科成绩、综合素质评价等,对照拟报考院校专业(类)的要求,逐项检查自己是否符合条件,避免出现因相关科目成绩偏低、身体条件不符合要求等原因被退档。还需关注拟报  考院校专业(类)学费情况(部分专业学费较高,考生要认真查询)。通过逐一分析,认真筛查,最终确定符合数量要求的志愿。平行志愿批次普通类每次最多可填报96个志愿,艺术类、体育类每次最多可填报70个志愿。考生要珍惜机会,尽可能多地选择并填报志愿。

第四步,排好志愿顺序。综合分析拟填报院校专业(类)往年录取情况和自己的成绩位次,把拟报志愿科学合理的排好顺序,原则上要把心仪的志愿填在靠前的位置。志愿之间要有合理 层次梯度,不宜全部填报高分段院校专业(类)志愿,也不必全部填报低分段的。




11. 对平行志愿的认识误区有哪些?

一般来说对平行志愿的认识误区主要有三个:

误区一:可以一档多投或多次投档。虽然在同一批(段)考生一次可以填报多个学校专业(类)志愿,但在实际投档过程中,投档机会最多只有一次,也就是说,即使有多个学校专业 (类)志愿符合投档要求,也只能投档到这些志愿中排在最前面的那一个,而且一旦投档,其余志愿随即失效,不再投档。投档后因某种原因被退档,即使后面还有符合投档条件的志愿,也不会再次投档。

误区二:平行志愿没有先后顺序。对考生个人来说,平行志愿是有先后顺序的。平行志愿的检索顺序就是考生所填报的1、2、3、4、5……志愿顺序,所以考生一定要精心安排志愿顺序,把最心仪的志愿填在靠前的位置。

误区三:实行平行志愿后没有了风险。平行志愿虽然可以降低风险,但不能消除所有风险:一是档案投不出去的风险。如果考生定位不准确,志愿填报过高,几个平行志愿之间没有拉开适 当的位次梯度,就可能导致所填报的志愿都达不到投档要求,考生档案无法投出。二是被退档的风险。一些学校专业(类)对考生的身体条件、外语语种、单科成绩等有要求。考生档案投到某个学校的专业(类),如果不符合该专业(类)的上述要求就会被退档。




12. 什么是志愿填报辅助系统?

志愿填报辅助系统是为了更好地服务考生,在填报正式志愿前供考生提前进行招生计划和往年院校专业(类)录取分数、位次查询,并对心仪院校专业(类)进行收藏及意向志愿预选。保存成功的意向志愿可在志愿填报系统中相应批次、计划性质、科类下进行一键加载。志愿填报辅助系统仅对集中填报志愿提供相关辅助功能,征集志愿时不提供。志愿填报辅助系统免费提供给考生使用。


13. 志愿填报辅助系统操作流程及注意事项有哪些?

考生在规定时间内,通过计算机网络访问省教育考试院网站(网址: http://www.hebeea.edu.cn,或直接访问 https://gk.he-beea.edu.cn)登

录系统。须使用IE11、Chrome 、Edge览器,其他类型及版本不予支持。系统不支持使用手机、平板等移动终端。

(1)系统登录。考生须使用用户名和密码登录系统,用户名 为考生本人的考生号或身份证号,密码为考生高考报名时所设置的密码。志愿填报辅助系统开放期间,同一考生在同一时间段只允许在一个终端登录,请广大考生妥善保管好密码及个人信息,谨防他人盗用。考生登录系统20分钟内无任何操作,再次进行操作时会被强制退出系统,考生需要重新登录。

(2)阅读提示。考生登录成功后,会显示温馨提示页面,请考生认真阅读。阅知后系统会自动显示批次列表页面。

    (3)计划查询。系统会根据考生报考类别及选考科目情况将考生能够填报的批次、计划性质、科类全部列出,考生可以通过点击“进入”按钮,进入到相应批次、计划性质、科类的计划查询页面。

(4)收藏招生计划。在招生计划列表中,考生可以对感兴趣的专业进行收藏。收藏的招生计划将会加入到收藏列表。

(5)添加意向志愿。在收藏列表中,考生可以将心仪专业添加为意向志愿。在意向志愿列表页面,考生可对意向志愿进行顺序调整或删除。保存成功的意向志愿可在志愿填报系统中相应批次、计划性质、科类下进行一键加载。需提醒的是:意向志愿调整完毕后,务必点击“保存”按钮进行保存,否则调整结果无效。

(6)系统退出。退出系统时应点击页面右上方的“退出系”,以确保系统正常退出。


14. 什么是河北省高考志愿填报智能参考系统?

河北省高考志愿填报智能参考系统是河北省教育考试院考试与招生杂志社组织开发的、新高考后帮助考生进行志愿填报的智能参考系统,在“专业(类)+学校”模式下帮助考生合理选择高考志愿,目的是为考生提供服务。

该系统包括院校信息、专业简介、百问百答、专业选择测试 等服务模块,方便考生及家长了解院校、专业信息、高考相关政策、明确报考方向,提高考生志愿填报效率。该系统只需考生输入成绩、选考科目等信息后,就可筛选出该考生能报考的专业(类),考生还可以按照自己对省份、城市、学校、专业等个人偏 好挑选出想报考的高校和专业(类),并可将志愿填报结果导出备查。


15. 志愿填报系统、志愿填报辅助系统和志愿填报智能参考系统有什么区别?

志愿填报辅助系统和志愿填报智能参考系统仅在集中填报志愿时提供辅助服务,不能代替志愿填报系统。

志愿填报系统是考生填报高考志愿的唯一途径,无论是集中填报志愿还是征集志愿,考生都必须在规定时间内登录省教育考试院网站,通过该系统进行志愿填报,未按时填报或在其他系统 填报的,都无法参加高考录取。使用志愿填报辅助系统和志愿填 报智能参考系统遴选的参考志愿必须通过志愿填报系统进行填报,方可生效。

正式填报志愿前,考生可使用志愿填报智能参考系统帮助查 询推荐志愿,根据志愿填报智能参考系统推荐的意向志愿,考生可登录志愿填报辅助系统,提前进行计划查询,并根据个人实际情况对心仪院校专业进行收藏及意向志愿预选,提前熟悉如何填报。正式填报志愿开始后,考生务必在规定时间内登录志愿填报 系统填报志愿。




16. 填报志愿时考生如何对待优惠加分政策?

对具备加分资格的考生,我省按照加分后的分数投档。其中,我省地方性加分项目只适用于我省高校省内招生。

对于同时符合多种加分条件的考生,投档时,报考省外院校的使用国家加分最高一项,报考河北省院校的使用国家加分与省内加分中最高一项。根据教育部有关规定,所有高考加分项目及分值均不得用于高校不安排分省招生计划的艺术类专业、高水平运动队、高校专项计划等招生项目。

录取时是否考虑加分因素,按院校的有关规定执行。由于部分院校不承认或部分承认优惠加分政策,因此,享有优惠加分政策投档的考生,有可能因实际文化成绩较低而被院校退档。请考生在填报志愿前,务必认真阅读有关院校招生章程或向院校咨询。


17. 艺术类专业考生填报志愿应注意哪些问题?

(1)填报使用我省艺术统考成绩作为专业成绩录取的院校专(类)志愿,考生专业统考成绩、文化成绩均需达到我省划定的艺术类专业、相应批次录取控制分数线。

(2)填报使用院校艺术类专业校考成绩录取的院校专业志愿,考生高考文化成绩需达到我省普通类专业批次录取最低控制分数线、省统考成绩合格且达到高校划定的最低成绩要求。

对于在相关专业领域具有突出才能和表现的考生,高校可探索制定高考文化课成绩破格录取办法。破格录取办法须经学校党委常委会审议并报所在地省级教育行政部门备案,提前在学校考试招生办法中向社会公布。破格录取考生名单须经学校招生工作领导小组审议并报生源所在地省级招委核准后在学校招生网站公示。

(3)填报实行省际联考的高校戏曲类专业,考生高考文化成绩需达到我省艺术类专业高考文化录取控制分数线。

(4)河北省普通高等学校艺术类专业招生计划不区分历史科目组合、物理科目组合,统一编制。艺术类计划也有少数专业只招首选科目为历史或只招首选科目为物理的考生,我省根据院校的要求在招生计划中予以公布,不符合相应选考科目要求的考生不能填报和录取。为了便于考生查阅,艺术类专业招生计划在 2024年河北省普通高等学校招生计划(历史科目组合·对口)》2024年河北省普通高等学校招生计划(物理科目组合)》中均有编列,但它们是同一计划,并非历史科目组合和物理科目组合各自单独的招生计划数,个别专业单独注明只招首选科目为历史或只招首选科目为物理的考生的除外。




18. 怎样填报艺术类平行志愿?

    (1)正确理解平行志愿投档原则和录取规则。平行志愿投档原则是各省根据本省实际情况将考生档案投档到学校专业的具体办法,我省艺术类平行志愿按综合成绩投档。录取规则是指考生档案投到学校后,学校对进档考生依据其向社会公布的招生章程(或招生简章)中相关规定进行录取的具体办法。投档原则解决的是我省如何向学校投档的问题,而录取规则解决的是学校对进档考生如何确定录取或退档的问题,在主体和时间顺序上不

一样。

(2)根据综合成绩的位次情况,合理确定拟填报学校专业(类)。我省艺术类各类别平行志愿投档综合成绩构成不同,美术 与设计类、音乐类、舞蹈类、书法类、表(导)演类,综合成绩= 高考文化总成绩(含政策性加分)×0.5+(专业成绩÷专业满 )×750×0.5,结果四舍五入保留3位小数。播音与主持类综合成绩=高考文化总成绩(含政策性加分)×0.7+(专业成绩÷ 专业满分)×750×0.3,结果四舍五人保留3位小数。上述各类艺术专业满分均为300分。

考生应根据自己高考文化总成绩和艺术统考专业成绩准确计出综合成绩,综合考虑院校办学实力、专业实力、往年录取情况,结合本人意愿进行填报。

(3)清楚掌握拟报考学校专业(类)特殊要求和相关信息。部分专业(类)对文化成绩、专业成绩、选考科目、身体条件、乐器种类、声乐唱法、舞蹈种类等有要求。考生务必仔细查阅拟报考院校的招生章程(或招生简章),结合招生计划的有关提示,准确填报,避免填报限报专业后被学校退档。

(4)注意规避平行志愿投档模式的风险。考生应认真研究分析招生院校专业的基本情况、招生计划和相关要求,结合分数、位次及本人意愿确定自己的填报志愿方案。普通类平行志愿风险在艺术类专业平行志愿投档录取过程中也同样存在,因此选择的学校专业(类)一定要准确恰当定位,志愿之间要适当拉开位次梯度。



19. 体育类考生填报志愿有哪些注意事项?

(1)使用河北省普通体育类专业测试成绩作为专业成绩录取的体育类专业全部实行平行志愿投档。在思想政治品德考核合格和身体条件符合要求、专业和文化成绩均达到录取控制分数线的前提下,按照“分数优先,遵循志愿”的原则,结合高校专业(类)要求和调档比例,依据考生综合成绩从高到低的顺序,逐个依次检索考生院校专业志愿进行投档。综合成绩计算公式为:综合成绩=高考文化总成绩(含政策性加分)×0.3+(专业成绩 ÷专业满分)×750×0.7,结果四舍五入保留3位小数。体育类专业满分400分。

(2)报考普通体育类院校专业(类)志愿的考生必须参加河 北省普通体育类专业测试,且专业成绩和文化成绩达到我省划定的相应批次(段)录取控制分数线。特别提醒:考生应注意平行志愿投档原则和录取规则的区别(可参阅“艺术类平行志愿填报注意事项”有关内容)。此外,要清楚了解拟报考院校对文化成 绩、专业成绩、身体条件等要求,避免错填限报专业。




20. 艺术类、体育类考生是否可以填报普通类专业志愿?

艺术类和体育类考生,可以填报普通类专业志愿,但每次填报志愿时(集中填报志愿或征集志愿),在同一批次(段)中只能选择同一个类别填报,不能既填报艺术或体育类志愿,同时又填报普通类志愿。


21. 填报军队院校志愿应注意哪些事项?

军队院校招生计划安排在本科提前批A 段,实行顺序志愿投档,设1次集中填报志愿和1次征集志愿,考生每次可填报1所学校,每所学校设6个专业志愿和1个专业服从调剂选项。

具体要求详见当年军队院校招生相关文件。


22. 填报招飞计划应注意哪些事项?

报考空军、海军、民航招飞计划的考生,须参加相关招飞单位组织的考核、面试、体检且合格。招生计划安排在本科提前批A 段,实行顺序志愿投档。

具体详见当年招飞单位相关要求。


23. 填报公安院校志愿应注意哪些事项?

报考公安院校的考生须为具有河北省户籍的普通高中毕业生,须参加由省公安厅统一组织的考察、面试体检和体能评测且合格。公安院校招生计划安排在本科提前批A 段和专科提前批,实行顺序志愿投档,设1次集中填报志愿和1次征集志愿,每批 (段)每次只能填报1所院校,每所学校设6个专业志愿和1个专业服从调剂选项。

具体要求详见当年公安院校招生相关文件。




24. 如何填报定向就业招生志愿?

定向就业招生计划单列,解放军陆军工程大学定向人防计划 安排在本科提前批A段,实行顺序志愿投档,只能填报1所院校志愿,设6个专业志愿和1个专业服从调剂选项。

其他院校定向计划安排在本科提前批C段,只能填报1所院 校志愿,设6个专业志愿和1个专业服从调剂选项。详见当年招生计划。


25. 高水平运动队招生如何填报志愿?

报考高水平运动队的考生,须经教育部“阳光高考”信息平 台公示无误后方可填报。高水平运动队在本科提前批C段录取,只设1次集中填报志愿,不进行志愿征集。可填报1所学校,设6个专业志愿和1个专业服从调剂选项。


26. 国家专项计划招生如何填报志愿?

国家专项计划志愿安排在本科提前批A B 段。通过国家专项计划资格审核的考生方可填报

本科提前批A段的国家专项计划实行顺序志愿,设集中填报 志愿和征集志愿,每次只能填报1所学校,设6个专业志愿和1个专业服从调剂选项。本科提前批B段的国家专项计划实行平行志愿,设集中填报志愿和征集志愿,每次最多可填报96个志愿,与免费医学定向生、公费师范生(含优师专项)同批一起填报,同批一起录取。


27. 高校专项计划招生如何填报志愿?

高校专项计划列在本科提前批C 段,实行顺序志愿,不进行志愿征集。经有关高校考核合格且在教育部“阳光高考”信息平台公示的考生方可填报。




28. 地方专项计划招生如何填报志愿?

地方专项计划安排在本科批,设集中填报志愿和两次征集志愿,每次最多可填报96个志愿,与其他普通类专业同批一起填报,同批一起录取。通过地方专项计划资格审核的考生方可填报。


29. 如何填报公费师范生(含优师专项)志愿?

教育部直属部分师范院校和我省部分师范院校在我省安排公 费师范生招生,批次安排在本科提前批B 段,设集中填报志愿和征集志愿,每次最多可填报96个志愿,与免费医学定向生、国家专项计划同批一起填报,同批一起录取。普通类的优师专项列在普通类本科提前批 B 段,编列到公费师范生类,单列院校代号,院校名称后单独标注“优师专项”,与公费师范生、免费医学定向生、国家专项计划一起实行平行志愿。除以上普通类公费师范(含优师专项)招生之外,艺术、体育、高校专项计划等招生类型的公费师范生(含优师专项)安排在相应类别和批次。考生被公费师范生(含优师专项)录取后须签订协议,详情请咨询有关院校。


30. 如何填报预科班志愿?

预科班招生包括少数民族本科预科班和边防军人子女预科班。含在普通本科批内,设集中填报志愿和征集志愿,每次最多可填报96个志愿,其他

普通类学校同批一起填报,同批一起录取。

其中少数民族本科预科班只招收少数民族考生,边防军人子女预科班只招收通过边防军人子女资格审查的考生。


31. 志愿填报截止后还可以补报志愿吗?

志愿填报系统将按照规定的志愿填报时间准时开放和关闭,考生须按照有关操作流程,在规定时间内完成志愿填报。志愿填报截止后,不接受任何形式的补报志愿。



 

32. 未填报集中志愿的考生,是否可以填报该(段)的征集志愿?

  可以。未录取的考生,都可按照征集志愿填报时间安排,登录省教育考试院网站查询征集计划,并在规定时间内填报征志愿


33. 如何对待“冷门”和“热门”专业?

所谓“冷门”,人们通常理解为报考人数相对较少,竞争度 相对较低的专业;所谓“热门”,是指报考的考生较多,竞争十分激烈的专业。 一般来说,“冷”与“热”是与当时的就业情况 相联系的,就业形势好的专业成为考生追逐的热点,然而相当部“热门”专业的就业并不是恒定的,报考时是“热门”,就业 时不一定是“热门”。因为社会对这些专业的需求可能逐渐饱和了,甚至过剩了,反倒成了“冷门”。相反有的专业报考时并不是很“热”,可到就业时却受录用单位的欢迎。有志向的考生不 要被“热门”“冷门”之类的字眼束缚而放弃自己的爱好和兴趣。


33. 如何对待“冷门”和“热门”专业?

所谓“冷门”,人们通常理解为报考人数相对较少,竞争度相对较低的专业;所谓“热门”,是指报考的考生较多,竞争十分激烈的专业。一般来说,“冷”与“热”是与当时的就业情况相联系的,就业形势好的专业成为考生追逐的热点,然而相当部“热门”专业的就业并不是恒定的,报考时是“热门”,就业时不一定是“热门”。因为社会对这些专业的需求可能逐渐饱和了,甚至过剩了,反倒成了“冷门”。相反有的专业报考时并不是很“热”,可到就业时却受录用单位的欢迎。有志向的考生不要被“热门”“冷门”之类的字眼束缚而放弃自己的爱好和兴趣。哪一个更有优势?组织管理能力、艺术修养、口头与书面表达能力,在同学中处于什么位置……。这些都是选择志愿的参考因素。

(3)气质和性格。众所周知,由于先天性遗传因素和后天教育环境的不同,使青少年形成了不同的气质和性格。如有的同学活泼开朗,有的则沉着稳健;有的是急脾气,有的是慢性子;有的反应灵活敏捷,有的则小心谨慎、三思而后行。气质和性格对 人的实践活动有一定的影响,在未来的事业中,每个考生总是要从事一定的实践活动。因此在选择志愿时应考虑考生本人的气质和性格。比如,要求反应迅速灵活的专业,对于多血质和胆汁质 的考生较为合适;要求耐久性和细心谨慎的专业,对于黏液质和 抑郁质的考生较为合适;要求性格活跃、善于交际的某些专业(如新闻、外交、旅游、艺术等),对于外向型性格的考生较为合适;要求沉静和有坚忍不拔毅力的专业(如数学、物理等抽象理论学科),则适合于内向性格的考生。



35. 河北招生考试信息服务网网上咨询活动何时开始?

我省将于6月中旬至高考录取结束,在河北招生考试信息服务网(网址: http://www.hebeeb.com) 设立网上咨询大厅,开展网上咨询活动,相关工作人员将在线与考生互动,解答有关志愿填报、录取等方面的疑问。




河北省2024年普通高校

招生工作新视点

 1.深化艺术类专业考试招生改革。按照教育部统一部署,2024年起,普通高校招生艺术类专业考试分为美术与设计类、音乐类(包括音乐表演、音乐教育两类,其中音乐表演类考试包括声乐、器乐两个方向)、舞蹈类、表(导)演类(包括戏剧影视表演、服装表演、戏剧影视导演三个方向)、播音与主持类、书法类、戏曲类等7个科类。其中,美术与设计类、音乐类、舞蹈类、表(导)演类、播音与主持类、书法类等6个科类实行省级统考;戏曲类实行省际联考。

2024年起,进一步提高文化课成绩要求,艺术史论、戏剧影视文学等部分艺术类专业,直接依据考生高考文化课成绩择优录取;美术与设计类、书法类、舞蹈类、表(导)演类、音乐类等5类省级统考的高考文化课成绩占比调整为50%,播音与主持类仍为70%;校考专业高考文化课成绩要求,调整为执行普通类专业批次录取控制分数线(有关破格录取的,按高校公布的考试招生办法执行)。严格控制校考范围,仅少数高校经教育部批准后,可在省级统考基础上组织校考。

2.改革高校高水平艺术团招生办法。2024年起,高校高水平艺术团不再从高校招生环节选拔,由相关高校从在校生中遴选培养。

3.深化高校高水平运动队考试招生改革。2024年起,进一步完善和规范高校高水平运动队考试招生工作,通过优化招生项目范围,严格报考条件和资格审核,改进考试评价方式,提高文化成绩要求,完善招生录取机制,选拔培养德智体美劳全面发展且具有较高体育竞技水平的学生,为奥运会、世界大学生运动会等重大体育比赛和国家竞技体育后备人才培养体系提供人才支撑。

4.高校选考科目优化调整。教育部印发的《普通高校本科招生专业选考科目要求指引(通用版)》,从2024年高考开始实施。这次高校选考科目优化调整,主要是强化了相关专业对高中学习物理、化学科目的基础性要求。学生可根据自身兴趣爱好及特长,统筹考虑国家和社会需要,结合拟报考高校及专业(类)的选考科目要求,合理确定选考科目。5.优化高职单招考试模式。2024年起,报考高职单招的考生在高考报名所在县(市、区)参加考试;普通高中毕业生文化素质不再使用高中学业水平合格性考试成绩折算替代,所有考生均须参加考试;高职单招报考缴费、打印准考证、查询成绩、填报志愿、查询录取结果等均须登录河北省教育考试院官方网站进行;志愿填报数量由5所院校增加至10所院校。

重要日程备忘

image.png


image.png


 


1.  什么是“学校负责、招办监督”的录取原则? 

高校和省级招办应按照“学校负责、招办监督”的原则实施新生录取工作。高校应按照向社会公布的招生章程中的录取规则进行录取。对思想政治品德考核合格、身体健康状况符合相关专业培养要求、投档成绩达到同批录取控制分数线并符合学校调档要求的考生,是否录取以及所录取的专业由高校自行确定,高校负责对已投档但未被录取考生的退档原因作出解释,高校不得超计划录取。省级招办负贵监督在本地区招生高校执行国家招生政 策、招生计划情况,纠正违反国家招生政策、规定和违背录取规则等行为。


2. 各批各类录取控制分数线是如何划定的? 

省招生委员会根据高校在我省安排的招生计划数、生源情况考生成绩等因素,分类别综合考虑,确定各批次录取控制分数线。

普通类根据考生高考文化总成绩(含政策性加分),按照历史科目组合、物理科目组合招生计划数的一定比例,结合生源等情况,分别划定本科、专科录取控制分数线以及特殊类型招生录取控制分数线。特殊类型招生录取控制分数线主要用于强基计划、高校专项计划及其他有相应文化成绩要求的院校(专业)招生。

艺术类不区分历史科目组合、物理科目组合,根据艺术各类别考生高考文化总成绩(含政策性加分)、专业考试成绩和招生计划数,结合生源等情况,划定各类别本科、专科录取控制分数线。

    体育类根据考生高考文化总成绩(含政策性加分)、专业考试成绩,按照历史科目组合、物理科目组合招生计划数的一定比例,结合生源等情况,分别划定本科、专科录取控制分数线。

对口类根据考生对口总成绩(含政策性加分),按照各专业类招生计划数的一定比例,结合生源等情况,分别划定本科、专科录取控制分数线。


3.  投档原则是什么?

普通高校招生普通类、体育类按照历史科目组合和物理科目组合,分别投档录取;艺术类一般不区分历史科目组合和物理科目组合, 一起投档录取,有特殊要求的结合高校有关要求投档。

投档模式分为顺序志愿和平行志愿两种。顺序志愿的调档比例原则上在120%以内,平行志愿的调档比例原则上在105%以内,高水平运动队、高校专项计划、部分艺术类等特殊类型招生的高校专业不限定投档比例。

(1)普通类

顺序志愿投档原则为“志愿优先、遵循分数”,将控制线上未录取的有志愿考生,结合高校要求,依据高校调档比例,按高考文化总成绩(含政策性加分)从高分到低分排序依次投档,由高校择优录取。

平行志愿投档原则为“分数优先、遵循志愿、 一次投档、不再补档”,将控制线上未录取的有志愿考生,结合高校各专业(类)要求,依据高校调档比例,按高考文化总成绩(含政策性 加分)从高分到低分排序,遵循考生的志愿顺序依次投档,由高校择优录取。

投档时,当遇到多名考生高考文化总成绩(含政策性加分)相同时,依次按语文数学两门成绩之和、语文数学两门中的单科最高成绩、外语单科成绩、首选科目单科成绩、再选科目单科最高成绩、再选科目单科次高成绩由高到低排序投档;如仍相同,比较考生志愿顺序,顺序在前者优先投档,志愿顺序相同则全部投档,是否录取由高校决定。

军队院校、综合评价招生等有特殊投档要求的,按照有关文件执行。

(2)艺术类、体育类

顺序志愿按照“志愿优先”的原则,根据考生志愿,将报考该校且符合条件的考生档案全部提供给有关高校审阅,由高校根据向社会公布的招生章程(或招生简章)中的录取规则进行录 取,遗留问题由高校负责解释。

平行志愿按照“分数优先、遵循志愿、一次投档、不再补”的原则,将控制线上未录取的有志愿考生,结合高校各专业(类)要求,依据高校各专业(类)调档比例,按综合成绩从高分到低分排序,遵循考生的志愿顺序依次投档,由高校择优录取。投档时,当遇到多名考生综合成绩相同时,依次按高考文化总成绩(含政策性加分)、语文数学两门成绩之和、语文数学两门中的单科最高成绩、外语单科成绩、首选科目单科成绩、再选科目单科最高成绩、再选科目单科次高成绩由高到低排序投档如仍相同,比较考生志愿顺序,顺序在前者优先投档,志愿顺序相同则全部投档,是否录取由高校决定。

综合成绩计算公式为:

①美术与设计类、音乐类、舞蹈类、书法类、戏剧影视表演 类、戏剧影视导演类、服装表演类:综合成绩=高考文化总成绩(含政策性加分)×0.5+(专业成绩÷专业满分) ×750×0.5, 结果四舍五入保留3位小数。其中,专业满分均为300分。

②播音与主持类:综合成绩=高考文化总成绩(含政策性加)×0.7+(专业成绩÷专业满分)×750×0.3,结果四舍五入保留3位小数。其中,专业满分为300分。

③体育类:综合成绩=高考文化总成绩(含政策性加分)×0.3+(专业成绩÷专业满分)×750×0.7,结果四舍五入保留3位小数。其中,专业满分为400分。

  (3)高水平运动队、高校专项计划等特殊类型

高水平运动队、高校专项计划等特殊类型招生,将填报志愿 且符合条件的考生档案全部提供给有关高校审阅,由高校根据向社会公布的招生章程中的录取规则进行录取,遗留问题由高校责解释。

(4)对口类

对口类专业按对口总分(含政策性加分)投档,实行平行志愿。投档原则为“分数优先、遵循志愿、一次投档、不再补档”, 将同一专业类控制线上未录取的有志愿考生,结合高校要求和调档比例确定投档考生数,按对口总分(含政策性加分)从高分到低分排序,遵循考生的志愿顺序依次投档。

遇到多名考生对口总分(含政策性加分)相同且多于应投考生数时,依次比较语文、数学、英语单科成绩,若单科成绩均相同,则全部投档。


4.  高校招生录取规则是什么? 

高校招生录取规则由高校依据国家有关规定制定,并通过招生章程向社会公布。主要内容有选考科目要求、调档比例、专业安排办法、身体要求、外语语种、成绩要求等。在统考成绩达到同批录取控制分数线的考生中,高校确定调阅考生档案的比例。我省在国家允许的投档比例范围内,按高校提出的调档比例和相关工作规程向其投放考生电子档案。招生院校在我省投放的考生电子档案范围内,按事先公布的录取规则择优录取。

以“学校”为志愿单位投档的高校,专业安排办法一般有以下三种方式:

(1)分数优先。所谓分数优先就是投档到该校的所有考生,从高分的学生开始依次按专业进行录取,也就是说,分数高的学 生优先按专业顺序进行录取。

(2)志愿优先。所谓志愿优先就是在报考该院校的考生中,首先考虑的是考生的专业志愿,也就是说先考虑第一专业志愿,再考虑第二专业志愿,依次类推。比如,一个考生未能达到第一专业志愿最低录取分数,将转人下面的专业志愿排序,依次类 推。如该专业前面已录满,不管该生分数多高,也不能进入该 专业。

(3)分数级差。所谓分数级差就是在考生的前后两个专业之间设定一个分数级差,比如分数级差是3分,那么某一专业第二 专业志愿的考生只要比报考该专业的第一志愿考生高出3分以上,报考该专业第三志愿的考生只要比报考该专业的第二志愿考生高3分以上,比报考该专业的第一志愿考生高出6分以上,就可以优先录取到本专业。

另外,还要了解高校对考生的加分政策、优惠条件、投档分数等其他要求。具体专业安排办法请认真阅读高校招生章程或向有关院校咨询。


5. 各批次(段)在录取时是否相互影响? 

录取是按批次(段)依次进行的,如果考生被 一批次(段)的院校录取,就不能参加以后批次(段)的录取,也不得转录;如果考生在某一批次(段)未被录取,将自动转人下一批(段)重新开始参加录取,其上一批次(段)所填志愿不会影响下一批次(段)录取。


6.  网上录取基本流程是什么?

(1)院校进行网络用户注册。

(2)院校核对招生计划。

(3)院校根据模拟投档(本科批)情况调整投档比例和招生计划。

(4)我省按院校调档要求提供考生电子档案

(5)院校网上审阅考生电子档案。

(6)院校拟定预录取名单和预退档名单。

(7)录取审核。

(8)统一打印、寄发录取新生名册等材料。

    (9)院校按新生名册寄发考生录取通知书。


7. 平行志愿投档是如何进行的? 

实行平行志愿的批次,投档时,按照“分数优先、遵循志愿、一次投档、不再补档”的原则,将控制线上未录取的有志愿考生按投档成绩从高到低排序,结合高校各专业(类)要求,依据高校调档比例确定投档考生数进行投档。具体地讲,就是先从排在第一位的考生开始,计算机根据该生填报的1、2、3、4、5……多个平行志愿专业(类)或院校,从第1个志愿开始依次 索,如果第1个志愿有空额就投档,如已满额,则检索第2个志愿,依此类推。一旦投档,该生后面的志愿即失效。然后再检索排在第二位的考生……直到所有专业(类)或院校计划满额或者没有符合投档条件的考生为止。如果某一考生填报的所有志愿都不符合投档条件,则不能被投档。

特别强调的是:在同一批(段)虽然考生一次可以填报多个志愿,但在实际投档过程中,投档机会最多只有一次,也就是说,无论有几个志愿符合投档要求,只能投档到排在前面的那一个,一旦投档,其余志愿随即失效,不再投档。投档后因某种原因被退档,即使后面还有符合投档条件的志愿,也不再投档。

每批(段)平行志愿投档后,如专业(类)或院校退档后出现缺额,根据公平原则,不能再续发考生档案。缺额计划通过征集志愿完成。


8.  征集志愿是如何投档的? 

征集志愿投档原则与集中填报志愿的投档原则相同。相关批次最后一次征集志愿录取时,如果线上生源不足,可能有降分投档的 


   9. 什么是投档线? 

在录取时,按照院校专业(类)的招生计划和调档要求,根据考生志愿填报情况,向招生院校专业(类)批量投档。批量投档的分数最低的考生成绩即为该院校专业(类)的(提档线、调档线、进档线

10.  国家确定的优惠加分项目有哪些?分值是多少?

符合下列条件的考生可在考生文化统考成绩总分基础上增加分值投档,由高校审查决定是否录取。

(1)少数民族自治县(含民族县)的少数民族考生增加10分

(2)归侨、归侨子女、华侨子女和台湾省籍(含台湾户籍)考生增加10分。

(3)自主就业退役士兵增加10分。

(4)烈士子女增加20分。

(5)在服役期间荣立二等功以上或被战区以上单位授予荣誉称号的退役军人增加20分。


11.  河北省确定的优惠加分项目有哪些?分值是多少?

具有河北省户籍的农村独生子女考生增加10分,加分范围只适用于我省高校在省内招生,可在考生文化统考成绩总分基础上增加分值投档,由高校审查决定是否录取。2016年1月1(含)以后出生的农村户口独生子女考生参加高考,不再享受加分政策。


12.  如何查询优惠加分信息? 

考生可登录河北省教育考试院官网“普通高考信息服务”平台,在“信息查询”栏目个人信息表中查看优惠加分项目是否公示通过。


13.  投档录取时如何使用优惠加分?

对于同时符合多种加分条件的考生,投档时,报考省外院校的使用国家加分最高一项,报考河北省院校的使用国家加分与内加分最高一项。具备加分资格的考生,我省按照加分后的分数投档,录取时是否考虑加分因素,由招生院校按有关规定执行。所有高考加分项目及分值均不得用于高校不安排分省招生计划的艺术类专业、高水平运动队、高校专项计划等招生项目



14.  优先录取的规定有哪些? 

优先录取是录取环节的照顾政策,包括两种照顾方式。 一是参加全国统考录取并达到有关高校投档要求的优先录取,指对符合这类照顾条件的考生,按有关高校投档要求进行投档后,高校 对进对进档考生中符合照顾资格的考生应予以优先录取。符合这些条 件的考生虽然没有享受加分,但根据志愿和分数等情况被投档到高校之后,应优先录取。二是同等条件下优先录取,指考生被投 档到高校之后,如果一些考生条件相当(如相关考试成绩、专业志愿、身体条件、获奖情况等条件相同),但因高校招生计划足不能全部录取时,应优先录取符合照顾条件的考生。两种优先录取类型的照顾体现在投档后的高校录取环节,在投档环节没有照顾。

(1)平时荣获二等功或者战时荣获三等功以上奖励军人的子女,一至四级残疾军人的子女,因公牺牲军人的子女,驻国家确定的三类以上艰苦边远地区和西藏自治区、解放军总部划定的二类以上岛屿工作累计满20年军人的子女,在国家确定的四类以上艰苦边远地区或者解放军总部划定的特类岛屿工作累计满10年军人的子女,在飞或停飞不满1年或达到飞行最高年限的空勤军人的子女,从事舰艇工作满20年军人的子女,在航天和涉核岗位工作累计满15年军人的子女,参加全国统考录取并达到有关高校专(类)投档要求的,应予以优先录取。烈士子女、因公牺牲军人的子女、现役军人子女报考军队校,在投档范围内优先录取。具体以当年有关文件为准。

(2)退出部队现役的考生、残疾人民警察参加全国统考录取并达到有关高校专业(类)投档要求的,在与其他考生同等条件下优先录取。

(3)公安烈士、公安英模、因公牺牲、一级至四级因公伤残 公安民警的子女参加全国统考录取的,按照《关于进一步加强和改进公安英烈和因公牺牲伤残公安民警子女教育优待工作的通知》(公政治[2018]27号)的有关规定执行。

(4)国家综合性消防救援队伍人员及其子女参加全国统考录取的,按照《关于做好国家综合性消防救援队伍人员及其子女教育优待工作的通知》(应急〔2019〕37号)的有关规定,参照军人有关优待政策执行。

(5)经共青团中央青年志愿者守信联合激励系统认定,获得5A级青年志愿者的,达到有关高校专业(类)投档要求的,在与其他考生同等条件下优先录取。


15. 军队院校如何录取? 

在本科提前批A 段录取,实行以学校为单位的顺序志愿模式,设1次集中填报志愿和1次征集志愿,每次可填报1所院校,每所院校设6个专业志愿和1个专业服从调剂选项。

集中填报志愿录取时,根据省军区招生办提供的政治考核、军检合格考生名单,依据考生志愿和分数,区分性别,按招生计划数量的120%投档(投档数量按照四舍五入取整)。总分相同的依次比较语文、数学、英语单科成绩。除同分考生外,不得超比例投档。军队院校根据考生填报的专业志愿,由高分到低分依次确定录取对象。达不到相应专业军检标准的,不得录取到该专业。

集中填报志愿录取结束后,如果没有完成招生任务,将向社会公布缺额计划,公开征集文化成绩达到我省特殊类型招生录取控制分数线且军检合格考生志愿。征集志愿录取时,按照缺额计数量的100%向军队院校投档。院校不得拒录符合录取条件的 征集志愿考生。烈士子女、因公牺牲军人的子女、现役军人子女,在投档范围内优先录取。具体规定以当年军校招生文件为准。



16. 公安院校如何录取? 

公安院校根据招生层次分别安排在本科提前批 A 段和专科提前批,均实行顺序志愿投档,设1次集中填报志愿和1次征集志愿,每次可填报1所院校,每所院校可填报6个专业志愿和1个专业服从调剂选项。

每批次集中填报志愿录取时,将报考该院校的政治考察和面 试、体检、体能测评均合格且达到相应批次控制线的考生,依据考生分数,按照招生计划数量的100%投档,招生院校负责审查录取。如集中填报志愿未完成招生计划,向社会公布缺额计划,进行征集志愿填报,并按照计划缺额数量的100%向招生院校投放征集志愿档案。招生院校负责对投档但未录取考生的解释及其他遗留问题的处理。具体规定以当年公安招生文件为准。


17. 司法类院校提前录取专业如何录取?

根据教育部、司法部有关文件规定,中国政法大学、西南政法大学、中南财经政法大学、华东政法大学、西北政法大学等学校的侦查学、治安学、边防管理和刑事科学技术等专业和中央司法警官学院各专业在本科提前批A段录取,设1次集中填报志愿1次征集志愿,每次可填报1所院校,每所院校设6个专业志 愿和1个专业服从调剂选项,实行顺序志愿投档。我省从集中填报志愿报考该院校的上线考生中,依据考生分数,按照招生院校提出的调档比例投档,招生院校负资审查录取。当集中填报志愿上线考生数量不足时,进行征集志愿投档录取。如个别司法院校进行政治考察、面试及体能测试,我省从政治考察、面试及体能测试合格且集中填报志愿报考该院校的上线考生中,依据考生分数,按照招生院校提出的调档比例投档,招生院校负资审查录取。当集中填报志愿上线考生数量不足时,进行征集志愿投档录取。具体规定以当年司法类院校招生政策为准。


18.  艺术类专业如何录取? 

艺术类专业招生录取分为本科提前批、专科提前批两个批次。本科提前批分为A、B 二段,按顺序依次录取;专科提前批分平行志愿和顺序志愿两种模式,考生同一个志愿只能选择其中一种模式填报,不能兼报。高校艺术类专业实行以统一高考为基础、省级统考为主体,依据高考文化成绩、专业考试成绩,参考学生综合素质评价的招生录取模式。

(1)高校不组织专业考试的艺术类专业,直接依据考生高考文化课成绩、参考考生综合素质评价,择优录取。

(2)使用省级统考成绩作为专业考试成绩的艺术类专业,在考生高考文化课成绩和省级统考成绩均达到河北省艺术类专业录取最低控制分数线基础上,依据考生高考文化课成绩和省级统考成绩按比例合成的综合成绩,实行以“专业(类)+学校”为单位的平行志愿模式择优录取,平行志愿填报数量为70个。

(3)少数组织校考的高校艺术类专业,在考生高考文化课成绩达到河北省普通类专业批次录取最低控制分数线、省级统考成绩合格且达到学校划定的最低成绩要求基础上,依据考生志愿、校考成绩择优录取,实行以学校为单位的顺序志愿模式。

对于在相关专业领域具有突出才能和表现的考生,高校可探 索制定高考文化课成绩破格录取办法。破格录取办法须经学校党委常委会审议并报所在地省级教育行政部门备案,提前在学校考试招生办法中向社会公布。破格录取考生名单须经学校招生工作领导小组审议并报生源所在地省级招委核准后在学校招生网站公示。

(4)实行省际联考的高校戏曲类专业使用省际联考成绩作为考生专业考试成绩。在考生高考文化课成绩达到河北省艺术类专业高考文化课录取控制分数线基础上,依据考生志愿和省际联考成绩择优录取。

(5)录取规则。由高校根据学校实际制定并向社会公布。录取时,按照我省投档原则向高校投档后,高校在投档生源范围内按照事先向社会公布的录取规则进行录取。遗留问题由高校负责解释。

具体投档原则见本章第3条,录取时间及相关要求按河北省有关文件执行。


19.  普通体育类专业如何录取? 

普通体育类专业招生录取分为本科提前批、专科提前批两个批次,本科提前批分为A、B两段,按顺序依次录取;本科提前B段、专科提前批,均实行平行志愿投档。投档原则见本章第3条。

河北体育学院少数民族传统体育项目安排在本科提前批A段录取,实行顺序志愿投档

录取规则:由高校根据实际制定并向社会公布。录取时,我省按照河北省投档原则向高校投档后,高校在投档生源范围内按照事先向社会公布的录取规则进行录取。遗留问题由高校负责解释。


20.  高水平运动队招生如何录取? 

2024年起,高水平运动队考生文化考试成绩全部使用全国统 一高考文化课考试成绩。试点高校按照本校发展定位和人才培养要求,合理确定本校运动队录取考生文化课成绩要求。相关文件 规定的部分“双一流”建设高校对考生的高考成绩要求须达到生源省份普通类本科批次录取控制分数线;其他高校对考生的高考成绩要求须达到生源省份普通类本科批次录取控制分数线80% 

对于体育专业成绩突出、具有特殊培养潜质的考生,高校可探索建立文化课成绩破格录取机制。破格录取办法须经学校党委常委会审议并报所在地省级教育行政部门备案,提前在学校高水平运动考试运动队考试招生办法中向社会公布。破格录取考生名单须经学校招生工作领导小组审议并报生源所在地省级招委核准后在学校招生网站进行公示。

高考成绩公布后、志愿填报前,省级招生考试机构向相关高校提供合格考生的高考文化成绩,高校在高考文化课成绩达到录取要求的合格考生范围内,依据体育专业测试成绩,择优确定入围考生名单并按规定公示。围考生人数不得超过学校运动队招生计划数。

报考高校高水平运动队的考生,在本科提前批C 段录取,根据考生所报志愿,将具有高水平运动队招生资格且经教育部“阳光高考”信息平台公示的考生,按高校要求,向高校提供档案,由高校审查决定是否录取。在录取考生中,高考文化课成绩不低于高校相关专业在生源省份录取分数线下20分的考生,可录取至对应的普通专业;其余考生限定录取至体育学类专业。高校应在普通本科批次录取结束前确定考生录取专业。

高水平运动队学生要接受所在院系和运动队双重管理,既要按本专业培养方案完成专业学习任务,也要认真履行参加训练和比赛的义务。


21. 定向就业招生如何录取? 

定向就业招生在同批非定向投档前,按照院校招生计划,在该校非定向本科批同科类普通类专业最低投档线下20分以内、同批控制分数线以上,根据考生志愿从高分到低分投档,由院校审核录取。在本批次非定向投档前审核录取完毕。其中定向西藏的最多降40分,不受控制线限制。

若院校完不成定向就业招生计划,则转为非定向计划执行。其中,定向西藏就业招生计划因报考生源不足确需调整计划的,要按程序由各高校主管部门报教育部审批。


22. 国家专项计划招生如何录取? 

国家专项计划招收具有国家专项计划招生资格的考生,在本科提前批A 段和本科提前批B段录取。设1次集中填报志愿和1次征集志愿。

本科提前批A段,实行顺序志愿投档。生源不足时,通过志愿征集和适当降分录取。

本科提前批B段,实行平行志愿投档,集中填报志愿录取控制分数线按我省招生计划中公布的分数线执行。生源不足时,通过征集志愿和适当降分录取。


23.  高校专项计划招生如何录取?

   高校专项计划招收具有高校专项计划招生资格的考生,在本科提前批C 段录取,执行高校规定的相关分数要求,

考生须经招生院校考核合格且经教育部“阳光高考”信息平台公示,根据考生志愿、文化成绩和招生院校要求投档,由高校审查决定是否录取。


24.  地方专项计划招生如何录取? 

地方专项计划招收具有地方专项计划招生资格的考生,在本科批录取,执行我省本科批文化控制线,实行平行志愿投档。


25.  国家专项计划、高校专项计划、地方专项计划录取考生 放弃入学资格或退学有什么后果? 

根据教育部规定,从2023年起,往年被国家专项计划、高校专项计划、地方专项计划录取后放弃入学资格或退学的考生,不再具有专项计划报考资格。


26.  少数民族本科预科班如何录取? 

少数民族本科预科班是根据少数民族学生的特点,采取特殊措施,着重提高文化基础知识,加强基本技能的训练,使学生在德育、智育、体育几个方面都得到进一步发展与提高,为在高校进行专业学习打下良好基础所开设的一种教学班制度。少数民族本科预科班从当年参加统考的少数民族考生中择优录取,在本科批进行,实行平行志愿投档。录取时,少数民族本科预科班录取分数不低于同批次同科类各有关高校非预科班普通类专业(不含地方专项计划)最低投档分数线下80分;征集志愿录取时,少数民族本科预科班录取分数不低于同批次同科类相应高校第一次投档录取时的非预科班普通类专业(不含地方专项计划)最低投档线下80分。


27. 边防军人子女预科班如何录取?

边防军人子女预科班只招收通过边防军人子女资格审查的考生,在本科批进行,实行平行志愿。其中,边防军人子女预科班录取分数不低于院校所在批次控制线下60分,重庆大学等部分高校的边防军人子女预科班录取分数要求不低于特殊类型招生录取控制分数线。


28.残疾考生报考普通高校录取有什么规定? 

对残障考生招生,有专门的绿色通道。近年经教育部批准,具备招收残障考生资格的院校有北京联合大学、天津理工大学、长春大学、郑州工程技术学院、长沙职业技术学院、贵州盛华职 业学院、河南推拿职业学院、滨州医学院、南京特殊教育师范学院、西安美术学院、郑州师范学院、乐山师范学院、山东特殊教育职业学院、浙江特殊教育职业学院、绥化学院、重庆师范大学等高校,招收对象为符合高考报名条件的残障考生,具体招生事宜考生可向招生院校咨询或查阅院校招生简章。


29.  单独招收残障考生的院校有哪些? 

对残障考生招生,有专门的绿色通道。近年经教育部批准, 具备招收残障考生资格的院校有北京联合大学、天津理工大学、长春大学、郑州工程技术学院、长沙职业技术学院、贵州盛华职业学院、河南推拿职业学院、滨州医学院、南京特殊教育师范学院、西安美术学院、郑州师范学院、乐山师范学院、山东特殊教育职业学院、浙江特殊教育职业学院、绥化学院、重庆师范大学等高校,招收对象为符合高考报名条件的残障考生,具体招生事宜考生可向招生院校咨询或查阅院校招生简章


30. 控制分数线上考生未被录取的主要原因有哪些? 

(1)填报了无效志愿。如汉族考生填报少数民族本科预科班志愿;不具备资格考生填报国家专项计划志愿;填报志愿时未按要求操作等。

(2)未达到所填报院校专业(类)的录取分数线。

(3)身体状况或相关科目等不符合院校专业(类)要求。如有的考生身体条件受限,盲目选报一些受限院校专业(类);没有参加外语口试的考生报考需要口试的专业(类)等。

(4)教育部规定,根据考生成绩,从高分到低分,按略多于招生计划总数确定控制分数线。既然按略多于招生计划总数确定控制分数线,那么这“略多于”的部分在录取过程中就有可能落选。


31. 如何准确查询录取结果? 

招生院校依法依规寄发的录取通知书,是已录取考生获得录取结果的正式渠道。

查询渠道:

(1)通过省教育考试院网站(网址:http://www.hebeea.edu.cn)省教育考试院微信公众号 (hbsksy)、省教育考试院APP客户端(扫描省教育考试院网站的二维码下载并安Android系统或iOS系统的手机客户端软件)查询本人录取结果。 

(2)通过县(市、区)高考咨询服务站查询本人录取结果

(3)通过院校网站查询。


32. 如何识别和防范招生诈骗行为? 

往往在高考录取工作开始前,一些不法分子受经济利益驱 使,利用考生和家长的急切心理,散布虚假信息、贩卖制造虑,针对志愿填报、招生录取等环节实施诈骗,严重扰乱考试招 生秩序。结合近年来个别地方出现的一些虚假诈骗情况,提醒广大考生和家长,通过官方渠道查询、咨询有关信息,勿存侥幸心理、勿信不实信息,谨防上当受骗。如果遭遇诈骗事件,应及时向公安机关报案,维护自身合法权益!

(1)认准官方权威渠道,“小道消息”不可信

【现象】近年来,一些社会机构和个人通过网络平台发布有 关涉考涉招信息,存在政策解读不准确、信息提供不真实等问题,有的甚至通过假冒、仿冒高校和招生考试机构的官方网站、公众账号编造散布虚假信息,迷惑考生和家长,实施诈骗,严重扰乱考试招生秩序。

【提醒】教育部会同有关部门深入实施高考护航行动,开展 “点亮权威考试招生机构官网标识”专项行动,已协调有关互联 网平台对各级招生考试机构和高校的官方网站、微信公众号等进 行了权威标识,便于广大考生和家长识别,获取正规的招生政策 和服务信息。此外,每年具有招生资格的全国高等学校名单都会 在教育部官方网站发布。考生和家长可通过教育部官方网站“文 献”栏目下的“全国高等学校名单查询”进行查询核实。在此提 醒广大考生和家长,在网上查询高校招生信息时,请认准“官”标识,谨防山寨账号或网站骗局。

(2)警惕“天价志愿辅导”陷阱,理性报考不迷信

【现象】近年来,每当高考成绩发布后,一些社会培训机构 和个人“网红”就通过“不浪费1分不浪费未来”“让1分都发 挥价值”等说法吸引流量,夸大志愿填报作用,制造贩卖志愿填 报焦虑。然后,包装打造“专家”“名师”、虚假编造“内部信”“内部数据”、宣称“大数据志愿辅导”“一对一咨询”等, 诱导考生及家长购买高价、天价志愿填报咨询服务。

【提醒】有关部门从未发放过“高考志愿规划师”这类职业资格证书,某些社会机构所谓的“志愿规划师”“专家名师”等大多是临时招募的社会人员,按照机构提供的“台词”给考生和 家长辅导。每年高考填报志愿前,省级招生考试机构都会发布当年高考成绩统计情况、近年来各高校录取情况和志愿填报参考材 料,高校也会介绍学校招生章程、招生办法和往年录取参考信息,并不存在所谓的“内部信息”“内部数据”。同时,在志愿填 报期间,各地都建立了招生考试机构、高校、中学等多方面志愿 填报咨询公共服务体系,多渠道多方式提供志愿填报咨询服务。考生可通过所在地各级招生考试机构和各高校官方网站、官方微信公众号或编发的志愿填报相关资料等查询所需参考信息,也可 通过拨打当地招生考试机构、高校和中学开通的咨询电话或线上咨询等方式了解相关政策和信息。

考生和家长不要迷信所谓高价、天价志愿填报咨询服务,要合自身情况,综合考虑个人志向、兴趣爱好、特长优势、生涯规划等多种因素,着眼于国家需要和社会需求,科学选择,自主填报。

(3)了解高校招生政策,“特殊渠道”不存在

【现象】高校招生录取期间,一些不法分子往往利用家长望  子成龙的心态,声称“认识某某领导”“认识某某招办”,甚至通过伪造文件、私刻印章、设立报名处和咨询电话、假冒高校招生 工作人员等,谎称可以“走后门”“搞特殊”,通过“内部指标”“机动计划”“计划外补招”“降分补录”等说法实施诈骗。

2021年,山东日照公安部门成功破获一起高考录取诈骗案,抓获犯罪嫌疑人赵某、刘某某。经查,犯罪嫌疑人赵某、刘某某 通过微信朋友圈等渠道,对外宣称能够帮助考生以特招的形式办理高校录取入学手续,从中骗取家长和考生的高额手续费。犯罪 嫌疑人赵某、刘某某已被依法审查,24万元被诈骗钱款已全部追回。

【提醒】教育部“三十个不得”招生工作禁令中明确要求,“不得控自扩大高校招生规模或调整高校招生计划”“不得无计划 或擅自突破计划规模进行招生或违反计划管理要求调整计划“省级招办不得违反投档工作程序或在政策之外降低标准向有关高校投放考生档案”“不得违反规定的招生程序降低标准录取考生、拒绝录取符合条件的考生”。在此提醒考生和家长,高校招生是严格按照招生政策规定和学校招生章程等实行计算机远程网  上录取的,有严格的工作流程和监督机制。高校招生计划由省级招生考试机构统一向社会公布,不存在所谓“内部指标”。高校在招生省份未完成的招生计划,会通过公开征集志愿录取,不存在所谓内部降低分数“补录”“补招”的情况。正规的招生录取不会产生任何附加费用,凡是需要收取保证金、录取费、指标费的, 一律不能相信。

(4)认清不同教育区别,擦亮眼睛看仔细

【现象】近年来,一些不法分子利用考生和家长急于上大学的心理,故意混淆普通高等学历教育与其他教育形式之间的别,以自考助学班、网络教育班、国际合作办学等入学通知书蒙 骗考生及家长。部分办学机构打着普通高等学历教育的幌子,实际招收其他教育形式的学员。

【提醒】普通高等学历教育学生均须通过高考招录。上述所谓的“录取通知书”不是普通高等学历教育的录取通知书,“入”后也不能进行普通高等学历教育新生学籍电子注册,更拿不到普通高等学校学历证书。考生和家长要擦亮眼睛,认真辨别普 通高等学历教育与其他教育形式,选择适合自己的学习进修方式。

(5)做好个人信息防护,谨防志愿被篡改

【现象】考生高考志愿是高校录取的重要依据。恶意篡改将 导致考生无法被心目中理想的高校录取。2021年,某省高考考生 周某(化名)及其家属到派出所报案,称发现自己的高考志愿被人篡改了,改成了比自己高考成绩低很多的院校。经立案侦查,确定其同学吴某(化名)有重大嫌疑。经讯问,吴某称觉得周某 平时影响了自己的学习,这样做是为了惩罚周某。吴某篡改别人 高考志愿的行为,受到了法律的制裁。

【提醒】律师和专家提醒,篡改高考志愿本质上是对考生受 教育权的侵犯。从司法实践来看,篡改高考志愿的行为可能涉及破坏计算机信息系统罪、侵犯公民通信自由罪以及侵犯公民个人信息罪。不要认为“无知者无畏”,不要以为事发了还能“恶 作剧”等理由搪塞,违法者终会受到法律的制裁。别提醒考生:有的考生委托中学、高校或他人代为填报志愿,将自己的考生号、登录密码等信息透露给委托人,录取时发现与本人设想相去甚远,想修改志愿但密码已被他人篡改,追悔 莫及。为维护考生本人利益,请务必提高防范意识,妥善保管好 自己的身份证号、考生号等个人信息和志愿填报系统密码等登录 信息,不要泄露给他人,防止志愿被他人篡改,更不要委托他人填报。


33. 录取常见问题有哪些? 

【提问】一名普通类物理科目组合考生,去年高考成绩439分,本科批集中填报志愿时报了某大学的能源化学工程专业没有被录取,该专业却出现在了征集计划中,请问是什么原因?

【回答】在这种情况下,学校进行志愿征集:一种是在集中 填报志愿投档时,我省依据院校招生计划,将符合条件的考生一次性投档到相应专业且满额,但个别进档考生因身体原因(如高血压病、心肌病等)不符合院校(专业)录取规则被退档,形成缺额;另一种是集中填报志愿投档后,学校向我省新增加了招生 计划。针对上述情况,根据公平原则,我省不再向院校递补考生档案,缺额计划和新增计划全部通过征集志愿完成。

【提问】填报本科批平行志愿时填写了某大学的装备与控制工程专业,成绩达到该校该专业投档最低分没有被录取,却被填在后面的志愿录取了,是什么原因?

【回答】出现此类情况是因为你没有达到同分排序最低成绩。普通类平行志愿投档,按照高考成绩从高分到低分,依次遵循考生志愿顺序进行,当遇到多名考生成绩相同时,计算机依次按语文数学两门成绩之和、语文数学两门中的单科最高成绩、外语单  科成绩、首选科目单科成绩、再选科目单科最高成绩、再选科目单科次高成绩由高到低排序投档;如仍相同,比较考生志愿顺序,顺序在前者优先投档,志愿顺序相同则全部投档,是否录取由高校决定。我省公布的平行志愿投档情况统计表,既有投档最低分,也有投档最低分同分考生排序成绩,方便考生查询

  【提问】某考生在本科批平行志愿填报了某大学的临床医学 专业,总成绩超过了该专业的投档最低分,该批录取结束后仍未查到录取结果,是什么原因?

【回答】出现这一情况说明考生没有被录取,原因是投档后,学校阅档过程中发现其不符合该专业的有关要求,将档案退回。比如,《普通高等学校招生体检工作指导意见》规定,对于色弱、色盲考生,化学类、化工与制药类、药学类、生物科学类、公安技术类、地质学类各专业、医学类各专业等高校可不予录取。此外,有的学校还在招生章程中进一步对裸眼视力、身高等其他身体条件提出要求。如果考生不符合学校的这些要求,就会被 退档。

在此特别提醒考生,填报志愿时一定要认真查看自身体检结果,仔细阅读《普通高等学校招生体检工作指导意见》和学校招生章程,避免因不符合要求被退档。


34. 新生入学报到前应做哪些准备? 

接到大学录取通知书,就意味着你将是一名大学新生了,必须从各方面做好成为一名大学生的准备。

(1)各种证件和材料的准备。为了能够按时报到,必须按照随录取通知书一起寄来的“新生须知”上的要求准备好报到所需的各种证件和材料,如准考证、身份证、户口迁移证明、高考纸介质档案(河北省2024年普通高校录取新生综合信息登记表)、高级中等教育阶段档案(或人事档案)和党、团关系等证件和材料,和录取通知书放在一起准备报到时用。

(2)经济上的准备。准备好报到时需交纳的学费、住宿费等院校要求的各种费用,同时要准备好到校后生活所需的伙食费及零用钱。

(3)学习和生活必需品的准备。尤其是到外地上学的学生,需要准备好四季的衣服、一些生活用品及学习用品。

(4)旅途准备。大多数新生都要到外地上学,人学报到需途旅行。因此行前要周密安排,买好火车票,凭录取通知书新生可以享受半价。要注意报到时间,及时从家里出发,以免影响报到注册。

     (5)思想准备。新生更重要的是做好思想上的准备。许多新生从小到大从未离开过父母,不习惯独立生活,而大学恰好是要 培养学生独立生活的能力,所以新生从思想上要做好离开父母自 己生活的准备,准备独立面对生活,解决可能出现的问题。同时,应该做好继续努力学习的准备,认识到迈入大学是人生路上一个新起点。大学的学习内容、学习方法与中学相比都有明显的不同,要有勇气处理即将遇到的一切困难和问题,对大学中可能 遇到的种种难题都要有战胜和克服它们的勇气和信心。

(6)考生凭录取通知书办理户口迁移手续,并按高校规定的时间及有关要求办理报到等手续。不能按时报到的,应向高校提出书面申请,经校方同意后,方可延期报到。新生在报到期间无故不报到的,将被取消入学资格。


35. 新生入学需要携带哪些档案?

考生档案包括高考纸介质档案,中学学籍档案(包括学籍及相关评价信息、党团关系等)或人事档案,入学报到时须一并交予高校学籍管理部门。高考纸介质档案为“河北省2024年普通高校录取新生综合信息登记表”,考生可凭居民身份证、准考证和高校录取通知书到所在中学或报名地的县(市、区)招办领取。中学学籍档案或者人事档案,需要到所在中学或者工作单位领取。



36.考生如何申请国家助学贷款? 

   家庭经济困难学生可申请办理国家助学贷款,解决学费、住宿费及部分生活费。本专科生每生每学年最高申请金额不超过 12000元。贷款学生在校学习期间的助学贷款利息全部由财政补贴,毕业后的利息由贷款学生本人全额支付,还款期限原则上按学制加15年确定,最长不超过22年。国家助学贷款是信用贷款,

   学生不需要办理贷款担保或抵押,但需要承诺按期还款,并承担 相关法律责任。国家助学贷款包括生源地信用助学贷款和校园地国家助学贷款,我省具体办理方式为:

(1)生源地信用助学贷款(国家开发银行河北省分行、农村信用社承办)。

贷款对象:河北籍家庭经济困难的全日制普通高校本专科生(含高职、第二学士学位、预科)和研究生。

贷款方式:由学生户籍所在地县级学生资助机构或农村信用社县(市、区)联社统一办理。

申请方式:学生在新学期开始前,登录河北省生源地信用助学贷款管理系统网站(网址: http://syd.hee.gov.cn) 或国家开发银行生源地助学贷款学生在线系统网站(网址: http://www.csls.cdb.com.cn), 填写统一的贷款申请表,学生和家长向户籍 所在地县级教育部门申请。

(2)校园地国家助学贷款(国家开发银行河北省分行承办目前仅在省属公办28所高校开办)。

贷款对象:家庭经济困难的全日制普通高校本专科生(含高职、第二学士学位、预科)和研究生。

申请方式:每年9月份,学生向所在高校申请。除以上介绍的国家助学贷款外,国家有关部门、高校、相关部门或团体、组织等,还有其他形式的助学、奖学方式,请考生注意向高校或有关部门咨询。




37. 高校如何进行新生入学复查?

按教育部规定,各普通高校在新生入学报到时须进行新生入学复查。依据录取相关材料、准考证、考生档案相关内容,进行身份复核、体检复查等。对特殊类型招生录取的新生,高校将组织专家组开展学专业复测和复核,对于专业测试不达标、入学前后两次测试成绩差异显著的情况,认真核实确认。对伪造材料取得报考资格者、冒名顶替者和体检舞弊及其他舞弊者,按照有关规定处理



38.  新生入 学 时 如 何购买合格的学 生公寓床上用 品 ? 

河北省高校的学生公寓管理部门为新生提供学生公寓床上用品代购服务,代购产品以进货价格销售。学校从河北省教育厅、河北省市场监督管理局共同组织的大中专学校学生公寓床上用品 集团购买活动上预订已取得河北省产品质量监督检验研究院出具产品质量合格报告的产品。新生购买时,要选择学校公寓管理部门设置的张贴产品质量合格报告的代销场所,不建议购买无产品质量合格报告的产品。按照教育部和国家市场监督管理总局有关  文件要求,为防止劣质床上用品流校园,公寓管理部门将对学生自购自带床上用品的品牌、生产企业等进行登记造册,如发现涉嫌质量问题产品,市场监管部门将提示学生进行更换,并对销售者进行查处。

在购买、使用过程中遇到问题,可与以下部门进行联系:

河北省学校后勤服务中心,电话:0311-66005763、66005765; 河北省产品质量监督检验研究院,电话:0311-67568286。

录取到省外高校的考生,相关问题请向录取高校咨询。




河北省2024年普通高校

招生工作新视点

 1.深化艺术类专业考试招生改革。按照教育部统一部署,2024年起,普通高校招生艺术类专业考试分为美术与设计类、音乐类(包括音乐表演、音乐教育两类,其中音乐表演类考试包括声乐、器乐两个方向)、舞蹈类、表(导)演类(包括戏剧影视表演、服装表演、戏剧影视导演三个方向)、播音与主持类、书法类、戏曲类等7个科类。其中,美术与设计类、音乐类、舞蹈类、表(导)演类、播音与主持类、书法类等6个科类实行省级统考;戏曲类实行省际联考。

2024年起,进一步提高文化课成绩要求,艺术史论、戏剧影视文学等部分艺术类专业,直接依据考生高考文化课成绩择优录取;美术与设计类、书法类、舞蹈类、表(导)演类、音乐类等5类省级统考的高考文化课成绩占比调整为50%,播音与主持类仍为70%;校考专业高考文化课成绩要求,调整为执行普通类专业批次录取控制分数线(有关破格录取的,按高校公布的考试招生办法执行)。严格控制校考范围,仅少数高校经教育部批准后,可在省级统考基础上组织校考。

2.改革高校高水平艺术团招生办法。2024年起,高校高水平艺术团不再从高校招生环节选拔,由相关高校从在校生中遴选培养。

3.深化高校高水平运动队考试招生改革。2024年起,进一步完善和规范高校高水平运动队考试招生工作,通过优化招生项目范围,严格报考条件和资格审核,改进考试评价方式,提高文化成绩要求,完善招生录取机制,选拔培养德智体美劳全面发展且具有较高体育竞技水平的学生,为奥运会、世界大学生运动会等重大体育比赛和国家竞技体育后备人才培养体系提供人才支撑。

4.高校选考科目优化调整。教育部印发的《普通高校本科招生专业选考科目要求指引(通用版)》,从2024年高考开始实施。这次高校选考科目优化调整,主要是强化了相关专业对高中学习物理、化学科目的基础性要求。学生可根据自身兴趣爱好及特长,统筹考虑国家和社会需要,结合拟报考高校及专业(类)的选考科目要求,合理确定选考科目。5.优化高职单招考试模式。2024年起,报考高职单招的考生在高考报名所在县(市、区)参加考试;普通高中毕业生文化素质不再使用高中学业水平合格性考试成绩折算替代,所有考生均须参加考试;高职单招报考缴费、打印准考证、查询成绩、填报志愿、查询录取结果等均须登录河北省教育考试院官方网站进行;志愿填报数量由5所院校增加至10所院校。

重要日程备忘

image.png


image.png


 


1.如何认定与处理考生的违规行为?

(一)按照《国家教育考试违规处理办法》(教育部令第33)规定,按以下情形认定考生的违规行为并进行处理:

(1)考生不遵守考场纪律,不服从考试工作人员的安排与要求,有下列行为之一的,应当认定为考试违纪:

①携带规定以外的物品进入考场或者未放在指定位置的;

②未在规定的座位参加考试的;

③考试开始信号发出前答题或者考试结束信号发出后继续答题的;

④在考试过程中旁窥、交头接耳、互打暗号或者手势的;

⑤在考场或者教育考试机构禁止的范围内,喧哗、吸烟或者实施其他影响考场秩序的行为的;

⑥未经考试工作人员同意在考试过程中擅自离开考场的;

⑦将试卷、答卷(含答题卡、答题纸等,下同)、草稿纸等考试用纸带出考场的;

⑧用规定以外的笔或者纸答题或者在试卷规定以外的地方书写姓名、考号或者以其他方式在答卷上标记信息的;

⑨其他违反考场规则但尚未构成作弊的行为。

考生有上述第(1)条所列考试违纪行为之一的,取消该科目的考试成绩。

(2)考生违背考试公平、公正原则,在考试过程中有下列行为之一的,应当认定为考试作弊:

①携带与考试内容相关的材料或者存储有与考试内容相关资料的电子设备参加考试的;

②抄袭或者协助他人抄袭试题答案或者与考试内容相关的资料的;

③抢夺、窃取他人试卷、答卷或胁迫他人为自己抄袭提供方便的;

④携带具有发送或者接收信息功能的设备的;

⑤由他人冒名代替参加考试的;

⑥故意销毁试卷、答卷或者考试材料的;

⑦在答卷上填写与本人身份不符的姓名、考号等信息的;

⑧传、接物品或者交换试卷、答卷、草稿纸的;

⑨其他以不正当手段获得或者试图获得试题答案、考试成绩的行为。

(3)教育考试机构、考试工作人员在考试过程中或者在考试结束后发现下列行为之一的,应当认定相关的考生实施了考试作弊行为:

①通过伪造证件、证明、档案及其他材料获得考试资格、加分资格和考试成绩的;

②评卷过程中被认定为答案雷同的;

③考场纪律混乱、考试秩序失控,出现大面积考试作弊现象的;

④考试工作人员协助实施作弊行为,事后查实的;

⑤其他应认定为作弊的行为。

考生有上述第(2)、(3)条所列考试作弊行为之一的,其所报名参加考试的各阶段、各科成绩无效。

(4)考生及其他人员应当自觉维护考试秩序,服从考试工作人员的管理,不得有下列扰乱考试秩序的行为:

①故意扰乱考点、考场、评卷场所等考试工作场所秩序;

②拒绝、妨碍考试工作人员履行管理职责;

③威胁、侮辱、诽谤、诬陷或者以其他方式侵害考试工作人员、其他考生合法权益的行为;

④故意损坏考场设施设备;

⑤其他扰乱考试管理秩序的行为。

考生有上述第(4)条所列行为之一的,应当终止其继续参 加本科目考试,其当次报名参加考试的各科成绩无效;考生及其他人员的行为违反《中华人民共和国治安管理处罚法》的,由公 安机关进行处理;构成犯罪的,由司法机关依法追究刑事责任。

(5)有下列情形之一的,可以视情节轻重,同时给予暂停参加该项考试1至3年的处理;情节特别严重的,可以同时给予暂停参加各种国家教育考试1至3年的处理:

①组织团伙作弊的;

②向考场外发送、传递试题信息的;

③使用相关设备接收信息实施作弊的;

④伪造、变造身份证、准考证及其他证明材料,由他人代替或者代替考生参加考试的。

(6)考生以作弊行为获得的考试成绩并由此取得相应的学位证书、学历证书及其他学业证书、资格资质证书或者入学资格的,由证书颁发机关宣布证书无效,责令收回证书或者予以没收;已经被录取或者入学的,由录取学校取消录取资格或者其学籍。

(7)在校学生、在职教师有下列情形之一的,教育考试机构 应当通报其所在学校,由学校根据有关规定严肃处理,直至开除学籍或者予以解聘:

①代替考生或者由他人代替参加考试的;

②组织团伙作弊的;

③为作弊组织者提供试题信息、答案及相应设备等参与团伙作弊行为的。

(二)按照《普通高等学校招生违规行为处理暂行办法》(教育部令第36号)规定,考生有下列情形之一的,应当如实记入其考试诚信档案。下列行为在报名阶段发现的,取消报考资格;在学前发现的,取消入学资格;入学后发现的,取消录取资格或者学籍;毕业后发现的,由教育行政部门宣布学历、学位证书无效,责令收回或者予以没收;涉嫌犯罪的,依法移送司法机关处理。

(1)提供虚假姓名、年龄、民族、户籍等个人信息,伪造、非法获得证件、成绩证明、荣誉证书等,骗取报名资格、享受优惠政策的;

(2)在综合素质评价、相关申请材料中提供虚假材料、影响录取结果的;

(3)冒名顶替学,由他人替考入学或者取得优惠资格的;

(4)其他严重违反高校招生规定的弄虚作假行为。

违反国家教育考试规定、情节严重受到停考处罚,在处罚结束后继续报名参加国家教育考试的,由学校决定是否予以录取。

教育部规定:教育考试机构应当建立国家教育考试考生诚信档案,记录、保留在国家教育考试中作弊人员的相关信息。国家教育考试考生诚信档案可以依申请接受社会有关方面的查询,并应当及时向招生学校或单位提供相关信息,作为招生参考条件。



 





 


2.  考试工作人员违纪舞弊应受何种处理? 

(一)按照《国家教育考试违规处理办法》(教育部令第33)规定,按以下情形认定考试工作人员的违规行为并进行处理:

(1)考试工作人员应当认真履行工作职责,在考试管理、组织及评卷等工作过程中,有下列行为之一的,应当停止其参加当年及下一年度的国家教育考试工作,并由教育考试机构或者建议其所在单位视情节轻重分别给予相应的行政处分:

①应回避考试工作却隐瞒不报的;

②擅自变更考试时间、地点或者考试安排的;

③提示或暗示考生答题的;

④擅自将试题、答卷或者有关内容带出考场或者传递给他人的;

⑤未认真履行职责,造成所负责考场出现秩序混乱、作弊严重或者视频录像资料损毁、视频考试系统不能正常工作的;

⑥在评卷、统分中严重失职,造成明显的错评、漏评或者积分差错的;

⑦在评卷中擅自更改评分细则或者不按评分细则进行评卷的;

⑧因未认真履行职责,造成所负责考场出现雷同卷的;

⑨擅自泄露评卷、统分等应予保密的情况的;

⑩其他违反监考、评卷等管理规定的行为。

(2)考试工作人员有下列作弊行为之一的,应当停止其参加 国家教育考试工作,由教育考试机构或者其所在单位视情节轻重分别给予相应的行政处分,并调离考试工作岗位;情节严重,构 成犯罪的,由司法机关依法追究刑事责任:

①为不具备参加国家教育考试条件的人员提供假证明、证件、档案,使其取得考试资格或者考试工作人员资格的;

②因玩忽职守,致使考生未能如期参加考试的或者使考试工作遭受重大损失的;

③利用监考或者从事考试工作之便,为考生作弊提供条件的;

④伪造、变造考生档案(含电子档案)的;

⑤在场外组织答卷、为考生提供答案的;

⑥指使、纵容或者伙同他人作弊的;

⑦偷换、涂改考生答卷、考试成绩或者考场原始记录材料的;

⑧擅自更改或者编造、虚报考试数据、信息的;

⑨利用考试工作便利,索贿、受贿、以权徇私的; 

⑩诬陷、打击报复考生的。

(3)对出现大规模作弊情况的考场、考点的相关责任人、负责人及所属考区的负责人,有关部门应当分别给予相应的行政处分;情节严重,构成犯罪的,由司法机关依法追究刑事责任。

(4)违反保密规定,造成国家教育考试的试题、答案及评分参考(包括副题及其答案及评分参考,下同)丢失、损毁、泄密,或者使考生答卷在保密期限内发生重大事故的,由有关部门视情节轻重,分别给予责任人和有关负责人行政处分;构成犯罪的,由司法机关依法追究刑事责任。

盗窃、损毁、传播在保密期限内的国家教育考试试题、答案及评分参考、考生答卷、考试成绩的,由有关部门依法追究有关人员的责任;构成犯罪的,由司法机关依法追究刑事责任。

(5)有下列行为之一的,由教育考试机构建议行为人所在单位给予行政处分;违反《中华人民共和国治安管理处罚法》的,由公安机关依法处理;构成犯罪的,由司法机关依法追究刑事责任:

①指使、纵容、授意考试工作人员放松考试纪律,致使考场秩序混乱、作弊严重的;

②代替考生或者由他人代替参加国家教育考试的;

③组织或者参与团伙作弊的;

④利用职权,包庇、掩盖作弊行为或者胁迫他人作弊的;

⑤以打击、报复、诬陷、威胁等手段侵犯考试工作人员、考生人身权利的;

⑥向考试工作人员行贿的;

⑦故意损坏考试设施的;

⑧扰乱、妨害考场、评卷点及有关考试工作场所秩序后果严重的。

国家工作人员有第(5)条所列行为的,教育考试机构应当建议有关纪检、监察部门,根据有关规定从重处理。

(二)按照《普通高等学校招生违规行为处理暂行办法》(教育部令第36号)规定,招生工作人员有下列情形之一的,其所在单位应当立即责令暂停其负责的招生工作,由有关部门视情节轻 重依法给予相应处分或者其他处理;涉嫌犯罪的,依法移送司法机关处理。

(1)违规更改考生报名、志愿、资格、分数、录取等信息的;

(2)对已录取考生违规变更录取学校或者专业的;

(3)在特殊类型招生中泄露面试考核考官名单或者利用职务便利请托考核评价的教师,照顾特定考生的;

(4)泄露尚未公布的考生成绩、考生志愿、录取分数线等可能影响录取公正信息的,或者对外泄露、倒卖考生个人信息的;

(5)为考生获得相关招生资格弄虚作假、徇私舞弊的;

(6)违反回避制度,应当回避而没有回避的;

(7)索取或收受考生及家长财物,接受宴请等可能影响公正履职活动安排的;

(8)参与社会中介机构或者个人非法招生活动的;

(9)其他影响高校招生公平、公正的行为。




3.  对教育考试机构作出的违规处理决定不服的如何申诉?

考生或者考试工作人员对教育考试机构作出的违规处理决定不服的,可以在收到处理决定之日起15日内,向其上一级教育考试机构提出复核申请;对省级教育考试机构或者承办国家教育考试的机构作出的处理决定不服的,也可以向省级教育行政部门或者授权承担国家教育考试的主管部门提出复核申请。

受理复核申请的教育考试机构、教育行政部门应对处理决定所认定的违规事实和适用的依据等进行审查,并在受理后30日内,按照下列规定作出复核决定:

(1)处理决定认定事实清楚、证据确凿,适用依据正确,程序合法,内容适当的,决定维持。

(2)处理决定有下列情况之一的,决定撤销或者变更:

①违规事实认定不清、证据不足的;

②适用依据错误的;

③违反本办法规定的处理程序的。

作出决定的教育考试机构对因错误的处理决定给考生造成的损失,应当予以补救。

申请人对复核决定或者处理决定不服的,可以依法申请行政复议或者提起行政诉讼。




 

 


 


4.  发现考生违规行为如何举报? 

在报名、体检、考试、录取期间,对违规行为可通过招生考试机构设置的举报电话、邮箱、举报箱等进行举报,也可以直接到当地招生考试机构或教育行政部门举报。


5.  违规招生的学校应受何种处理? 

按照《普通高等学校招生违规行为处理暂行办法》(教育部令第36号)规定,高校违反国家招生管理规定,有下列情形之一的,由主管教育行政部门责令限期改正,给予警告或者通报批评;情节严重的,给予减少招生计划、暂停特殊类型招生试点项目或者依法给予停止招生的处理。对直接负责的主管人员和其他直接责任人员,视情节轻重依法给予相应处分;涉嫌犯罪的,依法移送司法机关处理。

(1)发布违反国家规定的招生简章,或者进行虚假宣传、骗取钱财的:

(2)未按照信息公开的规定公开招生信息的;

(3)超出核定办学规模招生或者擅自调整招生计划的;

(4)违反规定降低标准录取考生或者拒绝录取符合条件的考生的;

(5)在特殊类型招生中出台违反国家规定的报考条件,或者弄虚作假、徇私舞弊,录取不符合条件的考生的;

(6)违规委托中介机构进行招生录取,或者以承诺录取为名向考生收取费用的;

(7)其他违反国家招生管理规定的行为。



6.  在艺术、体育、对口升学和高职单招等各类单招中违规的 考生如何处理?    

艺术、体育、对口升学和高职单招等各类单招考试是高考的

组成部分,依据教育部《国家教育考试违规处理办法》(教育部令第33号),考生在上述考试中被认定为违纪的,取消其该科目的考试成绩;被认定为作弊的,其当年高考报名参加考试的各阶段、各科成绩无效。


7.   《中华人民共和国刑法》涉及考试作弊入刑的 主要内容是什么?      

(1)在刑法第二百八十条之一后增加一条,第二百八十条之二:

盗用、冒用他人身份,顶替他人取得的高等学历教育人学资格、公务员录用资格、就业安置待遇的,处三年以下有期徒刑、拘役或者管制,并处罚金。

组织、指使他人实施前款行为的,依照前款的规定从重处罚。

国家工作人员有前两款行为,又构成其他犯罪的,依照数罪并罚的规定处罚。

(2)在刑法第二百八十四条后增加一条,作为第二百八十四条之一:

在法律规定的国家考试中,组织作弊的,处三年以下有期徒刑或者拘役,并处或者单处罚金;情节严重的,处三年以上七年 以下有期徒刑,并处罚金。

为他人实施前款犯罪提供作弊器材或者其他帮助的,依照前款的规定处罚。

为实施考试作弊行为,向他人非法出售或者提供第一款规定的考试的试题、答案的,依照第一款的规定处罚。

代替他人或者让他人代替自己参加第一款规定的考试的,处拘役或者管制,并处或者单处罚金。



8.   涉及高考作弊入刑的 主要内容是什么?  

普通高等学校招生考试及其特殊类型招生、特殊技能测试、面试等考试,均属于“法律规定的国家考试”。

在法律规定的国家考试中,组织作弊,具有下列情形之一的,应当认定为刑法第二百八十四条之一第一款规定的“情节严重”:

在普通高等学校招生考试中组织考试作弊的;导致考试推迟、取消或者启用备用试题的;考试工作人员组织考试作弊的;组织考生跨省、自治区、直市作弊的;多次组织考试作弊的;组织三十人次以上作弊的;提供作弊器材五十件以上的;违法所 得三十万元以上的;其他情节严重的情形。

具有避开或者突破考场防范作弊的安全管理措施,获取、记录、传递、接收、存储考试试题、答案等功能的程序、工具,以及专门设计用于作弊的程序、工具,应当认定为刑法第二百八十四条之一第二款规定的“作弊器材”。

对于是否属于刑法第二百八十四条之一第二款规定的“作弊器材”难以确定的,依据省级以上公安机关或者考试主管部门出具的报告,结合其他证据作出认定;涉及专用间谍器材、窃听、窃照专用器材、“伪基站”等器材的,依照相关规定作出认定。

组织考试作弊,在考试开始之前被查获,但已经非法获取考试试题、答案或者具有其他严重扰乱考试秩序情形的,应当认定为组织考试作弊罪既遂。

为实施考试作弊行为,非法出售或者提供法律规定的国家考试的试题、答案,具有下列情形之一的,应当认定为刑法第二百八十四条之一第三款规定的“情节严重”:

非法出售或者提供普通高等学校招生考试的试题、答案的;导致考试推迟、取消或者启用备用试题的;考试工作人员非法出 售或者提供试题、答案的;多次非法出售或者提供试题、答案的;向三十人次以上非法出售或者提供试题、答案的;违法所得三十万元以上的;其他情节严重的情形。

为实施考试作弊行为,向他人非法出售或者提供法律规定的国家考试的试题、答案,试题不完整或者答案与标准答案不完全一致的,不影响非法出售、提供试题、答案罪的认定。

代替他人或者让他人代替自己参加法律规定的国家考试的,应当依照刑法第二百八十四条之一第四款的规定,以代替考试罪定罪处罚。

对于行为人犯罪情节较轻,确有悔罪表现,综合考虑行为人替考情况以及考试类型等因素,认为符合缓刑适用条件的,可以宣告缓刑。

单位实施组织考试作弊、非法出售、提供试题、答案等行为的,依照本解释规定的相应定罪量刑标准,追究组织者、策划者、实施者的刑事责任。以窃取、刺探、收买违法非法获取法律规定的国家考试的试题、答案,又组织考试作弊或者非法出售、提供试题、答案,分别符合刑法第二百八十二条和刑法第二百八十四条之一规定的,以非法获取国家秘密罪和组织考试作弊罪或 者非法出售、提供试题、答案罪数罪并罚。

设立用于实施考试作弊的网站、通讯群组或者发布有关考试作弊的信息,情节严重的,应当依照刑法第二百八十七条之一的规定,以非法利用信息网络罪定罪处罚;同时构成组织考试作弊罪,非法出售、提供试题、答案罪,非法获取国家秘密罪等其他犯罪的,依照处罚较重的规定定罪处罚。




 


 


9. 《中国共产党纪律处分条例》涉及考试招生违规处理的主要内容是什么?          

.《中国共产党纪律处分条例》涉及考试招生违规处理的主要内容如下:

第一百二十一条 在考试、录取工作中,有泄露试题、考场舞弊、涂改考卷、违规录取等违反有关规定行为的,给予警告或者严重警告处分;情节较重的,给予撤销党内职务或者留党察看处分;情节严重的,给予开除党籍处分。


    10.   《中华人民共和国教育法》涉及考试招生违规处理的 主要内容是什么?             

第七十九条考生在国家教育考试中有下列行为之一的,由组织考试的教育考试机构工作人员在考试现场采取必要措施予以制止并终止其继续参加考试;组织考试的教育考试机构可以取消 其相关考试资格或者考试成绩;情节严重的,由教育行政部门责令停止参加相关国家教育考试一年以上三年以下;构成违反治安管理行为的,由公安机关依法给予治安管理处罚;构成犯罪的,依法追究刑事责任:

(一)非法获取考试试题或者答案的;

(二)携带或者使用考试作弊器材、资料的;

(三)抄袭他人答案的;

(四)让他人代替自己参加考试的;

(五)其他以不正当手段获得考试成绩的作弊行为。

第八十条  任何组织或者个人在国家教育考试中有下列行为之一,有违法所得的,由公安机关没收违法所得,并处违法所得一倍以上五倍以下罚款;情节严重的,处五日以上十五日以下拘留;构成犯罪的,依法追究刑事责任;属于国家机关工作人员的,还应当依法给予处分:

(一)组织作弊的;

(二)通过提供考试作弊器材等方式为作弊提供帮助或者便利的;

(三)代替他人参加考试的;

(四)在考试结束前泄露、传播考试试题或者答案的;

(五)其他扰乱考试秩序的行为。

第八十一条 举办国家教育考试,教育行政部门、教育考试机构疏于管理,造成考场秩序混乱、作弊情况严重的,对直接负资的主管人员和其他直接资任人员,依法给予处分;构成犯罪的,依法追究刑事责任。